Определите, в каком формате записаны координаты указанной точки (N59°45.123’ E32°31.683’):
Градусы минуты и секунды;
Градусы и доли градуса;
Градусы минуты и доли минут;
Ошибочная запись.
Правильный ответ на вопрос №103: № 3. Ситуация, в результате которой зона чрезвычайной ситуации не выходит за пределы территории одного субъекта Российской Федерации, при этом количество людей, погибших и (или) получивших ущерб здоровью, составляет свыше 50 человек, но не более 500 человек либо размер материального ущерба;
Пояснение к ответу на вопрос № 103
Здравствуйте, уважаемые участники Марафона!
Форматы записи географических координат (форматы GPS-координат)
Для записи географических координат может использоваться любой эллипсоид (или геоид), но чаще всего используются WGS 84 и Красовского (на территории РФ).
Координаты (широта от −90° до +90°, долгота от −180° до +180°) могут записываться:
в градусах (обозначается символом ° ) в виде десятичной дроби (современный вариант) (например, 54.97158, 73.38318)
в градусах (обозначается символом ° ) и минутах (обозначается символом ′ ) с десятичной дробью (например, 54°58.295′, 73°22.991′)
в градусах (обозначается символом ° ), минутах (обозначается символом ′ ) и секундах (обозначается символом ″ ) с десятичной дробью (исторически сложившаяся форма записи) (например, 54°58’17.7″, 73°22’59.4″)
другие варианты. Но они почти не используются в гражданских навигационных устройствах.
Разделителем десятичной дроби может служить точка или запятая. Положительные знаки координат представляются (в большинстве случаев опускаемым) знаком «+» либо буквами:
«N» или «с. ш.» — северная широта,
«E» или «в. д.» — восточная долгота.
Отрицательные знаки координат представляются либо знаком «−», либо буквами:
«S» или «ю. ш.» — южная широта,
«W» или «з. д. — западная долгота.
Буквы могут стоять как впереди, так и сзади. Единых правил записи координат не существует.
На картах поисковых систем по умолчанию показываются координаты в градусах с десятичной дробью со знаком «−» для отрицательной долготы. На картах Google и картах Яндекс вначале широта, затем долгота (до октября 2012 на картах Яндекс был принят обратный порядок: сначала долгота, потом широта). Эти координаты видны, например, при прокладке маршрутов от произвольных точек. При поиске распознаются и другие форматы. Хотя я вам могут дать такой совет. Если вам надо увидеть место только на Яндекс картах, Google картах или самым простых стилях OSM, то проще всего открыть сайты соответствующих карт, вставить в строку поиска координаты (или просто название нужного вам места) и нажать «поиск» (лупу) или Enter. Сайт покажет вам, что находится на этом месте. И не надо заниматься переводом из одних координат в другие.
В то же время часто используется и исконный способ записи с градусами, минутами и секундами. В настоящее время координаты могут записываться одним из множества способов или дублироваться двумя основными (с градусами и с градусами, минутами и секундами). Как пример, варианты записи координат знака «Нулевой километр автодорог Российской Федерации» — 55°45′21″ с. ш. 37°37′04″ в. д.
55,755831°, 37,617673° — градусы
N55.755831°, E37.617673° — градусы (+ доп. буквы)
55°45.35′N, 37°37.06′E — градусы и минуты (+ доп. буквы)
55°45′20.9916″N, 37°37′3.6228″E — градусы, минуты и секунды (+ доп. буквы)
Иногда необходимо один тип координат, а есть только другой. В этом случае их можно пересчитать в другой тип координат. Вот как это можно сделать
Для преобразования из градусов в градусы-минуты
Целая часть градусов переписывается как есть. Дробная часть градусов (она всегда меньше 1) умножается на 60, получается значение минут.
Пример: Имеются координаты N60.5643 E30.7543. Широта: Целая часть 60, дробная часть 0. 5643. Записываем целую часть N60°; умножаем 0.5643*60=33.858; Получается N60°33.858′. То же самое с со второй координатой, долготой: E30°; 0.7543*60=45.258 Итоговые координаты: N60°33.858′ E30° 45.258′
Для преобразования из градусы-минуты в градусы
Целая часть градусов переписывается как есть. Минуты (они всегда меньше 60) делятся на 60, и получается значение сотых долей градуса.
Пример: Имеются координаты N60°33.858′ E30° 45.258′ Широта: Целая часть 60, минуты 33.858 Записываем целую часть N60°; Делим значение минут 33.858/60=0.5643; Получается N60.5643° То же самое с со второй координатой, долготой: E30°; 45.258/60=0.7543 Итоговые координаты: N60.5643° E30.7543°
Для преобразования из градусы-минуты в градусы-минуты-секунды
Значение градусов переписывается как есть. Целая часть минут переписывается как есть. Дробная часть минут (она всегда меньше 1) умножается на 60, получается значение секунд.
Пример: Имеются координаты N60°33.858′ E30° 45.258′. Широта: Целая часть градусов 60, целая часть минут 33, дробная часть минут 0.858 Записываем целые часть N60° 33′; умножаем 0.858*60=51.48; Получается N60°33′ 51.48″. То же самое с со второй координатой, долготой: E30° 45′; 0.258*60=15.48 Итоговые координаты: N60°33′ 51.48″ E30° 45′ 15.48″
Для преобразования из градусы-минуты-секунды в градусы-минуты
Значение градусов переписывается как есть. Значение минут переписывается как есть. Значение секунд (оно всегда меньше 60) делится на 60, получается дробная часть минут.
Пример: Имеются координаты N60°33′ 51.48″ E30° 45′ 15.48″ Широта: значение градусов 60, значение минут 33, значение секунд 51.48 Записываем целые часть N60° 33′; делим 51.48/60=0.858; Получается N60°33.858′. То же самое с со второй координатой, долготой: E30° 45′; 15.48 /60=0.258; Итоговые координаты: N60°33. 858′ E30° 45.258′.
Преобразование градусы в градусы-минуты-секунды и обратно производится через преобразование в градусы-минуты (т.е необходимо произвести два пересчета). Точность координат при любом пересчете не изменяется.
Если у вас возникли вопросы или вы хотите обсудить данный ответ вы можете написать в обсуждениях в нашей группе в ВКонтакте по этой ссылке.
Тех, кто понимает, что читать, смотреть, знать и УМЕТЬ — это разные понятия, приглашаем на курсы в наш Учебный центр. Мы проводим только реальные практические тренинги и мастер-классы, тренинги и курсы по первой помощи и безопасности жизнедеятельности! Никаких скучных лекций, только передача опыта и навыков!
Официальный сайт: https://ucmchs.ru Официальный источник: https://ucmchs.ru/diffs/blog/ Youtube канал: https://www.youtube.com/channel/UCXpIFPBUXa1jwR6Yhi9IalQ Мы в социальных сетях: https://vk.com/ucmchs Найти нас можно #ucmchs , #уцмчс , #КурсыПервойПомощи
градусов/минут/секунд (DMS) против десятичных градусов (DD)
Для позиционирования мы можем найти любое место на Земле, используя координаты широты и долготы.
И мы измеряем эти координаты, используя десятичных градусов или градусов/минут/секунд .
В то время как линии широты находятся в диапазоне от -90 до +90 градусов, координаты долготы находятся в диапазоне от -180 до +180 градусов.
Вы заметили, как мы используем градусов для координат широты и долготы? Давайте начнем с некоторых ключевых примеров того, как мы используем угловые единицы.
Обзор географических систем координат
В географической системе координат (ГСК) мы можем ссылаться на любую точку на Земле по ее координатам долготы и широты. Поскольку GCS использует сферу для определения местоположения на Земле, мы используем углы, измеряемые в градусах от центра Земли до любой точки на поверхности.
Координаты (0°N, 0°E) — это пересечение экватора и нулевого меридиана. Самое смешное, что если вы посмотрите на это место на карте, то увидите, что это весь океан.
Но из-за того, что специалисты по ГИС иногда ошибочно определяют свой проект при добавлении координат XY, (0°N, 0°E) превратилось в вымышленное место, называемое «нулевым островом».
Когда мы движемся на север вдоль нулевого меридиана, значение долготы остается фиксированным на 0°. Но угол широты и координата увеличиваются, потому что мы движемся на север.
Если двигаться на север под углом 51,5°, вы окажетесь в Королевской обсерватории в Гринвиче, Англия, как показано на рисунке ниже. Собственно, поэтому линия долготы 0° является исходной точкой отсчета. От Гринвичского меридиана мы можем найти позиции на восток и запад.
Поскольку нулевой меридиан является начальной точкой координат долготы 0°, отсюда все ссылки.
Например, мы можем изменить угол на 80,4° на запад. Это смещает нас на 80,4° западной долготы от нулевого меридиана. И совершенно случайно Питтсбург расположен на этой линии долготы примерно на (40,4° с.ш., 80,4° з.д.)
Напомним:
Экватор — это 0° широты, где мы измеряем север и юг. Это означает, что все к северу от экватора имеет положительные значения широты. Принимая во внимание, что все к югу от экватора имеет отрицательные значения широты.
В качестве альтернативы, Гринвичский меридиан (или нулевой меридиан) — это нулевая линия долготы, от которой мы измеряем восток и запад.
Десятичные градусы и градусы/минуты/секунды
Одним из способов записи сферических координат (широт и долгот) является использование градусов-минут-секунд (ГМС). Минуты и секунды находятся в диапазоне от 0 до 60. Например, географическая координата Нью-Йорка, выраженная в градусах-минутах-секундах:
ШИРОТА: 40 градусов, 42 минуты, 51 секунда N ДОЛГОТА: 74 градуса, 0 минут, 21 секунда W
Но вы также можете выразить географические координаты в десятичных градусах. Это просто еще один способ представить то же самое место в другом формате. Например, это Нью-Йорк в десятичных градусах:
ШИРОТА: 40,714 ДОЛГОТА: -74,006
десятичные градусы и градусы/минуты/секунды.
Попробуй сам
Если сложить вместе две координаты в виде пары (X, Y), вы сможете найти что угодно на Земле в географической системе координат.
Что угодно!
Вы можете выразить координаты двумя способами. Например, вы можете использовать десятичные градусы или градусы-минуты-секунды. Но есть даже новые растущие способы обращения к миру, такие как What3Words.
После того, как ваши местоположения находятся в GCS, географы часто проецируют свои местоположения в Projected Coordinate System (PCS). PCS, такие как State Plane Coordinate System (SPCS) или Universal Transverse Mercator (UTM), всегда основаны на GCS, основанной на сфере.
Узнайте больше о картографических проекциях и о том, как проецировать данные.
Градусы-минуты-секунды, десятичные градусы, а также восточное и северное направления – пространственно оспариваемые
Опубликовано от matcham2.0
В этом году я работал над двумя испытаниями удобрений в разных штатах. Как правило, в проектах такого типа сотрудники из разных лабораторий договариваются применять одни и те же процедуры, проводить одни и те же измерения и отправлять данные одному человеку для анализа и публикации. Я получаю странное удовлетворение (а иногда и разочаровывающую) работу по форматированию данных единым образом.
Чтобы упростить задачу, мы отправляем каждому соавтору электронную таблицу, в которой есть место для всей информации, необходимой для анализа данных. Соавторы просто заполняют электронную таблицу своими данными в перечисленных единицах и отправляют ее обратно в лабораторию, систематизируя данные.
Эта система работает очень хорошо для большинства типов данных. Иногда соавторы предоставляют данные в единицах, отличных от форм запросов, но они постоянно отмечают, какие единицы они использовали, чтобы мы могли легко преобразовать их в единицы, используемые другими сайтами.
Единственным исключением являются широта и долгота. Соавторы постоянно сообщают свое местоположение в градусах, минутах и секундах, когда мы запрашиваем GPS-координаты в десятичных градусах. В этом году я узнал, что есть группа людей, которые думают, что если вы включаете десятые или сотые доли секунды в формате градусы-минуты-секунды, это означает, что это десятичные градусы. Эта запись в блоге предназначена для разъяснения различных форматов координат и предоставления некоторых ресурсов для преобразования между ними.
Существует три основных формата: градусы-минуты-секунды, десятичные градусы и восточное и северное направление. В формате градусы-минуты-секунды координаты лагеря Рэндалл составляют 43°04’06.0″N 89°24’46.4″W. Каждый градус делится на 60 минут, а каждая минута делится на 60 секунд. Таким образом, чтобы преобразовать 43°04’06,0″ с.ш. в формат десятичных градусов, нужно взять 43 + (4/60) + (6/3600) = 43,06833° с.ш. Используя тот же метод для преобразования долготы, вы получите 89,41289° з. д. , У NOAA есть отличный инструмент преобразования, если вы не хотите конвертировать их вручную. Этот более простой, но тоже хороший.
Восток и север используются реже, но для пространственного анализа они полезны. Восток и север основаны на картографической проекции Universal Transverse Mercator (узнайте о проекциях здесь), которая делит земной шар на 60 зон. Центральный меридиан (вертикальная линия долготы) каждой зоны имеет восточное направление на 500 000 метров, а экватор всегда имеет северное направление на 10 000 000 метров. За каждый метр, который вы перемещаете к северу от экватора или к востоку от меридиана, вы увеличиваете направление на север и восток на один метр соответственно. Восток и север Кэмп-Рэндалл — 303543,38 в.д., 4771228,45 с.ш. в зоне 16T. Я обычно использую этот конвертер.
Я настоятельно предпочитаю получать данные в десятичных градусах, потому что меньше вещей нужно очищать — мне не нужно проверять, что все пробелы между символом градуса и количеством минут удалены, и мне не нужно делать убедитесь, что минуты с одной цифрой имеют ноль впереди.
Функция — округление x в меньшую сторону (пример floor(4.5)==4.0)
ceiling(x)
Функция — округление x в большую сторону (пример ceiling(4.5)==5.0)
sign(x)
Функция — Знак x
erf(x)
Функция ошибок (или интеграл вероятности)
laplace(x)
Функция Лапласа
asech(x)
Функция — гиперболический арксеканс от x
csch(x)
Функция — гиперболический косеканс от x
sech(x)
Функция — гиперболический секанс от x
acsch(x)
Функция — гиперболический арккосеканс от x
Постоянные:
pi
Число «Пи», которое примерно равно ~3. 14159..
e
Число e — основание натурального логарифма, примерно равно ~2,7183..
i
Комплексная единица
oo
Символ бесконечности — знак для бесконечности
Cos 2 градусов — Найти значение Cos 2 градусов
LearnPracticeDownload
Значение cos 2 градусов равно 0,9993908. . . . Cos 2 градуса в радианах записывается как cos (2° × π/180°), т. е. cos (π/90) или cos (0,034906…). В этой статье мы обсудим способы нахождения значения cos 2 градусов на примерах.
Cos 2°: 0,9993908. . .
Cos (-2 градуса): 0,9993908. . .
Cos 2° в радианах: cos (π/90) или cos (0,0349065 . . . .)
Каково значение Cos 2 градусов?
Значение cos 2 градуса в десятичной системе равно 0,9993
. . .. Cos 2 градуса также можно выразить с помощью эквивалента данного угла (2 градуса) в радианах (0,03490 . . .)
Мы знаем, используя преобразование градусов в радианы, что θ в радианах = θ в градусах × (pi/ 180°) ⇒ 2 градуса = 2° × (π/180°) рад = π/90 или 0,0349. . . ∴ cos 2 ° = cos (0,0349) = 0,9993908. . .
Объяснение:
Для cos 2 градуса угол 2° лежит между 0° и 90° (первый квадрант). Поскольку функция косинуса положительна в первом квадранте, значение cos 2° = 0,9993908. . . Поскольку функция косинуса является периодической функцией, мы можем представить cos 2° как cos 2 градусов = cos(2° + n × 360°), n ∈ Z. ⇒ cos 2° = cos 362° = cos 722° и так далее. Примечание: Поскольку косинус является четной функцией, значение cos(-2°) = cos(2°).
Методы определения значения косинуса 2 градуса
Функция косинуса положительна в 1-м квадранте. Значение cos 2° составляет 0,99939. . .. Мы можем найти значение cos 2 градуса по:
Используя единичный круг
Использование тригонометрических функций
Cos 2 градуса с использованием единичной окружности
Чтобы найти значение cos 2 градуса с помощью единичной окружности:
Поверните ‘r’ против часовой стрелки, чтобы образовать угол 2° с положительной осью x.
Кос 2 градусов равен координате x (0,9994) точки пересечения (0,9994, 0,0349) единичной окружности и r.
Отсюда значение cos 2° = x = 0,9994 (приблизительно)
Cos 2° в терминах тригонометрических функций
Используя формулы тригонометрии, мы можем представить cos 2 градусов как:
± √(1-sin² (2°))
± 1/√(1 + tan²(2°))
± раскладушка 2°/√(1 + раскладушка²(2°))
±√(косек²(2°) — 1)/косек 2°
1/сек 2°
Примечание. Поскольку 2° лежит в 1-м квадранте, окончательное значение cos 2° будет положительным.
Мы можем использовать тригонометрические тождества для представления cos 2° как
-cos(180° — 2°) = -cos 178°
-cos(180° + 2°) = -cos 182°
sin(90° + 2°) = sin 92°
sin(90° — 2°) = sin 88°
☛ Также проверьте:
cos 11 градусов
соз 60 градусов
потому что 135 градусов
потому что 585 градусов
потому что 145 градусов
потому что 67 градусов
Примеры использования Cos 2 градусов
Пример 1. Найдите значение 2 cos(2°)/3 sin(88°).
Решение:
Используя тригонометрические тождества, мы знаем, что cos(2°) = sin(90° — 2°) = sin 88°. ⇒ cos(2°) = sin(88°) ⇒ Значение 2 cos(2°)/3 sin(88°) = 2/3
Пример 2: Упростить: 4 (cos 2°/sin 92°)
Решение:
Мы знаем, что cos 2° = sin 92° ⇒ 4 cos 2°/sin 92° = 4 (cos 2°/cos 2°) = 4(1) = 4
Пример 3: Найдите значение (cos² 1° — sin² 1°). [Подсказка: используйте cos 2° = 0,9994]
Решение:
Используя формулу cos 2a, (cos² 1° — sin² 1°) = cos(2 × 1°) = cos 2° ∵ cos 2° = 0,9994 ⇒ (cos² 1° — sin² 1°) = 0,9994
перейти к слайдуперейти к слайдуперейти к слайду
Готовы увидеть мир глазами математика?
Математика лежит в основе всего, что мы делаем. Наслаждайтесь решением реальных математических задач на живых уроках и станьте экспертом во всем.
Забронируйте бесплатный пробный урок
Часто задаваемые вопросы по Cos 2 Degrees
Что такое Cos 2 градуса?
Cos 2 градуса — значение тригонометрической функции косинуса для угла, равного 2 градусам. Значение cos 2° равно 0,9994 (приблизительно)
Каково точное значение cos 2 градусов?
Точное значение cos 2 градуса может быть задано с точностью до 8 знаков после запятой как 0,99939082.
Как найти значение Cos 2 градуса?
Значение cos 2 градуса можно рассчитать, построив угол 2° с осью x, а затем найдя координаты соответствующей точки (0,9994, 0,0349) на единичной окружности. Значение cos 2° равно координате x (0,9994). ∴ cos 2° = 0,9994.
Каково значение Cos 2 градусов относительно Cot 2°?
Мы можем представить функцию косинуса в терминах функции котангенса, используя тригонометрические тождества, cos 2° можно записать как cot 2°/√(1 + cot²(2°)). Здесь значение cot 2° равно 28,63625.
Как найти Cos 2° с точки зрения других тригонометрических функций?
Используя формулу тригонометрии, значение cos 2° может быть выражено через другие тригонометрические функции следующим образом:
± √(1-sin²(2°))
± 1/√(1 + tan²(2°))
± раскладушка 2°/√(1 + раскладушка²(2°))
± √(косек²(2°) — 1)/косек 2°
1/сек 2°
☛ Также проверьте: таблицу тригонометрии
Скачать БЕСПЛАТНЫЕ учебные материалы
Тригонометрия
Рабочие листы по математике и наглядный учебный план
Python 3 — это современный язык, на котором просто и приятно писать программы.
Для печати значений в Питоне есть функция print(). Внутри круглых скобок через запятую мы пишем то, что хотим вывести. Вот программа, которая делает несколько вычислений:
print(5 + 10)
print(3 * 7, (17 - 2) * 8)
print(2 ** 16) # две звёздочки означают возведение в степень
print(37 / 3) # один слэш — это деление с ответом-дробью
print(37 // 3) # два слэша считают частное от деления нацело
# это как операция div в других языках
print(37 % 3) # процент считает остаток от деления нацело
# это как операция mod в других языках
Для ввода данных в программу мы используем функцию input(). Она считывает одну строку.
Вот программа, которая считывает имя пользователя и приветствует его:
print('Как вас зовут?')
name = input() # считываем строку и кладём её в переменную name
print('Здравствуйте, ' + name + '!')
Мы будем писать программы, которые считывают данные, перерабатывают их и выводят какой-то результат. При запуске на компьютере такие программы считывают данные, которые пользователь вводит с клавиатуры, а результат выводят на экран.
Попробуем написать программу, которая считывает два числа и выводит их сумму. Для этого считаем два числа и сохраним их в переменные a и b, пользуясь оператором присваивания =. Слева от оператора присваивания в программах на Питоне ставится имя переменной — например, строка из латинских букв. Справа от оператора присваивания ставится любое выражение. Имя станет указывать на результат вычисления выражения. Проиграйте эту программу и посмотрите на результаты её работы:
a = input()
b = input()
s = a + b
print(s)
Мы видим, что программа выводит 57, хотя в реальной жизни 5 + 7 будет 12. Это произошло потому, что Питон в третьей строчке «сложил» две строки, а не два числа. В Питоне две строки складываются так: к первой строке приписывается вторая.
Обратите внимание, что в визуализаторе содержимое переменных a и b заключено в кавычки. Это означает, что в a и b лежат строки, а не числа.
В Питоне все данные называются объектами. Число 2 представляется объектом «число 2», строка 'hello' – это объект «строка 'hello'».
Каждый объект относится к какому-то типу. Строки хранятся в объектах типа str, целые числа хранятся в объектах типа int, дробные числа (вещественные числа) — в объектах типа float. Тип объекта определяет, какие действия можно делать с объектами этого типа. Например, если в переменных first и second лежат объекты типа int, то их можно перемножить, а если в них лежат объекты типа str, то их перемножить нельзя:
first = 5
second = 7
print(first * second)
first = '5'
second = '7'
print(first * second)
Чтобы преобразовать строку из цифр в целое число, воспользуемся функцией int(). Например, int('23') вернет число 23.
Вот пример правильной программы, которая считывает два числа и выводит их сумму:
a = int(input())
b = int(input())
s = a + b
print(s)
Ссылки на задачи доступны в меню слева. Эталонные решения теперь доступны на странице самой задачи.
3-8
9
Оценить
квадратный корень из 12
10
Оценить
квадратный корень из 20
11
Оценить
квадратный корень из 50
94
18
Оценить
квадратный корень из 45
19
Оценить
квадратный корень из 32
20
Оценить
квадратный корень из 18
92
Как найти решение системы уравнений
Все математические ресурсы SAT
16 диагностических тестов
660 практических тестов
Вопрос дня
Карточки
Learn by Concept
← Предыдущая 1 2 3 4 5 6 7 8 9 Следующая →
SAT Math Help »
Алгебра »
Уравнения / Неравенства »
Системы уравнений »
Как найти решение системы уравнений
Если 7 х + у = 25 и 6 х + у = 23, каково значение х ?
Возможные ответы:
2
11
6
7
20
Правильный ответ:
2
Объяснение:
Вы можете вычесть второе уравнение из первого, чтобы исключить y :
7 x + y = 25 – 6 x + y = 23: 7 x – 6 x = x ; г – г = 0; 25 – 23 = 2
x = 2
Вы также можете решить одно уравнение для y и подставить это значение вместо y в другое уравнение:
6 x 9091 2 + у = 23 → у = 23 – 6 x .
7 x + y = 25 → 7 x + (23 – 6 x ) = 25 → x + 23 = 25 → x = 2
Сообщить об ошибке 928 Возможные ответы :
27
6
16
31
Правильный ответ:
31
Пояснение:
Мы можем сложить эти уравнения друг с другом.
(7x + 3y = 20) + (–4x – 6y = 11) = (3x – 3y = 31)
Сообщить об ошибке
Рассмотрим три прямые, заданные следующими уравнениями:
x + 2y = 1
y = 2x + 3
4x — 3y = 2
Каково значение координаты x точки пересечения, является общим для ВСЕХ трех строк?
Возможные ответы:
Нет точки пересечения 0929 Правильный ответ: Нет точки пересечения
Объяснение:
Если точка пересечения лежит на всех трех линиях, то мы должны иметь возможность выбрать любые две линии, найти их точку пересечения и каждый раз получать одну и ту же точку пересечения. Другими словами, точка пересечения первых двух линий должна быть точкой пересечения второй и третьей линий.
Рассмотрим первую и вторую линии. Мы можем решить систему уравнений, подставив значение y из второго уравнения в первое.
у = 2х + 3
х + 2(2х + 3) = 1
х + 4х + 6 = 1
5х = -5
х = -1
у = 2(- 1) + 3 = 1
Точка пересечения первых двух линий (-1,1).
Теперь мы можем найти точку пересечения второй и третьей линий. Опять же, мы можем подставить значение y из второго уравнения в третье.
y = 2x + 3
4x — 3(2x + 3) = 2
4x -6x — 9 = 2
-2x = 11
x = -11/2
y = 2(-11/2)+3 = -8
Таким образом, вторая и третья прямые пересекаются в точке (-11/2,-8).
Поскольку точка пересечения первой и второй линий не совпадает с точкой пересечения второй и третьей линий, не существует точки, общей для ВСЕХ трех линий. Таким образом, точки пересечения нет.
Сообщить об ошибке
У каждой овцы 4 ноги, а у каждой курицы 2 ноги. Если мальчик с фермы насчитал 50 голов и 140 футов, сколько у него овец?
Возможные ответы:
30 овец
10 овец
25 овец
15 овец
20 овец
90 929 Правильный ответ:
20 овец
Объяснение:
Установите x как количество овец и y как количество цыплят. Это дает нам x+y=50 и 4x+2y=140. Мы хотим найти х. Решая первое уравнение, получаем y=50-x. Подставьте это во второе, и у вас получится 4x+2(50-x)=140. Умножение дает 4x+100-2x=140. Итак, 2х+100=140. 2х=40, х=20. Отдам 20 овец.
Сообщить об ошибке
Если 8x – 9 на 10 меньше 5, каково значение 4x?
Возможные ответы:
1
4
2
1/4
1/2
Правильный ответ :
2
Пояснение:
Первое, что необходимо сделать, это написать алгебраическое уравнение для задачи:0909
Таким образом, 4 * x = 2
Сообщить об ошибке
4x — 5y = 12
6y — 3x = -6
Количество A: x + y
Количество B: 6
Возможные ответы:
Количество A больше
Количество B больше
Отношение не может быть определено из предоставленной информации 09
Объяснение:
Сложите два уравнения:
4x — 5y = 12 плюс
6y — 3x = -6:
4x — 5y + (6y — 3x) = 12 + (-6)
4x — 3x + 6 лет — 5y = 12 — 6
x + y = 6
Сообщить об ошибке
Благотворительная организация набирает добровольцев для подготовки к сбору средств. Каждый волонтер может либо помочь установить столы, либо аукционные галереи. Волонтер может настроить 6 столов в час или 2 аукционные галереи в час. Есть 180 столов, которые нужно настроить, а также 12 аукционных галерей. Если у волонтеров будет 3 часа на подготовку, сколько волонтеров нужно записать?
Возможные ответы:
30
10
8
15
12
Правильный ответ: 12
Пояснение:
Узнайте, сколько волонтер может произвести за 3 часа.
6 столов/час * 3 часа = 18 столов/час
Необходимо настроить 180 столов. Если один доброволец может настроить 18 столов за 3 часа, то 10 добровольцев позаботятся о 180 столах.
Найдите x, затем подставьте в формулу, чтобы найти значение. x = 28 – 12 = 16
(3 * 16 + 2) * (–16 +10) = –300
Сообщить об ошибке
У Джои есть 1,50 доллара США. Если у него есть только четвертак и пятак, а всего у него 14 монет, сколько у него пятаков?
Возможные ответы:
10
3
6
5
8
Правильный ответ: 9 0930
10
Объяснение:
Установка x и количества четвертаков, которые у него есть, и y в качестве количества пятицентовых монет. x + y = 14 (всего монет), 0,25x + 0,05y = 1,50 (всего). Подставив x = 14 – y из первого уравнения во второе, мы получим y = 10. Следовательно, у Джоуи 10 пятицентовых монет.
Сообщить об ошибке
Футболист бьет по мячу со скоростью 8 м/с. Игрок бежит, чтобы получить его, как только мяч отброшен ногой со скоростью 4 м/с. Если принимающий игрок стартует за 12 м до мяча, какое расстояние он пройдет, прежде чем получит мяч?
Возможные ответы:
3 м
6 м
12 м
15 м
9 м
Правильный ответ:
12м
Объяснение:
Установив t как прошедшее время, нам нужно найти, когда 8t = 12 + 4t (это расстояние, пройденное мячом, по сравнению с расстоянием, пройденным игроком + разница с исходной точкой).
8.2. Числовые характеристики непрерывных случайных величин
Математическое ожидание непрерывной случайной величины Х, возможные значения которой принадлежат всей оси Ох, определяется равенством
Где Р(Х) — плотность распределения случайной величины Х. Предполагается, что интеграл сходится абсолютно. В частности, если все возможные значения принадлежат интервалу , то
Дисперсия непрерывной случайной величины Х, возможные значения которой принадлежат всей оси Ох определяется равенством
Если интеграл сходится, или равносильным равенством
В частности, если все возможные значения Х принадлежат интервалу , то
Или
Все свойства математического ожидания и дисперсии для дискретных случайных величин справедливы и для непрерывных величин.
Среднее квадратическое отклонение непрерывной случайной величины определяется равенством
.
Модой непрерывной случайной величины Х называется ее наиболее вероятное значение (для которого плотность вероятности Р(Х) достигает максимума).
Медианой непрерывной случайной величины Х называется такое ее значение, для которого
.
Вертикальная прямая , проходящая через точку с абсциссой, равной , геометрически делит площадь фигуры под кривой распределения на две равные части (рис. 8.7).
Рис. 8.7
Очевидно, что .
Начальный теоретический момент порядка k непрерывной случайной величины Х определяется равенством
.
Центральный теоретический момент порядкаK непрерывной случайной величины Х определяется равенством
.
Если все возможные значения Х принадлежат интервалу , то
, .
Очевидно, что ; ; ; ; . Центральные моменты выражаются через начальные моменты по формулам:
,
,
.
Математическое ожидание М(Х), или первый начальный момент, характеризует среднее значение распределения случайной величины Х; второй центральный момент, или дисперсия , — степень рассеяния распределения Х относительно М(Х).
Третий центральный момент служит для характеристики асимметрии распределения.
Величина называется Коэффициентом асимметрии случайной величины.
А = 0, если распределение симметрично относительно математического ожидания.
Четвертый центральный момент характеризует крутость распределения.
Эксцессом случайной величины называется число
.
Кривые более островершинные, чем кривая для нормального распределения, обладают положительным эксцессом, более плосковершинные — отрицательным эксцессом.
Пример 8.7. Дана функция
При каком значении параметра С эта функция является плотностью распределения некоторой непрерывной случайной величины Х? Найти математическое ожидание и дисперсию случайной величины Х.
Решение. Для того чтобы Р(Х) была плотностью вероятности некоторой случайной величины Х, она должна быть неотрицательна, т. е. , откуда и она должна удовлетворять свойству 4 плотности вероятности.
Следовательно,
Откуда
.
Найдем интеграл , применив метод интегрирования по частям
Таким образом,
И плотность распределения имеет вид
Следовательно,
Дисперсия
Вначале найдем
Теперь
Пример 8.8. Случайная величина Х распределена по «закону прямоугольного треугольника» в интервале (рис. 8.8).
1. Написать выражение плотности распределения.
2. Найти функцию распределения F(Х).
3. Найти вероятность попадания случайной величины Х на участок от до А.
4. Найти характеристики величины Х: М(Х), D(Х), , .
Решение. Так как площадь прямоугольного треугольника есть площадь фигуры, ограниченной кривой распределения и осью абсцисс, то она равна единице: И, следовательно, . Уравнение прямой АВ в отрезках имеет вид , откуда , то есть функция плотности распределения имеет вид
Найдем функцию распределения F(Х):
Если , то
Если , то
Если , то
Таким образом,
Вероятность попадания случайной величины Х на участок от До А определяется по формуле
.
Найдем математическое ожидание:
Следовательно,
,
.
Так как , а , ,
,
То .
Пример 8.9. По данным задачи 8.5 найти математическое ожидание М(Х), дисперсию D(Х), моду М0(Х) и медиану Ме(Х).
Решение. Так как
То .
Дисперсия
Вначале найдем
.
Следовательно,
График плотности вероятности Р(Х) имеет вид (рис. 8.9)
Рис. 8.9
Плотность вероятности р(Х) максимальна при х = 2, это означает, что М0(Х) = 2.
Из условия Найдем медиану Ме(Х): ; откуда
Пример 8.10. Дана функция
Найти коэффициент асимметрии и эксцесс случайной величины Х.
Решение. Плотность распределения случайной величины Х равна
Так как асимметрия , эксцесс , то найдем начальные моменты первого, второго, третьего и четвертого порядков:
Тогда
Так как то Следовательно,
Пример 8.11. Плотность случайной величины Х задана следующим образом:
Найти моду, медиану и математическое ожидание Х.
Решение. Найдем математическое ожидание Х:
.
Так как плотность распределения достигает максимума при Х = 1, то М0(Х) =1. Медиану Ме(Х) найдем из условия . Для этого вначале найдем функцию распределения :
Если , то
Если , то
Если , то
Таким образом,
Уравнение равносильно уравнению , откуда .
Пример 8.12. Случайная величина Х задана плотностью распределения
Найти математическое ожидание функции (не находя предварительно плотности распределения ).
Решение. Воспользовавшись формулой для вычисления математического ожидания функции от случайного аргумента Х
Где А и B — концы интервала, в котором заключены возможные значения Х, получим
Пример 8.13. Случайная величина Х задана плотностью распределения
Найти моду, математическое ожидание и медиану величины Х.
Решение. Так как , то отсюда видно, что при Х = 4 плотность распределения достигает максимума и, следовательно, М0(Х) = 4 (можно было найти максимум методами дифференциального исчисления).
Кривая распределения симметрична относительно прямой Х = 4, поэтому М(Х) = Ме(Х) = 4.
< Предыдущая
Следующая >
Error
Sorry, the requested file could not be found
More information about this error
Jump to. ..
Jump to…Согласие на обработку персональных данных Учебно-тематический планАвторы и разработчики курсаИнформация для студентов и преподавателейВводная лекцияIntroductory lectureЛекция о системе обозначений Lecture on the notation systemВидеолекция (часть 1)Lecture (Part 1)Видеолекция 2. Операции над функциями. Свойства функции.Lecture 2. Operations on functions. The properties of the functionТеоретический материал Практическое занятие. Исследование свойств функций по определениюPractical lesson. Investigation of the properties of functions by definitionЗадачи для самостоятельной работыРешения задачТест 1.1.1(Часть 1). Числовые функцииQuiz 1.1.1 (part 1)Тест 1.1.1(Часть 2). Числовые функцииQuiz 1.1.1 (part 2)Видеолекция 1. Числовая последовательность Lecture 1. Numeric sequenceВидеолекция 2. Предел числовой последовательностиLecture 2. The limit of a numeric sequence.Practical lesson 1. Study of properties of a numerical sequence by conventionПрактическое занятие 1 (часть 2)Теоретический материалЗадачи для самостоятельной работыРешения задачТест 1. 1.2. Числовые последовательностиВидеолекция 1. Предел функции в точкеLecture 1. The limit of a function at a pointВидеолекция (часть 2)Практическое занятие 1. Вычисление пределов, неопределенности.Practical lesson 1. Calculation of limits. UncertaintiesПрактическое занятие 2. Вычисление пределов. Замечательные пределы.Practical lesson 2. Calculation of limits. Remarkable limits.Задачи для самостоятельной работыРешения задачТест 1.1.3. Предел функции в точкеВидеолекция. Непрерывность функции в точкеLecture 1. Сontinuity of a function at a pointПрактическое занятие. Исследование функций на непрерывность. Классификации точек разрываPractical lesson. The study of function continuity and classification of discontinuity pointsЗадачи для самостоятельной работыРешения задачТест 1.1.4. Непрерывность функции в точкеВидеолекция (часть 1)Lecture 1. Differential calculus of functions of a single variableВидеолекция (часть 2)Lecture 2. Differentiation of a function given parametricallyПрактическое занятие 1. Правила дифференцированияПрактическое занятие 2. Логарифмическое дифференцирование. Дифференцирование функции, заданной параметрическиPractical lesson 1. Logarithmic differentiation. Differentiating a function defined parametricallyPractical lesson 2. Rules of differentiationЗадачи для самостоятельной работыРешения задачТаблица производныхТест 1.1.5 Производная функцииВидеолекция 1. Геометрический и физический смысл производнойLecture 1. Geometric and physical meaning of the derivativeВидеолекция 2. Дифференциал функцииLecture 2. Differential of a functionПрактическое занятие 1. Геометрический смысл производнойPractical lesson 1. The geometric meaning of the derivativeПрактическое занятие 2. Производные и дифференциалы высших порядковPractical lesson 2. Higher-order derivatives and differentialsЗадачи для самостоятельной работыРешения задачТест 1.1.6. Геометрический и физический смысл производнойQuiz 1.1.6. Geometric and physical sense of the derivativeВидеолекция 1. Основные теоремы дифференциального исчисления. Lecture 1. Basic theorems of differential calculusВидеолекция 2. Исследование функций на монотонность и выпуклостьLecture 2. The study of the monotonicity of the functionПрактическое занятие 1. Исследование свойств функций с помощью производнойPractical lesson 1. Studying the properties of functions using a derivativeПрактическое занятие 2. Правило ЛопиталяPractical lesson 2. L’Hospital’s ruleЗадачи для самостоятельной работы (Часть 1)Решения задач (Часть 1)Задачи для самостоятельной работы (Часть 2)Решения задач (Часть 2)Тест 1.1.7 (часть 1). Исследование свойств функции с помощью производнойQuiz 1.1.7 (part 1)Тест 1.1.7 (Часть 2). Исследование свойств функции с помощью производнойQuiz 1.1.7 (part 2)Теоретический материал (Часть 1)Задачи для самостоятельной работы (Часть 1)Решения задач (Часть 1)Теоретический материал (Часть 2)Задачи для самостоятельной работы (Часть 2)Решения задач (Часть 2)Тест 1.1.8. Асимптоты графика функцииВидеолекция. Дифференциальное и интегральное исчислениеLecture. Differential and Integral CalculationЗадачи для самостоятельной работыРешения задачТаблица интеграловТест 1.2.1. Неопределенный интегралВидеолекция. Неопределенный интеграл: методы интегрирования.Lecture. Indefinite integral: methods of integration.Практическое занятие. Внесение функции под знак дифференциалаPractical lesson. Adding a function under the sign of the differentialЗадачи для самостоятельной работыРешения задачТест 1.2.2. Методы интегрированияВидеолекция 1. Интегрирование дробно-рациональных функций (часть1)Lecture 1. Integration of fractional-rational functions (part 1)Видеолекция 2. Интегрирование дробно-рациональных функций (часть 2)Lecture 2. Integration of fractionally rational functions (part 2)Практическое занятие 1. Интегрирование иррациональных выражений (часть 1)Practical lesson 1. Integration of irrational expressions (part 1)Практическое занятие 2. Интегрирование тригонометрических функцийPractical lesson 2. Integration of trigonometric functionsЗадачи для самостоятельного решенияРешения задачТест 1. 2.3. Интегрирование рациональных дробей, тригонометрических и иррациональных функцийВидеолекция. Определенный интеграл: интеграл РиманаLecture. Definite integral: Riemann integral. Практическое занятие 1. Вычисление определенного интегралаPractical lesson 1. Calculating a certain integralЗадачи для самостоятельной работыРешения задачТест 1.2.4. Определенный интегралВидеолекция LectureЗадачи для самостоятельного решенияРешения задачТест 1.2.5 Приложения определенного интегралаВидеолекция. Несобственный интегралыLecture. Improper integralЗадачи для самостоятельного решенияРешения задачТест 1.2.6. Несобственные интегралыВидеолекция 1. Функции нескольких переменныхLecture 1. Functions of Multiple VariablesВидеолекция 2. Частные производныеLecture 2. Partial derivativesПрактическое занятие. Функция двух переменныхPractical lesson. Function of several variablesЗадачи для самостоятельной работыРешения задачТест 1.3.1. Функции нескольких переменных (основные понятия)Quiz 1.3.1Видеолекция Дифференцируемость функции двух переменныхLecture. Differentiable functions of two variablesПрактическое занятие 1. Производные и дифференциалы высших порядковПрактическое занятие 2. Понятие дифференциала первого и второго порядкаPractical lesson 2. The concept of the first- and second-order differentialЗадачи для самостоятельной работыРешения задач Тест 1.3.2. Дифференцирование функции нескольких переменныхQuiz 1.3.2Видеолекция 1. Дифференцирование сложной функции, заданной неявноLecture 1. Differentiation of a complex function and a function given implicitlyВидеолекция 2. Производная по направлению. ГрадиентLecture 2. The directional derivative and the gradientПрактическое занятие 1. Производная по направлению, градиентPractical lesson 1. The directional derivative, the gradientПрактическое занятие 2. Исследование свойств функций по определениюPractical lesson 2. Investigating function properties by defenition Практическое занятие 3. Дифференцирование сложной функции и дифференцирование функции, заданной неявноPractical lesson 3. Differentiation of a composite function and differentiation of implicitly defined functionЗадачи для самостоятельного решенияРешения задачТест 1. 3.3. Частные производныеQuiz 1.3.3Видеолекция 1. Экстремум функции двух переменныхВидеолекция 2. Экстремумы функции в замкнутой областиЗадачи для самостоятельной работы (Часть 1)Решения задач (Часть 1)Задачи для самостоятельной работы (Часть 2)Решения задач (Часть 2)Тест 1.3.4. Экстремум функции двух переменныхQuiz 1.3.4Видеолекция 1. Двойной интеграл Lecture 1. Double integral Видеолекция 2. Вычисление двойного интегралаLecture 2. Calculation of the double integralПрактическое занятие 1. Вычисление двойного интегралаPractical lesson 1. Calculating a certain integralПрактическое занятие 2. Вычисление двойного интегралаPractical lesson 2. Calculating a certain integralЗадачи для самостоятельного решения (Часть 1)Решения задач (Часть 1)Задачи для самостоятельного решения (Часть 2)Решения задач (Часть 2)Тест 1.3.5. Двойной интегралQuiz 1.3.5Видеолекция. Криволинейные интегралыLecture. Curvilinear integralsПрактическое занятие. Вычисление криволинейные интегралов I и II родаPractical lesson. Calculating curvilinear integrals 1 and 2 kind Задачи для самостоятельного решенияРешения задачТест 1.3.6. Криволинейные интегралыАттестация по модулю 1Итоговое тестирование по курсу (2-1)Видеолекция 1. Система линейных уравнений: основные понятияПрактическое занятие 1. Системы линейных уравненийPractical lesson (part 1). Systems of linear equationsТеоретический материал (лекция 1)Задачи для самостоятельной работы 1Решения задач 1Видеолекция 2. Решение систем линейных уравнений методом ГауссаПрактическое занятие 2. Решение систем линейных уравнений методом гауссаPractical lesson (part 2). The system of linear equationsТеоретический материал (лекция 2)Задачи для самостоятельной работы 2Решения задач 2Видеолекция 3. Исследование систем линейных уравненийLecture 3. A system of linear equationsPractical lesson (part 3). The system of linear equationsПрактическое занятие 3. Исследование систем линейных уравненийТеоретический материал (лекция 3)Задачи для самостоятельной работы 3Решения задач 3Тест 2. 1.1. Системы линейных уравненийСправочник (часть 1)Справочник (часть 2)Справочник (часть 3)Видеолекция 1. Векторное пространствоLecture 1. Vector spaceВидеолекция 2. линейная зависимость векторов. Базис векторного пространстваLecture 2. Linear dependence of vectors and the concept of the basis of the vector systemПрактическое занятие 1. Арифметическое векторное пространствоPractical lesson 1. Arithmetic vector spaceПрактическое занятие 2. Линейная зависимость векторов. Базис векторного пространстваPractical lesson 2. Linear dependence of vectors and the concept of the basis of the vector systemТеоретический материал (лекция 1)Задачи для самостоятельной работы 1Решения задач 1Теоретический материал (лекция 2)Задачи для самостоятельной работы 2Решения задач 2Тест 2.1.2. Арифметическое n-мерное векторное пространствоСправочник (часть 1)Справочник (часть 2)Видеолекция 1. Исследование систем линейных уравненийLecture 1. Study systems of linear equationsВидеолекция 2. Однородная система линейных уравненийLecture 2. Homogeneous system of equationsПрактическое занятие 1. Фундаментальная система решений однородной системы линейных уравненийPractical lesson 1. Fundamental system of solutionsПрактическое занятие 2Practical lesson 2Теоретический материал (лекция 1)Теоретический материал (лекция 2)Задачи для самостоятельной работыРешения задачТест 2.1.3. Исследование систем линейных уравненийСправочникВидеолекция 1. Матрицы и определителиLecture 1. Matrix determinantВидеолекция 2. Операции над матрицамиLecture 2. Operations on matricesВидеолекция 3. Обратная матрицаLecture 3. Inverse matrixПрактическое занятие 1. Операции над матрицамиPractical lesson 1. The operations on matrices Практическое занятие 2. Вычисление определителейТеоретический материал (лекция 1)Задачи для самостоятельной работы 1Решения задач 1Теоретический материал (лекция 2)Задачи для самостоятельной работы 2Решения задач 2Теоретический материал (лекция 3)Тест 2.1.4. МатрицыQuiz 2.1.4. MatricesСправочник (часть 1)Справочник (часть 2)Справочник (часть 3)Видеолекция 1. Прямоугольная декартова система координатLecture 1. Rectangular Cartesian coordinate systemТеоретический материалПрактическое занятие. Решение задач в координатахPractical lesson. Solution of problems in coordinatesЗадачи для самостоятельной работыРешения задачТест 2.2.1. Декартова система координатСправочникВидеолекция 1. Скалярное произведение векторовLecture 1. Scalar product of vectorsТеоретический материал (Часть 1)Видеолекция 2. Векторное и смешанное произведения векторовLecture 2. Vector and mixed products of vectorsПрактическое занятие 1. Скалярное произведение векторовPractical lesson 1. Scalar product of vectorsПрактическое занятие 2. Применение произведений векторов при решении задачPractical lesson 2. vector and mixed product of vectors to solve themТеоретический материал (Часть 2)Задачи для самостоятельной работы 1Решения задач 1Тест 2.2.2.(часть 1). Скалярное произведение векторов. Длина вектора. Векторное произведение векторов. Смешанное произведение векторовЗадачи для самостоятельной работы 2Решения задач 2Тест 2. 2.2. (часть2). Скалярное произведение векторов. Длина вектора. Векторное произведение векторов. Смешанное произведение векторовСправочник (Часть 1)Справочник (Часть 2)Видеолекция. Уравнения прямой на плоскости и в пространствеLecture. Equation of a straight line on a plane and in spaceТеоретический материалПрактическое занятие 1. Уравнения прямой на плоскостиPractical lesson 1. Related to the equation of a straight line on a planeЗадачи для самостоятельной работы 1Решение задач 1Практическое занятие 2. Взаимное расположение прямыхPractical lesson 2. The relative position of straight lines.Задачи для самостоятельной работы 2Решение задач 2Тест 2.2.3. Уравнения прямойСправочникВидеолекция. Уравнение плоскости. Взаимное расположение прямой и плоскостиТеоретический материалПрактическое занятие. Уравнение плоскости. Взаимное расположение прямой и плоскости Practical lesson. Equation of a plane Задачи для самостоятельной работы 1Решение задач 1Задачи для самостоятельной работы 2Практическое занятие 2. Взаимное расположение плоскостейPractical lesson 2. Relative position of planesРешение задач 2Тест 2.2.4. Уравнения плоскостиСправочникВидеолекция 1. ЭллипсLecture 1. EllipseТеоретический материал Часть 1Практическое занятие 1. ЭллипсPractical lesson 1. EllipseЗадачи для самостоятельной работы 1Решение задач 1Видеолекция 2. Гипербола и параболаLecture 2. Hyperbola and parabolaТеоретический материал (Часть 2)Практическое занятие 2. Гипербола и параболаЗадачи для самостоятельной работы 2Решение задач 2Тест 2.2.5. Кривые второго порядкаСправочник (Часть 1)Справочник (Часть 2)Аттестация по модулю 2Анкета обратной связиИтоговое тестирование по курсу (1-2)Итоговое тестирование по курсу (2)Видеолекция 1. Основные понятия теории вероятностей Lecture 1. Basic concepts of probability theoryВидеолекция 2. Вероятность случайного событияLecture 2. Probability of a random eventПрактическое занятие 1. Классическая вероятностьPractical lesson 1. Classical probabilityЗадачи для самостоятельной работы (часть 1)Решения задач (часть 1)Практическое занятие 2. Операции над событиями. Practical lesson (part 2). Algebra of events. Properties of probabilitiesЗадачи для самостоятельно работы (часть 2)Решения задач (часть 2)Теоретический материалТест 3.1.1. Классическая вероятностьВидеолекция 1. Условная вероятностьLecture 1. Conditional probabilityПрактическое занятие 1. Условная вероятность. Формула полной вероятности. Формула БайесаPractical lesson 1. Conditional probability. The formula of total probability, Bayes ‘ formulaЗадачи для самостоятельной работы. Условная вероятностьРешения задач. Условная вероятностьВидеолекция 2. Повторные независимые опыты и формула БернуллиLecture 2. Repeated Independent Experiments and the Bernoulli FormulПрактическое занятие 2. Схема БернуллиPractical lesson 2. Bernoulli’s formulaЗадачи для самостоятельной работы. Схема БернуллиРешения задач. Схема БернуллиТеоретический материалТест 3.1.2. Условная вероятностьВидеолекция 1. Дискретные лучайные величиныLecture 1. Discrete random variablesВидеолекция 2. Числовые характеристики дискретных случайных величинПрактическое занятие. Дискретные случайные величиныPractical lesson. Discrete random variablesЗадачи для самостоятельного решенияРешения задачЛабораторная работа. Законы распределения дискретных случайных величинLaboratory work 1. Distribution Laws of Discrete Random VariablesЛабораторная работаРешения задач (лабораторная работа)Теоретический материалТест 3.2.1. Дискретные случайные величиныВидеолекция 1. Непрерывные случайные величиныВидеолекция 2. Частные случаи распределений случайных величинLecture 2. Special cases of distributions of random variablesПрактическое занятие. Непрерывные случайные величиныPractical lesson. Continuous random variableЗадачи для самостоятельного решенияРешения задачЛабораторная работа (видео). Законы распределения непрерывных случайных величинLaboratory work (video). Distribution Laws of Continuous Random VariablesЛабораторная работаРешения задач (лабораторная работа)Тест 3.2.2. Непрерывные случайные величиныТеоретический материалТест 3.3.1. Законы больших чиселВидеолекция 1. Система случайных величин (часть 1)Видеолекция 2. Система случайных величин (часть 2)Lecture 2. Systems of random variables (part 2)Практическое занятие. Система случайных величинЗадачи для самостоятельной работыРешения задачЛабораторная работаРешение задачи (лабораторная работа)Теоретический материалТест 3.4.1. Совместный закон распределенияВидеолекция 1. Характеристическая функция случайной величиныLecture 1. Characteristic function of a random variableВидеолекция 2. Свойства характеристической функции случайной величиныLecture 2. Properties of characteristic functions random variable Практическое занятие 1. Вычисление характеристической функции случайной величиныPractical lesson 1. Calculation of Characteristic Functions Практическое занятие 2. Проверка устойчивости для стандартных распределенийPractical lesson 2. Testing the robustness for standard distributions.Задачи для самостоятельного решения (часть 1)Задачи для самостоятельного решения (часть 2)Решения задач (часть 1)Решения задач (часть 2)Тест 3.4.2. (данное тестирование по теме 1)Видеолекция. Основные понятия математической статистикиLecture. The basic concepts of mathematical statisticsЛабораторная работа (видео). Основные понятия математической статистикиLaboratory work (video). Basic concepts of mathematical statisticsТеоретический материалЛабораторная работа. Основные понятия математической статистикиРешения задач (лабораторная работа)Тест 3.5.1. Основные понятия математической статистикиQuiz 3.5.1.Видеолекция. Статистические оценки параметров генеральной совокупности. Lecture. Statistical estimates of general population parametersЛабораторная работа 1 (видео). Статистические оценки параметров генеральной совокупностиLaboratory work 1 (video). Statistical estimators of the parameters of the populationЛабораторная работа 1. Статистические оценки параметров генеральной совокупностиРешения задач 1Лабораторная работа 2 (видео). Минимальный или оптимальный объем выборочной совокупностиLaboratory work 2(video). Minimum or optimal sample sizeЛабораторная работа 2. Минимальный или оптимальный объем выборочной совокупностиРешения задач 2Теоретический материалТест 3. 5.2. Статистические оценкиQuiz 3.5.2Видеолекция. Зависимость между величинами. Виды зависимостейLecture. Dependence between quantities. Types of dependenciesТеоретический материал 1Лабораторная работа 1 (видео, часть 1). Парный корреляционный анализLaboratory work 1 (video, part 1). Pair correlation analysisЛабораторная работа 1. Парный корреляционный анализЛабораторная работа 1 (видео, часть 2). Множественный корреляционный анализРешение задач 1Лабораторная работа 2 (видео, часть 2). Парный регрессионный анализLaboratory work 2 (video, part 2). Paired Regression AnalysisЛабораторная работа 2. Парный регрессионный анализРешения задач 2Теоретический материал 2Тест 3.5.3. Зависимость между величинамиQuiz 3.5.3Лекция. Статистические гипотезы Теоретический материалЛабораторная работа (видео). Статистический критерий хи-квадратLaboratory work. The Chi-Square StatisticЛабораторная работа 1. Критерий хи-квадратРешения задач (Критерий хи-квадрат)Лабораторная работа 2. Критерий ПирсонаЛабораторная работа (расчетная таблица)Решения задач (Критерий Пирсона)Тест 3. 6.1. Проверка статистических гипотез: основные понятияQuiz 3.6.1Видеолекция. Проверка статистических гипотезLecture. Testing statistical hypothesesЛабораторная работа 1 (видео). Сравнение средних выборочных совокупностей при известных дисперсиях генеральных совокупностейLaboratory work 1. Comparison of Sampled Population Means with Known Population VariancesЛабораторная работа 1. Сравнение средних выборочных совокупностей при известных дисперсиях генеральных совокупностейРешения задач (лабораторная работа 1)Лабораторная работа 2 (часть 1). Сравнение средних независимых выборочных совокупностей при неизвестных дисперсиях генеральных совокупностейLaboratory work 2 (part 1). Comparison of means of independent sample populations with unknown variances of general populationsЛабораторная работа 2 (часть 2). Сравнение средних зависимых выборочных совокупностей при неизвестных дисперсиях генеральных совокупностейLaboratory work 2 (part 2). Comparison of mean dependent sample populations with unknown variances of general populationsЛабораторная работа 2. Проверка статистических гипотез о сравнении средних выборочных совокупностей, если не известны дисперсии генеральных совокупностейРешения задач (лабораторная работа 2)Теоретический материалТест 3.6.2. Проверка гипотезQuiz 3.6.2Аттестация по модулю 3Итоговое тестирование по курсу 1-2-3Итоговое тестирование по курсу для математических специальностейИтоговое тестирование по курсу (3)
4.2: Ожидаемое значение и дисперсия непрерывных случайных величин
Последнее обновление
Сохранить как PDF
Идентификатор страницы
3268
Кристин Кутер
Колледж Святой Марии
Теперь рассмотрим математическое ожидание и дисперсию для непрерывных случайных величин. {\infty}_{-\infty}\! х\cdot f(x)\, dx.\notag$$
Формула для ожидаемого значения непрерывной случайной величины является непрерывным аналогом ожидаемого значения дискретной случайной величины, где вместо суммирования по всем возможным значениям мы интегрируем (вспомните разделы 3.6 и 3.7).
Для дисперсии непрерывной случайной величины определение такое же, и мы по-прежнему можем использовать альтернативную формулу, данную теоремой 3.7.1, только теперь мы интегрируем для вычисления значения: 92\notag$$
Пример \(\PageIndex{1}\)
Рассмотрим снова контекст примера 4.1.1, где мы определили непрерывную случайную величину \(X\) для обозначения времени, в течение которого человек ожидает прибытия лифта. PDF \(X\) был задан как $$f(x) = \left\{\begin{array}{l l} x, & \text{for}\ 0\leq x\leq 1 \\ 2-x, & \text{for}\ 1< x\leq 2 \\ 0, & \text{иначе} \end{array}\right.\notag$$ Применяя определение 4. 2.1, мы вычислить ожидаемое значение \(X\): 92 = \frac{7}{6} — 1 = \frac{1}{6} \\ \Rightarrow\ \text{SD}(X) &= \sqrt{\text{Var}(X)} = \frac{1}{\sqrt{6}} \приблизительно 0,408 \end{align*}
Наверх
Была ли эта статья полезной?
Тип изделия
Раздел или страница
Автор
Кристин Кутер
Показать оглавление
да
Теги
На этой странице нет тегов.
Непрерывная случайная величина — определение, формулы, среднее значение, примеры
Непрерывная случайная величина — это случайная величина, которая может принимать континуум значений. Другими словами, случайная величина называется непрерывной, если она принимает значение, попадающее в определенный интервал.
Непрерывные случайные величины используются для обозначения таких измерений, как рост, вес, время и т. д. Площадь под кривой плотности используется для представления непрерывной случайной величины. В этой статье мы узнаем об определении непрерывной случайной величины, ее среднем значении, дисперсии, типах и связанных примерах.
1.
Что такое непрерывная случайная величина?
2.
Непрерывные формулы случайных величин
3.
Среднее значение и дисперсия непрерывной случайной величины
4.
Типы непрерывных случайных величин
5.
Непрерывная случайная величина против дискретной случайной величины
6.
Часто задаваемые вопросы о непрерывной случайной величине
Что такое непрерывная случайная величина?
Непрерывная случайная величина и дискретная случайная величина — это два типа случайных величин. Случайная величина — это переменная, значение которой зависит от всех возможных исходов эксперимента. Непрерывная случайная величина определяется в диапазоне значений, а дискретная случайная величина определяется точным значением.
Определение непрерывной случайной величины
Непрерывная случайная величина может быть определена как случайная величина, которая может принимать бесконечное число возможных значений. Благодаря этому вероятность того, что непрерывная случайная величина примет точное значение, равна 0. Для описания характеристик непрерывной случайной величины используются кумулятивная функция распределения и функция плотности вероятности.
Пример непрерывной случайной величины
Предположим, что функция плотности вероятности непрерывной случайной величины X задана как 4x 3 , где x ∈ [0, 1]. Необходимо определить вероятность того, что X примет значение от 1/2 до 1. Это можно сделать, интегрируя 4x 3 между 1/2 и 1. Таким образом, требуемая вероятность равна 15/16.
Непрерывные формулы случайных величин
Функция плотности вероятности (PDF) и кумулятивная функция распределения (CDF) используются для описания вероятностей, связанных с непрерывной случайной величиной. Формулы непрерывных случайных величин для этих функций приведены ниже.
PDF непрерывной случайной величины
Функцию плотности вероятности непрерывной случайной величины можно определить как функцию, которая дает вероятность того, что значение случайной величины попадет в диапазон значений. Пусть X — непрерывная случайная величина, тогда формула для PDF f(x) задается следующим образом:
Чтобы PDF непрерывной случайной величины была достоверной, она должна удовлетворять следующим условиям: 9{\ infty} е (х) dx = 1 \). Это означает, что общая площадь под графиком PDF должна быть равна 1,.
f(x) > 0. Отсюда следует, что функция плотности вероятности непрерывной случайной величины не может быть отрицательной.
CDF непрерывной случайной величины
Кумулятивную функцию распределения непрерывной случайной величины можно определить путем интегрирования функции плотности вероятности. Его можно определить как вероятность того, что случайная величина X примет значение, меньшее или равное определенному значению x. Формула для cdf непрерывной случайной величины, оцененной между двумя точками a и b, приведена ниже: 9{b}f(x)dx\)
Среднее значение и дисперсия непрерывной случайной величины
Среднее значение и дисперсию непрерывной случайной величины можно определить с помощью функции плотности вероятности f(x).
Среднее значение непрерывной случайной величины
Среднее значение непрерывной случайной величины можно определить как средневзвешенное значение случайной величины X. Оно также известно как математическое ожидание непрерывной случайной величины. Формула дается следующим образом: 9{2}f(x)dx\)
Типы непрерывных случайных величин
Непрерывная случайная величина обычно используется для моделирования ситуаций, связанных с измерениями. Например, возможные значения температуры в любой день. Поскольку температура может быть любым действительным числом в заданном интервале, для ее описания требуется непрерывная случайная величина. Ниже приведены некоторые важные непрерывные случайные величины, связанные с определенными распределениями вероятностей.
Равномерная случайная величина
Непрерывная случайная величина, которая используется для описания равномерного распределения, называется равномерной случайной величиной. Такое распределение описывает события, которые происходят с одинаковой вероятностью. PDF равномерной случайной величины выглядит следующим образом:
\(f(x) = \left\{\begin{matrix} \frac{1}{ba} & a\leq x\leq b\\ 0 & в противном случае \end{matrix}\right.\)
Нормальная случайная величина
Непрерывная случайная величина, используемая для моделирования нормального распределения, называется нормальной случайной величиной. {2})\), то формула для PDF задается следующим образом: 9{2}}{2}}\)
Экспоненциальная случайная величина
Экспоненциальные распределения — это непрерывные распределения вероятностей, которые моделируют процессы, в которых определенное количество событий происходит непрерывно с постоянной средней скоростью, \(\lambda\geq0\) . Таким образом, непрерывная случайная величина, используемая для описания такого распределения, называется экспоненциальной случайной величиной. PDF-файл имеет следующий вид:
f(x) = λe −λx
Непрерывная случайная величина против дискретной случайной величины
Для моделирования случайного явления используются как дискретные, так и непрерывные случайные величины. Различия между непрерывной случайной величиной и дискретной случайной величиной приведены в таблице ниже:
Непрерывная случайная величина
Дискретная случайная величина
Значение непрерывной случайной величины попадает в диапазон значений. {\infty}xf(x)dx\) 9{2}f(x)dx\)
Примерами непрерывной случайной величины являются равномерная случайная величина, экспоненциальная случайная величина, нормальная случайная величина и стандартная нормальная случайная величина.
Примерами дискретной случайной величины являются биномиальная случайная величина, геометрическая случайная величина, случайная величина Бернулли и случайная величина Пуассона.
Статьи по теме:
Вероятность и статистика
Вероятностные правила
Калькулятор функции плотности вероятности
Важные примечания о непрерывной случайной переменной
Непрерывная случайная величина — это переменная, используемая для моделирования непрерывных данных, и ее значение попадает в интервал значений.
Функция плотности вероятности непрерывной случайной величины задается как f(x) = \(\frac{\mathrm{d} F(x)}{\mathrm{d} x}\) = F'(x). 9{2}f(x)dx\).
Равномерная случайная величина, экспоненциальная случайная величина, нормальная случайная величина и стандартная нормальная случайная величина являются примерами непрерывных случайных величин.
Часто задаваемые вопросы о непрерывной случайной величине
Что такое непрерывная случайная величина в теории вероятностей?
Непрерывная случайная величина может быть определена как переменная, которая может принимать любое значение в заданном интервале. Обычно это такие измерения, как рост, вес, время и т. д. 9{2}f(x)dx\).
Какие существуют типы непрерывной случайной величины?
Существует три наиболее часто используемых непрерывных распределения вероятностей, таким образом, существует три типа непрерывных случайных величин. Они даны следующим образом:
Однородная случайная величина
Стандартная нормальная случайная величина
Экспоненциальная случайная величина.
Как найти CDF непрерывной случайной величины?
Чтобы найти кумулятивную функцию распределения непрерывной случайной величины, проинтегрируйте функцию плотности вероятности между двумя пределами.
Остроугольный, прямоугольный и тупоугольный треугольники
Урок 20. Геометрия 7 класс
На данном уроке мы даём определения остроугольному, прямоугольному и тупоугольному треугольникам. Более подробно останавливаемся на прямоугольном треугольнике. И как всегда решаем задачи, применяя полученные теоретические знания.
Конспект урока «Остроугольный, прямоугольный и тупоугольный треугольники»
Теорема о сумме углов
треугольника:
Сумма
углов треугольника равна 180 градусов.
Из
теоремы следует, что если в треугольнике один из углов является прямым или
тупым, то сумма двух других углов данного треугольника не больше 90 градусов, а следовательно, каждый из них острый.
По
величине углов выделяют следующие виды треугольников.
Определение:
Остроугольный
треугольник — это треугольник, у которого все три
угла острые.
Определение:
Тупоугольный
треугольник — это треугольник, у которого один из
углов тупой.
Определение:
Прямоугольный
треугольник — это треугольник, у которого один из
его углов является прямым.
Нужно
знать, что стороны прямоугольного треугольника имеют специальные названия.
Итак, две стороны, образующие прямой угол,
называются катетами, а сторона, противолежащая прямому углу, называется гипотенузой.
Если взять прямоугольный лист бумаги и разрезать
его, получим:
Получим две модели прямоугольного треугольника.
Пример.
Доказать, что угол с вершиной на окружности,
опирающийся на диаметр, — прямой.
Для начала соединим точку В
с точкой О, которая является центром нашей окружности. Так как отрезки ОА, ОВ и
ОС равны как радиусы окружности, то треугольники АОВ и ВОС являются
равнобедренными. А значит, у них углы при основаниях равны. Обозначим градусные
меры этих углов m и n. Тогда ∠АОВ=2n, так как он является внешним углом треугольника
ВОС, смежным с ∠ВОС. А нам известно, что внешний угол
треугольника равен сумме двух внутренних, не смежных с ним.
А так как сумма углов треугольника равна 180
градусов, то:
Что и требовалось доказать.
Пример.
Доказать, что если в равнобедренном треугольнике
АВС один из углов равен 60 градусов, то он равносторонний.
Если ∠А при
основании равнобедренного треугольника АВС равен 60 градусов, то и второй ∠С при основании равен 60 градусам. Получаем:
Следовательно, треугольник АВС равносторонний.
Пусть ∠В при
вершине равнобедренного треугольника АВС равен 60 градусам. Тогда получим:
А так как углы А и С-
углы при основании равнобедренного треугольника, то они равны между собой и
равны 60 градусам. А следовательно, и в этом случае
треугольник АВС является равносторонним. Что и требовалось доказать.
Пример.
Доказать, что в прямоугольном треугольнике АВС
медиана, проведённая к гипотенузе АВ, равна половине гипотенузы.
Отложив ∠2=∠1, получаем:
Треугольник ADC является равнобедренным. А
следовательно, отрезок DA=DC.
Так как по условию угол АВС — прямой, то:
Известно, что сумма острых углов прямоугольного
треугольника равна 90 градусов, то есть:
Тогда из равенств получаем:
Из этого следует, что ВСD равнобедренный треугольник, у которого стороны DB и DC равны.
Следовательно, СD -
медиана и СD равняется половине гипотенузы АВ. Что и
требовалось доказать.
Предыдущий урок 19
Теорема о сумме углов треугольника
Следующий урок 21
Теорема о соотношениях между сторонами и углами треугольника
Получите полный комплект видеоуроков, тестов и презентаций
Геометрия 7 класс
Чтобы добавить комментарий зарегистрируйтесь или войдите на сайт
Остроугольный треугольник – определение и свойства
4. 5
Средняя оценка: 4.5
Всего получено оценок: 316.
4.5
Средняя оценка: 4.5
Всего получено оценок: 316.
В школьном курсе геометрии изучают разные виды треугольников. В задачах очень часто рассматривают остроугольный треугольник, поэтому стоит особенно пристально изучить свойства этой фигуры.
Материал подготовлен совместно с учителем высшей категории Харитоненко Натальей Владимировной.
Опыт работы учителем математики — более 33 лет.
Определение понятия
Треугольником называют фигуру, состоящую из трех точек, и трех отрезков их соединяющих. В зависимости от углов треугольник может быть:
Прямоугольным, если один из углов равен 90 градусов;
Тупоугольный, если один из углов тупой, т.е. больше 90 градусов;
Остроугольным, если все углы треугольника острые.
Для решения задач с остроугольными треугольниками часто приходится использовать теорему синусов или косинусов.
Еще в Древней Греции математики изучали треугольники. Именно греки разработали основы современной геометрии, куда входит и множество теорем о треугольниках. Например, автор теоремы Пифагора родом из Древней Греции.
Характеристики
В остроугольном треугольнике каждый угол меньше 90 градусов. Но сумма углов в треугольнике всегда равна 180. В любой фигуре вершины обозначают заглавными латинскими буквами.
Одним из элементов треугольника, вместе со сторонами и углами, является внешний угол. Внешний угол это угол, смежный с внутренним углом треугольника.
У любого треугольника 6 внешних углов, по 2 на каждый внутренний. Любой внешний угол остроугольного треугольника всегда будет тупым.
Линии остроугольного треугольника
Остроугольный треугольник обладает рядом свойств.
Медиана геометрической фигуры будет делить сторону, на которую она опущена, пополам. Причем можно провести этот отрезок с любой вершины. Медианы пересекаются в одной точке, и эта точка делит каждую из них в отношении 2:1.
Рис. 1. Медианы в остроугольном треугольнике
Известно, что если провести три высоты в остроугольном треугольнике, то они будут пересекаться в одной точке, которую называют ортоцентром. Эти отрезки опускают под прямым углом к противоположным сторонам. Высоты в остроугольном треугольнике разделяют эту фигуру на прямоугольные треугольники.
Рис. 2. Высоты в остроугольном треугольнике
Биссектрисы в остроугольном треугольнике не только делят углы пополам. Эти отрезки пересекаются в точке, которая является центром вписанной окружности.
Также биссектриса разделяет сторону остроугольного треугольника на две части, которые пропорциональны соответствующим боковым сторонам. Данное утверждение нужно запомнить, чтобы решать некоторые задачи.
Рис. 3. Биссектрисы в остроугольном треугольнике
Свойства
Если суммировать числовые значения любых двух сторон остроугольного треугольника, то обязательно получим цифру, которая будет больше третьего отрезка данной геометрической фигуры.
Средняя линия в остроугольном треугольнике параллельна одной из сторон данной фигуры и равна ее половине.
Что мы узнали?
В остроугольном треугольнике каждый угол меньше 90 градусов. Общая сумма углов здесь также равняется 180 градусов. Нельзя забывать о характерных линиях треугольника. Поскольку с их помощью легко вычислить стороны данной треугольной фигуры или центр определенной окружности. А если в условиях задач по геометрии указаны углы, то можно воспользоваться тригонометрическими функциями.
Тест по теме
Доска почёта
Чтобы попасть сюда — пройдите тест.
Людмила Рогатина
10/10
Алексей Рудых
10/10
Константин Никитич
9/10
Ярик Бондарев
10/10
Глеб Быков
10/10
Оценка статьи
4.5
Средняя оценка: 4.5
Всего получено оценок: 316.
А какая ваша оценка?
4.013 острый угол Стоковые фото, картинки и изображения
угол разные степени. набор векторных иконок, состоящих из углов разных градусовPREMIUM
Угол разных градусов. символ геометрии, математики. набор векторных значков, состоящих из углов разной степени. vector illustrationPREMIUM
Типы треугольников на основе длины стороны, набор математической формы треугольника, линейная иллюстрация, черно-белый цвет — vectorPREMIUM
Типы треугольников: по длинам сторон (равносторонний, равнобедренный, разносторонний) по внутренним углам (прямой, тупой, острый). векторная иллюстрация для образовательных и научных целей. PREMIUM
Типы углов: тупой, прямой, острый, прямой, дополнительный и противоположный. векторный набор изолирован на странице бумажной тетради.ПРЕМИУМ
Типы треугольников на белом фоне векторная иллюстрацияПРЕМИУМ
Типы коллекции векторных иллюстраций треугольников. пример равносторонних 3 равносторонних, равнобедренных 2 равносторонних или одинаковых угловых фигур. изучить набор руководств по геометрии шпаргалки. образовательная информация.PREMIUM
Типы треугольников на белом фоне векторная иллюстрацияPREMIUM
Абстрактный витражный фон, цветные элементы расположены в радужном спектреPREMIUM
Измерительные инструменты. линейки, треугольник и транспортир набор векторной иллюстрацииПРЕМИУМ
Векторные измерительные инструментыPREMIUM
Типы треугольниковPREMIUM
Иллюстрация острого углаPREMIUM
Угол 180, 45,30,90 градусов векторная иллюстрация. символ геометрии, математики. набор векторных значков, состоящих из углов разной степени на белом изолированном фоне. слои сгруппированы для удобства редактирования иллюстрации. для вашего дизайна.PREMIUM
Геометрический ретро-узор с треугольниками. абстрактный мозаичный фон.PREMIUM
Абстрактный витражный фон, цветные элементы расположены в радужном спектреPREMIUM
Угол с разными градусами. значок тупого и острого угла. треугольник с углами 90, 45, 120, 180 и 60 градусов. символ меры и математики. vector.PREMIUM
Светящаяся неоновая линия под острым углом 45 градусов значок изолирован на черном фоне. векторная иллюстрацияPREMIUM
Школьные инструменты, векторный набор линеек. инструментальная линейка для измерения и инструментальная линейка сантиметр и миллиметровая шкала иллюстрацияPREMIUM
Белые бумажные треугольники как энергетический дерзкий абстрактный случайный узор в ярком свете с черными строгими тенями, острыми углами, видом сверху. современный простой абстрактный фон.PREMIUM
Углы разные. набор векторных иконок, состоящих из углов разных градусовPREMIUM
Абстрактный фон витража, цветные элементы расположены в радужном спектреPREMIUM
Абстрактный витражный фон, цветные элементы расположены в радужном спектреPREMIUM
Типы треугольниковPREMIUM
Абстрактный геометрический узор круга для декоративной круглой рамки. вектор арт.PREMIUM
Розетка с гильошированным узором для сертификата или других ценных бумаг.PREMIUM
Абстрактный витражный фон, цветные элементы расположены в радужном спектреPREMIUM
Активный яркий динамический абстрактный узор из бумажных треугольников в сияющем свете с резкими тенями, углы насыщенного фиолетового и розового цвета, вид сверху. Воздушный нежный фон будущего в минималистском стиле.PREMIUM
Иллюстрация в стиле витража с абстрактным ярким фоном, радужной плиткой, овальной картинкой.PREMIUM
Набор углов 45, 90, 180 и 360 градусов. набор символов круга вращения стрел. символы математических знаков геометрии. полная стрелка полного вращения.PREMIUM
Школьные инструменты, векторный набор линеекPREMIUM
Абстрактная форма случайной геометрии. генеративное искусство, геометрический зигзаг, крест-накрест, угловатая, острая иллюстрация. странный, странный красочный элемент дизайнаPREMIUM
Коралловый абстрактный геометрический бесшовный рисунок плитки с остроугольным ромбовидным повторомPREMIUM
Изометрическая линия с острым углом 45 градусов, выделенная на синем и оранжевом фоне. серебряная квадратная пуговица. векторная иллюстрацияPREMIUM
Геометрический узор плитки. абстрактный бесшовный фон с ромбомPREMIUM
Значок тупоугольного треугольника. элементы значка геометрической фигуры для концепции и веб-приложений. значок иллюстрации для дизайна и разработки веб-сайтов, разработки приложений. значок премиум на белом фонеPREMIUM
Красочный фон с имитацией цветного стекла.PREMIUM
Черная линия под острым углом 45 градусов на белом фоне. набор иконок красочные. векторная иллюстрацияPREMIUM
Значок школьной угловой линейки. очертить школьный значок линейки углов для веб-дизайна, изолированных на белом фонеPREMIUM
Иконки линий математики. линейный набор. набор векторных линий качества. PREMIUM
Набор иконок под разными углами. 30, 45, 60, 90, 120, 150 градусов. геометрический символ, математические элементы, изолированные на белом фоне. образовательные школьные учебные материалы по геометрии. векторПРЕМИУМ
Линия острого угла 45 градусов значок изолирован бесшовный узор на синем фоне. vectorPREMIUM
Абстрактная форма случайной геометрии. генеративное искусство, геометрический зигзаг, крест-накрест, угловатая, острая иллюстрация. странный, странный красочный элемент дизайнаPREMIUM
Дорогая белая ручка с синими чернилами на черном фонеPREMIUM
Светящаяся неоновая линия под острым углом 45 градусов значок изолирован на фоне кирпичной стены. векторная иллюстрацияPREMIUM
Набор иконок углов. тупые, прямые, острые, прямые, дополнительные и противоположные углы .ПРЕМИУМ
Абстрактная форма случайной геометрии. генеративное искусство, геометрический зигзаг, крест-накрест, угловатая, острая иллюстрация. странный, странный красочный элемент дизайнаPREMIUM
Квадратная иллюстрация плотникаPREMIUM
Значок школьной угловой линейки. контур школы угол линейка вектор значок цвет плоский изолированный на беломPREMIUM
Цветные треугольники геометрический фон для вашего дизайнаPREMIUM
типы треугольников на основе сторон, математические основные формы силуэт — vectorPREMIUM
Абстрактная форма случайной геометрии. генеративное искусство, геометрический зигзаг, крест-накрест, угловатая, острая иллюстрация. странный, странный красочный элемент дизайна. PREMIUM
Векторный набор зеленых стрелок. PREMIUM
Абстрактная форма случайной геометрии. генеративное искусство, геометрический зигзаг, крест-накрест, угловатая, острая иллюстрация. странный, странный красочный элемент дизайнаPREMIUM
Виды треугольной неоновой вывески. светящийся значок урока геометрии. векторная иллюстрация для дизайна. школьная концепцияPREMIUM
Розетка с гильошированным узором для сертификатов или других ценных бумаг. ПРЕМИУМ
Бесшовный зеленый ромб. геометрическая плитка лаймового цвета. PREMIUM
Математика. линейный набор. набор векторных линий качества, таких как острый угол, плюс, кругPREMIUM
Коллекция математических углов. Набор иконок 22,5, 30, 45, 60, 90, 120, 150, 180, 270 и 360 градусов. набор иконок под разными углами. vector illustration.PREMIUM
Различные острые углы в углах треугольника — значения от 15 до 90 градусовPREMIUM
Иконки математических линий. линейный набор. качество векторной линии set.PREMIUM
Спальня в Арле (1888) Винсент Ван Гог раскраска для взрослыхPREMIUM
Стеклянные светящиеся яркие треугольники.PREMIUM
Типы равнобедренного треугольника — геометрические фигуры для детейPREMIUM
Типы треугольников с картинками ~ БЗУ НАУКА
Виды треугольников с картинками
В нашей повседневной
жизни, мы должны иметь дело с различными видами форм. В этих формах некоторые
формы открыты, а некоторые формы закрыты. Чтобы различать эти формы, мы
сохранили названия этих форм. Следовательно, замкнутая фигура, имеющая три
стороны, которые соединены, называется треугольником. Наряду с тремя сторонами,
у треугольника тоже три угла. По этим сторонам и углам треугольники
подразделяются на разные виды. Здесь мы обсудим все типы треугольников.
с помощью картинок.
На основе
стороны, треугольники делятся на три категории. Эти три категории
треугольники, основанные на сторонах, объясняются ниже;
1.
Неравносторонний треугольник
разносторонний
треугольник — это такой треугольник, у которого длины сторон различны. Это значит
что в разностороннем треугольнике нет равных сторон. Вместе с
различны длины сторон, меры всех углов этого треугольника равны
также разные.
2.
Равнобедренный треугольник
Треугольник
у которого две равные стороны (одинаковая длина) называется равнобедренным треугольником.
Две стороны равнобедренного треугольника, имеющие одинаковую длину, называются
стороны и третья сторона этого треугольника, которая имеет другую длину, известна
в качестве базы. Угол, образованный двумя равными сторонами равнобедренного
треугольник называется углом при вершине, а два других угла называются основанием
углы. Важнейшее качество этого треугольника состоит в том, что углы при основании
эти треугольники равномерны.
3.
Равносторонний треугольник
Это
существенный вид треугольника относительно сторон, потому что этот треугольник имеет три
стороны одинаковой длины. Другими словами, мы можем сказать, что равносторонний треугольник
это треугольник, у которого все три стороны равны. Мы также можем сказать
равносторонний треугольник как правильный треугольник, потому что наряду с тремя равными
стороны, этот треугольник также имеет три равных угла. В целом
измерение всех углов треугольника равно 180°. Поэтому каждый угол
равносторонний треугольник равен 60°.
Соответствующие сообщения:
На основе
углов, мы можем разделить эти треугольники на пять различных категорий. Эти
пять категорий треугольников, основанных на углах, объясняются ниже;
4.
Острый треугольник
Как мы знаем
что у треугольника три угла. Если все углы треугольника имеют
меньше 90°, этот треугольник известен как остроугольный треугольник. Самое важное
Особенность этого треугольника в том, что он может быть равнобедренным, разносторонним или равносторонним.
треугольник.
5.
Прямоугольный треугольник
Треугольник
который имеет хотя бы один прямой угол (угол точного измерения 90°) равен
известен как прямоугольный треугольник. Прямоугольный треугольник может быть равнобедренным
или разносторонний треугольник, но он не может быть равносторонним треугольником.
6.
Тупоугольный треугольник
Треугольник
который имеет хотя бы один угол, размер которого больше 90°, но меньше
больше 180° называется тупоугольным треугольником. Рисуя тупоугольный треугольник, вы
нельзя нарисовать более одного тупого угла. Тупоугольный треугольник тоже может быть
равнобедренный или разносторонний треугольник, но он не может быть равносторонним треугольником.
7.
Равносторонний треугольник
Треугольник
у которого все три угла равны, называется равноугольным
треугольник. Это означает, что все углы треугольника должны быть равны 60°. Все
равноугольные треугольники также являются равносторонними.
8.
Косой треугольник
Треугольник, который
треугольник, не являющийся прямоугольным, называется косоугольным. Это означает, что
косоугольный треугольник может быть остроугольным, тупоугольным,
равноугольный треугольник, но он не может быть прямоугольным треугольником.
Вместе с
понимания различных типов треугольников, нам также необходимо получить
представление об основных свойствах треугольников. Эти свойства
поясняется ниже;
·
Согласно
к свойству суммы углов треугольника, сумме всех углов треугольника
должен быть 180°.
·
Если
складываем две стороны треугольника, их сумма будет больше третьей стороны
треугольника.
·
Сторона, лежащая против наибольшего угла треугольника, называется
большая сторона треугольника.
·
Если
мы знаем две стороны треугольника, мы можем легко найти третью сторону треугольника
треугольник с помощью теоремы Пифагора.
·
Если
мы знаем два угла треугольника, мы можем легко найти третий угол
треугольника по формуле суммы углов треугольника.
Это факт
что если мы хотим научить студентов базовой концепции треугольников, мы
учащимся нужно будет привести примеры треугольников из реальной жизни. Некоторый
важные и интересные примеры треугольников из жизни приведены ниже;
·
Мост
из нас ежедневно видят дорожные знаки. Эти дорожные знаки являются лучшими примерами
равнобедренные треугольники, потому что все углы и стороны этих дорожных знаков
равны.
·
Бермуды
треугольник также известен как еще один реальный пример треугольников. Бермуды
треугольник — это слабо очерченная треугольная область в Атлантическом океане. в
Бермудский треугольник, более 50 кораблей и 20 самолетов — моя стерически
исчезнувший.
·
Пирамиды
также являются лучшими реальными примерами треугольников. Это древние
горы, построенные египтянами. Формы этих пирамид
являются тетраэдрическими, то есть в этих древних
горы.
·
Кому
поддерживают мосты, строятся треугольные формы. Эти треугольные формы
известны как ферменные мосты. Эти поддерживающие формы могут распределять вес
мостов.
·
Парусный спорт
лодки также являются лучшими реальными примерами треугольных форм. Несколько лет
раньше дизайн этих форм был квадратным, но в наши дни почти все
парусные формы имеют треугольную форму.
·
крыши домов треугольные. Эти формы крыш не пропускают дождь
и снег, чтобы остаться в течение более длительного периода.
·
Пока
строя лестницы, конструкторы используют знания о треугольнике. Его
причина в том, что они должны строить эти лестницы в треугольной форме.
Эти лестницы образуют прямоугольные треугольники. Когда нам нужно разместить лестницу
перед стеной мы должны сделать треугольную форму.
·
Кому
сделать некоторые здания привлекательными и интересными, эти здания построены
в треугольной форме. Наиболее ярким примером таких зданий
является Эйфелева башня.
·
Мы
должны использовать треугольные формулы, чтобы найти высоту столба или горы. Для
по этой причине мы используем понятие прямоугольного треугольника.
·
Бутерброды
а пицца – самая любимая еда молодого поколения.
Пусть при пересечении прямых а и b секущей АВ накрест лежащие углы равны. Например, ∠ 4 = ∠ 6. Докажем, что а || b.
Предположим, что прямые а и b не параллельны. Тогда они пересекаются в некоторой точке М и, следовательно, один из углов 4 или 6 будет внешним углом треугольника АВМ. Пусть для определенности ∠ 4 — внешний угол треугольника АВМ, а ∠ 6 — внутренний. Из теоремы о внешнем угле треугольника следует, что ∠ 4 больше ∠ 6, а это противоречит условию, значит, прямые а и 6 не могут пересекаться, поэтому они параллельны.
Следствие 1 . Две различные прямые на плоскости, перпендикулярные одной и той же прямой, параллельны (рис.2).
Замечание. Способ, которым мы только что доказали случай 1 теоремы 1, называется методом доказательства от противного или приведением к нелепости. Первое название этот способ получил потому, что в начале рассуждения делается предположение, противное (противоположное)
тому, что требуется доказать. Приведением к нелепости он называется вследствие того, что, рассуждая на основании сделанного предположения, мы приходим к нелепому выводу (к абсурду). Получение такого вывода заставляет нас отвергнуть сделанное вначале допущение и принять то, которое требовалось доказать.
Задача 1. Построить прямую, проходящую через данную точку М и параллельную данной прямой а, не проходящей через точку М.
Решение. Проводим через точку М прямую р перпендикулярно прямой а (рис. 3).
Затем проводим через точку М прямую b
перпендикулярно прямой р. Прямая b
параллельна прямой а согласно следствию из
теоремы 1.
Из рассмотренной задачи следует важный вывод: через точку, не лежащую на данной прямой, всегда можно провести прямую, параллельную данной .
Основное свойство параллельных прямых состоит в следующем.
Аксиома параллельных прямых. Через данную точку, не лежащую на данной прямой, проходит только одна прямая, параллельная данной.
Рассмотрим некоторые свойства параллельных прямых, которые следуют из этой аксиомы.
1) Если прямая пересекает одну из двух параллельных прямых, то она пересекает и другую (рис.4).
2) Если две различные прямые параллельны третьей прямой, то они параллельны (рис.5).
Справедлива и следующая теорема.
Теорема 2. Если две параллельные прямые пересечены секущей, то:
накрест лежащие углы равны;
соответственные углы равны;
сумма односторонних углов равна 180°.
Следствие 2. Если прямая перпендикулярна к одной из двух параллельных прямых, то она перпендикулярна и к другой (см. рис.2).
Замечание. Теорема 2 называется обратной теореме 1. Заключение теоремы 1 является условием теоремы 2. А условие теоремы 1 является заключением теоремы 2. Не всякая теорема имеет обратную, т. е. если данная теорема верна, то обратная теорема может быть неверна.
Поясним это на примере теоремы о вертикальных углах. Эту теорему можно сформулировать так: если два угла вертикальные, то они равны. Обратная ей теорема была бы такой: если два угла равны, то они вертикальные. А это, конечно, неверно. Два равных угла вовсе не обязаны быть вертикальными.
Пример 1. Две параллельные прямые пересечены третьей. Известно, что разность двух внутренних односторонних углов равна 30°. Найти эти углы.
Решение. Пусть условию отвечает рисунок 6.
Вопрос 1. Какие углы называются смежными? Ответ. Два угла называются смежными, если у них одна сторона общая, а другие стороны этих углов являются дополнительными полупрямыми. На рисунке 31 углы (a 1 b) и (a 2 b) смежные. У них сторона b общая, а стороны a 1 и a 2 являются дополнительными полупрямыми.
Вопрос 2. Докажите, что сумма смежных углов равна 180°. Ответ. Теорема 2.1. Сумма смежных углов равна 180°. Доказательство. Пусть угол (a 1 b) и угол (a 2 b) — данные смежные углы (см. рис.31). Луч b проходит между сторонами a 1 и a 2 развёрнутого угла. Поэтому сумма углов (a 1 b) и (a 2 b) равна развёрнутому углу, т. е. 180°. Что и требовалось доказать.
Вопрос 3. Докажите, что если два угла равны, то смежные с ними углы также равны. Ответ.
Из теоремы 2.1 следует, что если два угла равны, то смежные с ними углы равны. Допустим, углы (a 1 b) и (c 1 d) равны. Нам нужно доказать, что углы (a 2 b) и (c 2 d) тоже равны. Сумма смежных углов равна 180°. Из этого следует, что a 1 b + a 2 b = 180° и c 1 d + c 2 d = 180°. Отсюда, a 2 b = 180° — a 1 b и c 2 d = 180° — c 1 d. Так как углы (a 1 b) и (c 1 d) равны, то мы получаем, что a 2 b = 180° — a 1 b = c 2 d. По свойству транзитивности знака равенства следует, что a 2 b = c 2 d. Что и требовалось доказать.
Вопрос 4. Какой угол называется прямым (острым, тупым)? Ответ. Угол, равный 90°, называется прямым углом. Угол, меньший 90°, называется острым углом. Угол, больший 90° и меньший 180°, называется тупым.
Вопрос 5. Докажите, что угол, смежный с прямым, есть прямой угол. Ответ. Из теоремы о сумме смежных углов следует, что угол, смежный с прямым углом, есть прямой угол: x + 90° = 180°, x= 180° — 90°, x = 90°.
Вопрос 6. Какие углы называются вертикальными? Ответ. Два угла называются вертикальными, если стороны одного угла являются дополнительными полупрямыми сторон другого.
Вопрос 7. Докажите, что вертикальные углы равны. Ответ. Теорема 2.2. Вертикальные углы равны. Доказательство. Пусть (a 1 b 1) и (a 2 b 2)- данные вертикальные углы (рис. 34). Угол (a 1 b 2) является смежным с углом (a 1 b 1) и с углом (a 2 b 2). Отсюда по теореме о сумме смежных углов заключаем, что каждый из углов (a 1 b 1) и (a 2 b 2) дополняет угол (a 1 b 2) до 180°, т.е. углы (a 1 b 1) и (a 2 b 2) равны. Что и требовалось доказать.
Вопрос 8. Докажите, что если при пересечении двух прямых один из углов прямой, то остальные три угла тоже прямые. Ответ. Предположим, что прямые AB и CD пересекают друг друга в точке O. Предположим, что угол AOD равен 90°. Так как сумма смежных углов равна 180°, то получаем, что AOC = 180°-AOD = 180°- 90°=90°. Угол COB вертикален углу AOD, поэтому они равны. То есть угол COB = 90°. Угол COA вертикален углу BOD, поэтому они равны. То есть угол BOD = 90°. Таким образом, все углы равны 90°, то есть они все – прямые. Что и требовалось доказать.
Вопрос 9. Какие прямые называются перпендикулярными? Какой знак используется для обозначения перпендикулярности прямых? Ответ. Две прямые называются перпендикулярными, если они пересекаются под прямым углом. Перпендикулярность прямых обозначается знаком \(\perp\). Запись \(a\perp b\) читается: «Прямая a перпендикулярна прямой b».
Вопрос 10. Докажите, что через любую точку прямой можно провести перпендикулярную ей прямую, и только одну. Ответ. Теорема 2.3. Через каждую прямую можно провести перпендикулярную ей прямую, и только одну. Доказательство. Пусть a — данная прямая и A — данная точка на ней. Обозначим через a 1 одну из полупрямых прямой a с начальной точкой A (рис. 38). Отложим от полупрямой a 1 угол (a 1 b 1), равный 90°. Тогда прямая, содержащая луч b 1 , будет перпендикулярна прямой a.
Допустим, что существует другая прямая, тоже проходящая через точку A и перпендикулярная прямой a. Обозначим через c 1 полупрямую этой прямой, лежащую в одной полуплоскости с лучом b 1 . Углы (a 1 b 1) и (a 1 c 1), равные каждый 90°, отложены в одну полуплоскость от полупрямой a 1 . Но от полупрямой a 1 в данную полуплоскость можно отложить только один угол, равный 90°. Поэтому не быть другой прямой, проходящей через точку A и перпендикулярной прямой a. Теорема доказана.
Вопрос 11. Что такое перпендикуляр к прямой? Ответ. Перпендикуляром к данной прямой называется отрезок прямой, перпендикулярной данной, который имеет одним из своих концов их точку пересечения. Этот конец отрезка называется основанием перпендикуляра.
Вопрос 12. Объясните, в чём состоит доказательство от противного. Ответ. Способ доказательства, который мы применили в теореме 2.3, называется доказательством от противного. Этот способ доказательства состоит в том, что мы cначала делаем предположение, противоположное тому, что утверждается теоремой. Затем путем рассуждений, опираясь на аксиомы и доказанные теоремы, приходим к выводу, противоречащему либо условию теоремы, либо одной из аксиом, либо доказанной ранее теореме. На этом основании заключаем, что наше предположение было неверным, а значит, верно утверждение теоремы.
Вопрос 13. Что называется биссектрисой угла? Ответ. Биссектрисой угла называется луч, который исходит из вершины угла, проходит между его сторонами и делит угол пополам.
Которые лежат в одной плоскости и либо совпадают, либо не пересекаются. В некоторых школьных определениях совпадающие прямые не считаются параллельными, здесь такое определение не рассматривается.
Свойства
Параллельность — бинарное отношение эквивалентности , поэтому разбивает всё множество прямых на классы параллельных между собой прямых.
Через любую точку можно провести ровно одну прямую, параллельную данной. Это отличительное свойство евклидовой геометрии , в других геометриях число 1 заменено другими (в геометрии Лобачевского таких прямых минимум две)
2 параллельные прямые в пространстве лежат в одной плоскости.
При пересечении 2 параллельных прямых третьей, называемой секущей :
Секущая обязательно пересекает обе прямые.
При пересечении образуется 8 углов, некоторые характерные пары которых имеют особые названия и свойства:
Накрест лежащие углы равны.
Соответственные углы равны.
Односторонние углы в сумме составляют 180°.
В геометрии Лобачевского
В геометрии Лобачевского в плоскости через точку C вне данной прямой A B проходит бесконечное множество прямых, не пересекающих A B . Из них параллельными к A B называются только две. Прямая C E называется равнобежной (параллельной) прямой A B в направлении от A к B , если:
точки B и E лежат по одну сторону от прямой A C ;
прямая C E не пересекает прямую A B , но всякий луч, проходящий внутри угла A C E , пересекает луч A B .
Аналогично определяется прямая, равнобежная A B в направлении от B к A .
Все остальные прямые, не пересекающие данную, называются ультрапараллельными или расходящимися .
См. также
Wikimedia Foundation
.
2010
.
Смотреть что такое «Накрест лежащие» в других словарях:
Эта теорема о параллельных прямых. Об угле, опирающемся на диаметр, см. другую теорему. Теорема Фалеса одна из теорем планиметрии. Если на одной из двух прямых отложить последовательно несколько равных отрезков и через их концы провести… … Википедия
Русский орден Св. Анны, учрежден был владетельным герцогом Шлезвиг Гольштинским Карлом Фредериком в 1736 г. в честь супруги своей цесаревны Анны Петровны (дочери Петра Великого) и причислен к русским орденам императором Петром III. Орден Св. Анны …
Для испытания охотничьих ружейных стволов учреждены во всех западно европейских государствах. Наиболее известные из них находятся в Лондоне, Бирмингеме, Люттихе, Зуле и Сент Этьене. По новым, недавно введенным в Англии правилам каждый ствол… … Энциклопедический словарь Ф.А. Брокгауза и И.А. Ефрона
Так называется один из способов количественного определения содержания веществ в растворах; методы К. применимы к количественному определению всех тех веществ, которые дают окрашенные растворы, или могут быть, с помощью какой либо реакции,… … Энциклопедический словарь Ф.А. Брокгауза и И.А. Ефрона
Жалуемый за особые заслуги или отличие знак, установленной формы, носимый на ленте, цепи или иным образом. Имеются указания, что в восточной римской империи еще со времен Константина Великого императоры установляли кавалерские товарищества или… … Энциклопедический словарь Ф.А. Брокгауза и И.А. Ефрона
Жалуемый за особые заслуги или отличие знак, установленной формы, носимый на ленте, цепи или иным образом. Имеются указания, что в вост. римской империи еще со времен Константина Вел., императоры установляли кавалерские товарищества или ордена,… … Энциклопедический словарь Ф.А. Брокгауза и И.А. Ефрона
Второе семейство этого отряда состоит из одного рода и вида моржа (Odobenus rosmarus)*, самого огромного из всех ластоногих. * Моржи имеют в анатомии черты сходства с ушастыми тюленями и также происходят от примитивного медведеобразного… … Жизнь животных
— (др. греч. παραλληλόγραμμον от παράλληλος параллельный и γραμμή линия) это четырёхуго … Википедия
Пересечения прямых (анимация) Аксиома параллельности Евклида, или пятый постулат одна из аксиом, лежащ … Википедия
Пересечения прямых (анимация) Аксиома параллельности Евклида, или пятый постулат одна из аксиом, лежащих в основании классической планиметрии. Впервые приведена в «Началах» Евклида : И если прямая, падающая на две прямые, образует внутренние и … Википедия
Свойства параллельных прямых – основные определения
4.3
Средняя оценка: 4.3
Всего получено оценок: 300.
4.3
Средняя оценка: 4.3
Всего получено оценок: 300.
Свойства параллельных прямых крайне часто встречаются при решении задач и доказательств теорем. Произвольные прямые – редкость, но есть такие фигуры, как квадрат или параллелограмм, где параллельные прямые могут стать основой задачи, а без знания свойств параллельных прямых решить такие задачи невозможно.
Материал подготовлен совместно с учителем высшей категории Харитоненко Натальей Владимировной.
Опыт работы учителем математики — более 33 лет.
Что такое свойства параллельных прямых
Для начала выделим определения, которые необходимо знать для изучения свойств параллельных прямых.
Параллельные прямые это прямые, которые не имеют общих точек, или прямые, которые не пересекаются и лежат в одной плоскости.
Пересечение означает, что у двух объектов есть общая точка или набор точек. Поэтому когда в геометрии говорят, что прямые имеют общую точку, имеется в виду, что они пересекаются.
При пересечении двух прямых секущей, образуются накрест лежащие, соответственные и односторонние углы.
Существует аксиома параллельных прямых, которая крайне важна при доказательстве некоторых свойств и является основным свойством параллельных прямых. Аксиома гласит, что через точку на плоскости можно провести только одну прямую, параллельную данной.
Две группы свойств параллельных прямых
Свойств у параллельных прямых всего 5, но они делятся на две большие группы: следствия из аксиомы параллельных прямых и следствия из признаков параллельности прямых. Начнем с первой группы.
Следствия из аксиомы параллельных прямых
Следствие 1
Если одна из двух параллельных прямых, параллельна третьей, то и другая прямая ей параллельна.
Кажется, что это логично и не требует доказательства. Но в геометрии количество утверждений, не требующих обоснования, крайне мало, и каждое из них носит название – аксиома.
Аксиомы были выведены еще на заре геометрии и с тех пор мало что изменилось. Большая часть современных теорем выведена на основании аксиом Древней Греции. Эти утверждения единственные, что в математике не требует доказательства.
Проведем две параллельные прямые а и b. Прямая с параллельна прямой а. Предположим, что при этом с не параллельна прямой b. Тогда у нее должна быть какая-то точка пересечения К. То есть через точку К проходит две прямые с и b. При этом каждая из этих прямых должна быть параллельна прямой а.
То есть, через одну точку на плоскости проведены две прямые, параллельные данной. Это невозможно, потому что противоречит аксиоме параллельных прямых. Значит изначальное предположение было неверным и прямые с и b параллельны.
Рис. 1. Иллюстрация следствия.
Следствие 2
Следствие 2 очень важно, так как говорит о секущей двух параллельных прямых. Свойство гласит: если прямая пересекает одну из параллельных прямых, то она пересечет и вторую.
Доказательство также ведется методом от противного. Проведем две прямые: а и b. Представим, что прямая с пересекает прямую а, но не пересекает прямую b. Тогда прямые c и b параллельны. При этом с пересекает а, то есть у этих прямых есть общая точка К.
Тогда через точку К проходит прямая а и прямая с, но каждая из них параллельна b. Значит, через одну точку проходит две прямых параллельных прямой b, а это невозможно по аксиоме параллельных прямых. Значит изначальное предположение было неверным и прямая с пересекает каждую из прямых а и b, что и требовалось доказать.
Рис. 2. Рисунок к доказательству.
Следствия из признаков параллельности
Эту группу запомнить проще всего. Свойств параллельности прямых всего 3 и каждому из них соответствует свое следствие.
Прямые параллельны, если накрест лежащие углы при секущей равны. Следствие вполне логично: Накрест лежащие углы при двух параллельных прямых и секущей равны.
Прямые параллельны, если соответственные углы равны. Следствие: соответственные углы при параллельных прямых и секущей равны.
Прямые параллельны, если сумма односторонних углов равна 180. Следствие: сумма односторонних углов при параллельных прямых и секущей равны 180
Рис. 3. Иллюстрация признаков.
Что мы узнали?
Мы дали понятие параллельным прямым, выделили две большие группы свойств параллельных прямых и доказали два свойства. Разобрались с использованием аксиомы параллельных прямых при доказательстве теорем в геометрии.
Тест по теме
Доска почёта
Чтобы попасть сюда — пройдите тест.
Девлет Болтаев
5/5
Иван Маковецкий
5/5
Иришка Чипрягова
5/5
Елена Иванова
5/5
Константин Никитич
5/5
Манижа Гуломджонова
5/5
Екатерина Лобычева
4/5
Оценка статьи
4. 3
Средняя оценка: 4.3
Всего получено оценок: 300.
А какая ваша оценка?
Параллельные прямые — определение, свойства
Две или более прямых, лежащих в одной плоскости и никогда не пересекающихся друг с другом, называются параллельными прямыми . Они равноудалены друг от друга и имеют одинаковый наклон. Давайте узнаем больше о параллельных прямых, свойствах параллельных прямых и углах, которые образуются при пересечении параллельных прямых секущей.
1.
Что такое параллельные линии?
2.
Параллельные линии и поперечные
3.
Свойства параллельных линий
4.
Часто задаваемые вопросы о параллельных линиях
Что такое параллельные линии?
Параллельные линии — это прямые линии, которые никогда не пересекаются друг с другом, как бы мы их ни растягивали. Обратите внимание на следующий рисунок, на котором показаны параллельные линии. Линия «а» параллельна линии «b», а линия «p» параллельна линии «q».
Параллельные линии и поперечные
При пересечении любых двух параллельных прямых другой прямой, называемой секущей, образуется множество пар углов. В то время как некоторые углы конгруэнтны (равны), другие являются дополнительными. Обратите внимание на следующий рисунок, чтобы увидеть параллельных прямых, пересекаемых поперечной . Параллельные прямые обозначены как L1 и L2, которые пересекаются секущей. Восемь отдельных углов были образованы двумя параллельными прямыми и секущей. Каждый угол был помечен с помощью алфавита.
Ниже приведены пары углов, образованных двумя параллельными прямыми L1 и L2.
Соответствующие углы: Следует отметить, что пара соответствующих углов равна по размеру. На данном рисунке четыре пары соответствующих углов, то есть ∠a = ∠e, ∠b = ∠f, ∠c = ∠g и ∠d = ∠h
Альтернативные внутренние углы: Альтернативные внутренние углы образуются внутри двух параллельных линий, которые пересекаются секущей. Они равны в меру. На этом рисунке ∠c = ∠e, ∠d = ∠f
Альтернативные внешние углы: Альтернативные внешние углы образованы по обе стороны от поперечной, и они равны по размеру. На этом рисунке ∠a = ∠g, ∠b = ∠h
Последовательные внутренние углы: Последовательные внутренние углы или ковнутренние углы образуются на внутренней стороне поперечной и являются дополнительными. Здесь ∠c + ∠f = 180°, а ∠d + ∠e = 180°
Вертикально противоположные углы: Вертикально противоположные углы образуются, когда две прямые пересекают друг друга, и они равны по размеру. Здесь ∠a = ∠c, ∠b = ∠d, ∠e = ∠g, ∠f = ∠h
Свойства параллельных линий
Параллельные линии можно легко идентифицировать по основным свойствам, приведенным ниже.
Параллельные линии — это прямые линии, которые всегда находятся на одинаковом расстоянии друг от друга.
Параллельные линии никогда не пересекаются, независимо от того, насколько они вытянуты в любом направлении.
Как узнать, параллельны ли линии?
Помимо характеристик, приведенных выше, когда любые две параллельные прямые пересекаются секущей, их можно идентифицировать по следующим свойствам.
Любые две прямые называются параллельными, если соответствующие углы, образованные таким образом, равны.
Любые две прямые называются параллельными, если образованные таким образом альтернативные внутренние углы равны.
Любые две прямые называются параллельными, если образованные таким образом альтернативные внешние углы равны.
Любые две прямые называются параллельными, если смежные внутренние углы по одну сторону от секущей являются дополнительными.
Уравнение параллельных линий
Уравнение прямой линии, как правило, записывается в форме наклона и точки пересечения, представленной уравнением y = mx + b, где «m» — это наклон, а «b» — это точка пересечения с осью y. Значение «m» определяет наклон или градиент и сообщает нам, насколько крута линия.
Следует отметить, что наклон любых двух параллельных линий всегда одинаков. Например, если наклон линии с уравнением y = 4x + 3 равен 4. Следовательно, любая линия, параллельная y = 4x + 3, будет иметь одинаковый наклон, то есть 4. Параллельные прямые имеют разные y- пересекаются и не имеют общих точек.
Параллельные линии Символ
Параллельные линии — это линии, которые никогда не пересекаются, независимо от того, как долго мы их продлеваем. Для обозначения параллельных прямых используется символ || . Например, AB II PQ указывает, что линия AB параллельна линии PQ. Символ, обозначающий непараллельные прямые, равен ∦ .
Примеры параллельных линий в реальной жизни
Ниже приведены несколько примеров параллельных линий в реальной жизни:
Железнодорожные пути
Зебра-переход
Лестница и перила
☛ Ссылки по теме
Точки и линии
Пересекающиеся линии
Линейный сегмент
Свойства параллельных линий
Параллельные и перпендикулярные линии
Уравнение прямой
Часто задаваемые вопросы о параллельных линиях
Что такое параллельные линии в математике?
Параллельные линии — это линии, которые всегда находятся на одном и том же расстоянии друг от друга и никогда не пересекаются. Для обозначения параллельных прямых используется символ || . Например, AB || CD означает, что линия AB параллельна линии CD.
Что такое параллельные линии и перпендикулярные линии?
Параллельные линии — это линии, которые равноудалены друг от друга и никогда не пересекаются, независимо от того, насколько они могут быть продлены в любом направлении. Например, противоположные стороны прямоугольника представляют собой параллельные линии. С другой стороны, если любые две прямые пересекаются друг с другом в точке 90°, они называются перпендикулярными линиями. Например, смежные стороны прямоугольника являются перпендикулярными линиями, поскольку они пересекают друг друга под углом 90°.
Какие бывают типы углов в параллельных линиях?
Когда любые две параллельные прямые пересекаются секущей, они образуют множество пар углов, таких как соответствующие углы, чередующиеся внутренние углы, чередующиеся внешние углы и последовательные внутренние углы.
Что происходит, когда параллельные прямые пересекаются секущей?
При пересечении любых двух параллельных прямых секущей образуются следующие углы.
Соответственные углы, равные по размеру.
Чередующиеся внутренние углы, равные по размеру.
Альтернативные внешние углы, равные по размеру.
Последовательные внутренние углы, которые являются дополнительными.
Как выглядят параллельные линии?
Параллельные линии выглядят как железнодорожные пути, которые никогда не пересекаются и всегда равноудалены. Противоположные стороны прямоугольника также представляют собой параллельные линии, находящиеся на равном расстоянии друг от друга.
Каков наклон параллельных линий?
Если две линии параллельны, они имеют одинаковый наклон. Например, если уравнение прямой линии y = 1/2x + 17, ее наклон равен 1/2. Теперь, если линия имеет одинаковый наклон 1/2 в той же плоскости, она будет параллельна данной линии.
Каковы реальные примеры параллельных линий?
Реальные примеры параллельных линий включают железнодорожные пути, края тротуаров, перила лестницы, бесконечные рельсовые пути, противоположные стороны линейки, противоположные края ручки, ластика и т. д.
Каково правило параллельных линий?
Правило для параллельных линий состоит в том, что линии не должны пересекаться друг с другом. Другими словами, если две прямые в одной плоскости находятся на одинаковом расстоянии друг от друга и никогда не пересекаются, то такие прямые называются параллельными.
Что такое символ параллельных линий?
Для обозначения параллельных линий используется символ ||. Любые две параллельные прямые AB и CD изображаются как AB||CD.
Параллельные прямые пересекаются в бесконечности?
Нет, само определение предполагает, что параллельные линии никогда не пересекаются. Следовательно, параллельные прямые не пересекались бы даже на бесконечности.
Параллельные прямые имеют одно и то же уравнение?
Нет, параллельные прямые не имеют одинакового уравнения, но имеют одинаковый наклон. Например, если уравнение линии представлено как y = 4x + 2, это означает, что наклон этой линии равен 4. Таким образом, другая прямая линия в той же плоскости с таким же наклоном 4 будет параллельна заданная линия.
Параллельные прямые равны по длине?
Нет, параллельные линии могут быть разной длины, но они должны находиться на одинаковом расстоянии друг от друга.
Есть ли в треугольнике параллельные линии?
Нет, в треугольнике нет параллельных прямых. Так как треугольник всегда имеет 3 пересекающиеся стороны; а мы знаем, что параллельные прямые никогда не пересекаются друг с другом, следовательно, в треугольнике не может быть параллельных прямых.
Сколько параллельных линий имеет шестиугольник?
Шестиугольник — это многоугольник с шестью сторонами. В правильном шестиугольнике три пары параллельных прямых.
Какие углы образуются при пересечении параллельных прямых секущей?
При пересечении двух параллельных прямых секущей образуются следующие углы:
Соответствующие углы
Альтернативные внутренние углы
Альтернативные внешние углы
Последовательные внутренние углы
Вертикально противоположные углы
Параллельные прямые и пары углов
Параллельные прямые и пары углов
Показать рекламу
Скрыть рекламу О рекламе
Параллельные линии
Линии параллельны, если они всегда находятся на одном и том же расстоянии друг от друга (называемом «эквидистантными») и никогда не пересекутся. Просто помните:
Всегда на одинаковом расстоянии друг от друга и никогда не касаясь
.
Красная линия параллельна синей линии в каждом из следующих примеров:
Пример 1
Пример 2
Параллельные линии также указывают в одном направлении.
У параллельных линий так много общего. Жаль, что они никогда не встретятся!
Попробуйте сами:
Пары углов
Когда параллельные прямые пересекаются другой прямой (называемой секущей), вы видите, что многие углы совпадают, как в этом примере:
Эти углы можно составить из пар углов , которые имеют специальные названия.
Нажмите на каждое имя, чтобы выделить его:
Теперь поиграйте с ним здесь. Попробуйте перетаскивать точки и выбирать разные типы углов. Вы также можете включить или выключить «Параллельность»:
Проверка параллельных линий
Некоторые из этих специальных пар углов можно использовать для проверки параллельности прямых:
Если Любая пара Из . ..
Пример:
Соответствующие углы равны
а = е
или
Альтернативные внутренние углы равны
с = ж
или
Альтернативные внешние углы равны
б = г
или
Последовательные внутренние углы в сумме составляют 180°
d + f = 180°
… тогда строки Параллельные
Примеры
Эти прямые параллельны, потому что пара соответствующих углов равна.
Уравнение плоскости, проходящей через данную точку и перпендикулярной данной прямой онлайн
С помощю этого онлайн калькулятора можно построить уравнение плоскости, проходящей через данную точку и перпендикуляной данной прямой. Дается подробное решение с пояснениями. Для построения уравнения плоскости задайте вид уравнения прямой (канонический или параметрический) введите координаты точки и коэффициенты уравнения прямой в ячейки и нажимайте на кнопку «Решить».
Очистить все ячейки?
Инструкция ввода данных. Числа вводятся в виде целых чисел (примеры: 487, 5, -7623 и т.д.), десятичных чисел (напр. 67., 102.54 и т.д.) или дробей. Дробь нужно набирать в виде a/b, где a и b (b>0) целые или десятичные числа. Примеры 45/5, 6.6/76.4, -7/6.7 и т.д.
Уравнение плоскости, проходящей через данную точку и перпендикулярной данной прямой − теория, примеры и решения
Задана точка M0(x0, y0, z0) и прямая L:
Построить уравнение плоскости α, проходящей через точку M0 и перпендинулярной прямой L.
Решение. Уравнение плоскости, проходящей через точку M0 и имеющий нормальный вектор n={A, B, C} имеет следующий вид:
A(x−x0)+B(y−y0)+C(z−z0)=0.
(2)
Направляющий вектор прямой L имеет вид q={m, p, l}. Поскольку прямая L и плоскость α перпендикулярны друг другу, следовательно нормальный вектор плоскостти и направляющий вектор прямой должны быть коллинеарны (Рис.1). Тогда вместо координат нормального вектора плоскости нужно подставить координаты направляющего вектора прямой L. Получим следующее уравнение плоскости:
m(x−x0)+p(y−y0)+l(z−z0)=0.
(3)
Упростим уравнение (3):
mx+py+lz+D=0,
(4)
где D=−mx0−px0−lx0.
Таким образом уравнение (4) определяет плоскость, проходящей через точку M0(x0, y0, z0) и перпендикулярной прямой (1).
Ответ. Уравнение плоскости прпоходящей через точку M0(x0, y0, z0) и перпендикулярной прямой (1) имеет вид (4).
Пример 1. Найти уравнение плоскости α, проходящую через точку M0(3, −1, 2) и перпендикулярной прямой L:
Решение. Уравнение плоскости α, проходящей через точку M0(x0, y0, z0) и имеющий нормальный вектор n={A, B, C} представляется формулой (2).
Направляющий вектор прямой L имеет следующий вид: :
q={m, p, l}={2, 5, 4}
Для того, чтобы прямая L была перпендикулярна плоскости α, нормальный вектор плоскости α должен быть коллинеарным направляющему вектору прямой L, т. е. уравнение плоскости (2) примет следующий вид:
m(x−x0)+p(y−y0)+l(z−z0)=0.
(8)
Подставляя координаты точки M0 и направляющего вектора q в (8), получим:
Упростим уравнение (9):
2x+5y+4z−9=0.
(10)
Ответ: Уравнение плоскости, проходящей через точку M0(3, −1, 2) и перпендикулярной прямой (7) имеет вид (10).
Пример 2. Найти уравнение плоскости α, проходящую через точку M0(4, 3, −6) и перпендикулярной прямой L, заданной параметрическим уравнением:
Решение. Приведем параметрическое уравнение (11) к каноническому виду:
Уравнение плоскости α, проходящей через точку M0(x0, y0, z0) и имеющий нормальный вектор n={A, B, C} представляется формулой:
A(x−x0)+B(y−y0)+C(z−z0)=0.
(12)
Направляющий вектор прямой L имеет следующий вид:
q={m, p, l}={−5, 3, 11}
Для того, чтобы прямая L была перпендикулярна плоскости α, нормальный вектор плоскости α должен быть коллинеарным направляющему вектору прямой L, т.е. уравнение плоскости (12) примет следующий вид:
m(x−x0)+p(y−y0)+l(z−z0)=0.
(13)
Подставляя координаты точки M0 и направляющего вектора q в (13), получим:
Упростим уравнение (13):
−5x+3y+11z+77=0.
(14)
Ответ. Уравнение плоскости, проходящей через точку M0(4, 3, −6) и перпендикулярной прямой (11) имеет вид (14).
Как найти уравнение перпендикулярной прямой
Все ресурсы по алгебре 1
10 Диагностические тесты
557 практических тестов
Вопрос дня
Карточки
Learn by Concept
← Предыдущая 1 2 3 Следующая →
Алгебра 1 Помощь »
Функции и линии »
Уравнения прямых »
Перпендикулярные линии »
Как найти уравнение перпендикулярной прямой
Найдите уравнение перпендикулярной прямой в точке .
Возможные ответы:
Правильный ответ:
Объяснение:
Наклон должен быть обратным отрицательным, а линия должна проходить через точку (3,2). Таким образом, наклон становится , и он подключается к , чтобы найти -intercept.
Сообщить об ошибке
Какая линия перпендикулярна ?
Возможные ответы:
Правильный ответ:
Объяснение:
Перпендикулярные линии имеют отрицательные обратные наклоны друг к другу. Поскольку исходное уравнение имеет наклон , перпендикулярная линия должна иметь наклон . Единственным другим уравнением с наклоном является .
Сообщить об ошибке
Какое уравнение дает прямую, перпендикулярную проходам через ?
Возможные ответы:
Правильный ответ:
Объяснение:
Сначала преобразуйте данное уравнение в форму пересечения наклона.
В этом формате мы можем сказать, что наклон равен . Наклон перпендикулярной линии будет обратной отрицательной величиной, что составляет .
Затем подставьте наклон в форму пересечения наклона, чтобы получить точку пересечения, используя точку, указанную в вопросе.
Перпендикулярное уравнение становится . Это уравнение можно переписать в формате вариантов ответов.
или
Сообщить об ошибке
Какая из этих прямых перпендикулярна?
Возможные ответы:
Ни один из других ответов
Правильный ответ:
Объяснение:
Перпендикулярные линии имеют наклоны, которые являются отрицательными обратными величинами. Наклон данной линии равен 9, поэтому линия, перпендикулярная ей, должна иметь наклон, эквивалентный ее отрицательной обратной величине, которая равна .
Сообщить об ошибке
Какая из этих прямых перпендикулярна ?
Возможные ответы:
Правильный ответ:
Объяснение:
Перпендикулярные линии имеют наклоны, которые являются отрицательными обратными величинами. Данная линия имеет наклон . Отрицательная обратная величина равна , поэтому перпендикулярная линия должна иметь наклон . Единственная линия с наклоном .
Сообщить об ошибке
Найдите уравнение прямой, перпендикулярной и содержащей точку (5,3).
Возможные ответы:
Правильный ответ:
Объяснение:
Чтобы найти уравнение прямой, нам нужно знать наклон и точку, проходящую через прямую. Зная это, мы можем использовать уравнение, где m — наклон линии, а — точка на линии. Для перпендикулярных линий наклоны являются отрицательными обратными величинами. Наклон равен 5, поэтому наклон перпендикулярной линии будет иметь наклон . Мы знаем, что перпендикулярная линия должна содержать точку (5,3), поэтому у нас есть вся необходимая информация. Теперь мы можем использовать уравнение
Сообщить об ошибке
Линия проходит через следующие точки:
: (2,3)
: (4,7)
Найдите уравнение прямой , которая перпендикулярна прямой и проходит через точку .
Возможные ответы:
Правильный ответ:
Объяснение:
Уравнение прямой записывается в следующем формате:
1) Итак, первый шаг – найти наклон .
равно изменению , деленному на изменение .
Итак,
2) Перпендикулярный наклон линии с наклоном 2 противоположен обратному значению 2, то есть .
3) Следующим шагом будет поиск . Нам не нужно находить уравнение исходной линии; все, что нам нужно от исходной линии, это наклон. Итак, все, что нам нужно, это перпендикулярная линия. Мы можем найти значения для и из одной точки перпендикулярной линии, подставить их и найти .
Наша точка равна (4,7)
Итак,
Затем мы просто вводим наше значение для , и мы имеем как функцию .
Сообщить об ошибке
Возможные ответы:
Правильный ответ:
Объяснение:
Сообщить об ошибке
Напишите уравнение прямой, перпендикулярной с точкой пересечения .
Возможные ответы:
Правильный ответ:
Объяснение:
Эта задача сначала опирается на знание формы пересечения наклона линии, , где m — наклон, а b — пересечение оси y.
Чтобы линия была перпендикулярна другой линии, ее наклон должен быть обратной отрицательной величиной. В этом случае мы ищем прямую, перпендикулярную . Эта линия имеет наклон 2, она же . Это означает, что отрицательный обратный наклон будет . Нам говорят, что точка пересечения по оси Y этой новой строки равна 4,9.0005
Теперь мы можем ввести эти две новые части информации, чтобы получить уравнение
.
Сообщить об ошибке
Напишите уравнение прямой, проходящей через точку , перпендикулярную прямой.
Возможные ответы:
Правильный ответ:
Объяснение:
Чтобы решить задачу такого типа, мы должны быть знакомы с формой линии, пересекающей наклон, где m — наклон, а b — точка пересечения с осью y. Линия, к которой наша линия перпендикулярна, имеет уравнение пересечения наклона, что означает, что наклон равен .
Наклон перпендикулярной линии был бы обратной отрицательной величиной, поэтому наш наклон равен .
Мы еще не знаем точку пересечения по оси y нашей линии, поэтому мы можем написать уравнение только так:
.
Мы знаем, что точка находится на этой линии, поэтому, чтобы найти b, мы можем подставить -2 вместо x и 3 вместо y:
Сначала мы можем умножить, чтобы получить .
Получается наше уравнение:
Либо вычитая 3 с обеих сторон, либо просто критически взглянув на это, мы можем увидеть, что b = 0,
Наш оригинал становится , или просто .
Сообщить об ошибке
← Назад 1 2 3 Далее →
Уведомление об авторских правах
Все ресурсы по алгебре 1
10 Диагностические тесты
557 практических тестов
Вопрос дня
Карточки
Learn by Concept
4.6: Параллельные и перпендикулярные линии
Последнее обновление
Сохранить как PDF
Идентификатор страницы
61388
Цели обучения
Определение наклона параллельных и перпендикулярных линий.
Найти уравнения параллельных и перпендикулярных линий
Определение параллели и перпендикуляра
Параллельные линии — это линии в одной плоскости, которые никогда не пересекаются. Две невертикальные линии в одной плоскости с наклонами \(m_{1}\) и \(m_{2}\) параллельны, если их наклоны одинаковы, \(m_{1}=m_{2}\) . Рассмотрим следующие две линии:
Рассмотрим соответствующие им графики:
Рисунок \(\PageIndex{1}\)
Обе линии имеют наклон \(m=\frac{3}{4} \) и, следовательно, параллельны.
Перпендикулярные прямые — это прямые, лежащие в одной плоскости и пересекающиеся под прямым углом (\(90\) градусов). Две невертикальные линии в одной плоскости с наклонами \(m_{1}\) и \(m_{2}\) перпендикулярны, если произведение их наклонов равно \(−1: m1⋅m2=−1\) . Мы можем найти \(m_{1}\) и получить \(m_{1}=\frac{−1}{m_{2}}\). В этой форме мы видим, что перпендикулярные линии имеют наклоны, которые являются отрицательными обратными величинами или противоположными обратными величинами. Например, если задан наклон
\(m=-\frac{5}{8}\)
, то наклон перпендикулярной линии обратно пропорционален:
\(m_{\perp}=\frac{8}{5}\)
Математическая запись \(m_{⊥}\) читается как «\(m\) перпендикулярно». Мы можем проверить, что два наклона образуют перпендикулярные линии, если их произведение равно \(−1\).
Геометрически мы замечаем, что если линия имеет положительный наклон, то любая перпендикулярная линия будет иметь отрицательный наклон. Кроме того, подъем и разбег между двумя перпендикулярными линиями меняются местами.
Рисунок \(\PageIndex{2}\)
Наклоны перпендикулярных прямых противоположны обратным величинам, поэтому не забудьте найти обратную величину и изменить знак. Другими словами,
Если \(m=\frac{a}{b}\), то \(m_{\perp}=-\frac{b}{a}\)
Определение наклона перпендикуляра линию можно выполнить мысленно. Ниже приведены некоторые примеры
Данный уклон
Наклон перпендикулярной линии
\(m=\frac{1}{2}\)
\(м_{\перп}=-2\)
\(m=-\frac{3}{4}\)
\(m_{\perp}=\frac{4}{3}\)
\(м=3\)
\(m_{\perp}=-\frac{1}{3}\)
\(т=-4\)
\(m_{\perp}=\frac{1}{4}\)
Таблица \(\PageIndex{1}\)
Пример \(\PageIndex{1}\)
Определить наклон прямой, параллельной \(y=−5x+3\).
Решение :
Поскольку данная линия имеет форму пересечения наклона, мы можем видеть, что ее наклон равен \(m=−5\). Таким образом, наклон любой линии, параллельной данной линии, должен быть одинаковым, \(m_{∥}=−5\). Математическая запись \(m_{∥}\) читается как «\(m\) параллельно».
Ответ :
\(m_{∥}=−5\)
Пример \(\PageIndex{2}\)
Определить наклон линии, перпендикулярной \(3x−7y=21\) .
Решение :
Сначала найдите \(y\) и представите прямую в виде точки пересечения.
В этой форме мы можем видеть, что наклон данной линии равен \(m=\frac{3}{7}\), и, таким образом, \(m_{⊥}=−\frac{7}{3} \).
Ответ :
\(m_{⊥}=−\frac{7}{3}\)
Упражнение \(\PageIndex{1}\)
Найдите наклон линии, перпендикулярной \( 15х+5у=20\).
Ответить
\(m_{\perp}=\frac{1}{3}\)
Нахождение уравнений параллельных и перпендикулярных прямых
Мы видели, что график прямой полностью определяется двумя точками или одной точкой и ее наклоном. Часто вас будут просить найти уравнение линии с учетом некоторого геометрического соотношения, например, параллельна ли линия другой линии или перпендикулярна ей.
Пример \(\PageIndex{3}\)
Найдите уравнение прямой, проходящей через \((6, −1)\) и параллельной \(y=\frac{1}{2}x+2 \)
Решение
Здесь заданная линия имеет наклон \(m=\frac{1}{2}\), а наклон параллельной прямой равен \(m_{∥}=\frac{1}{2 }\). Поскольку вам дана точка и наклон, используйте форму точки-наклона линии, чтобы определить уравнение.
Важно иметь геометрическое понимание этого вопроса. Нас попросили найти уравнение прямой, параллельной другой прямой, проходящей через определенную точку.
Рисунок \(\PageIndex{3}\)
Через точку \((6, −1)\) мы нашли параллельную прямую, \(y=\frac{1}{2 }x−4\), показан пунктиром. Обратите внимание, что наклон такой же, как у данной линии, но точка пересечения \(y\) отличается. Если иметь в виду геометрическую интерпретацию, то будет легче запомнить процесс, необходимый для решения задачи.
Пример \(\PageIndex{4}\)
Найдите уравнение прямой, проходящей через \((−1, −5)\) и перпендикулярной \(y=−\frac{1}{4}x +2\).
Решение :
Данная линия имеет наклон \(m=−\frac{1}{4}\), поэтому \(m_{⊥}=+\frac{4}{1}=4\ ). Подставьте этот наклон и данную точку в форму точка-наклон.
Геометрически мы видим, что линия \(y=4x−1\), показанная пунктиром ниже , проходит через \((−1, −5)\) и перпендикулярен данной прямой.
Рисунок \(\PageIndex{4}\)
Не всегда данная линия имеет форму пересечения наклона. Часто приходится выполнять дополнительные действия для определения уклона. Общие шаги для нахождения уравнения линии изложены в следующем примере.
Пример \(\PageIndex{5}\)
Найдите уравнение прямой, проходящей через \((8, −2)\) и перпендикулярной \(6x+3y=1\).
Решение :
Шаг 1 : Найдите уклон \(м\). Сначала найдите наклон данной линии. Чтобы сделать это, найдите \(y\), чтобы изменить стандартную форму на форму пересечения наклона, \(y=mx+b\).
В этой форме вы можете видеть, что наклон равен \(m=−2=−\ frac{2}{1}\), и, таким образом, \(m_{⊥}=\frac{−1}{−2}=+\frac{1}{2}\).
Шаг 2 : Подставьте найденный вами наклон и заданную точку в форму уравнения точки-наклона для прямой. В этом случае наклон равен \(m_{⊥}=\frac{1}{2}\), а заданная точка равна \((8, −2)\).
Найдите уравнение прямой, перпендикулярной \(x−3y =9\) и проходящей через \((−\frac{1}{2}, 2)\).
Ответить
\(y=-3x+\frac{1}{2}\)
При нахождении уравнения прямой, перпендикулярной горизонтальной или вертикальной линии, лучше всего учитывать геометрическую интерпретацию.
Пример \(\PageIndex{7}\)
Найдите уравнение прямой, проходящей через \((−3, −2)\) и перпендикулярной \(y=4\).
Решение :
Мы знаем, что \(y=4\) является горизонтальной линией, и мы хотим найти перпендикулярную прямую, проходящую через \((−3, −2)\).
Рисунок \(\PageIndex{5}\)
Если провести линию перпендикулярно заданной горизонтальной линии, результатом будет вертикальная линия.
Рисунок \(\PageIndex{6}\)
Уравнения вертикальных прямых имеют вид \(x=k\). Поскольку он должен проходить через \((−3, −2)\), мы заключаем, что \(x=−3\) является уравнением. Все упорядоченные парные решения вертикальной линии должны иметь одну и ту же координату \(x\).
Ответ :
\(x=−3\)
Мы можем переписать уравнение любой горизонтальной линии \(y=k\) в форме пересечения наклона следующим образом:
\(y=0x +k\)
Записав в этой форме, мы видим, что наклон равен \(m=0=\frac{0}{1}\). Если мы попытаемся найти наклон перпендикулярной линии, найдя обратную обратную, мы столкнемся с проблемой: \(m_{⊥}=−\frac{1}{0}\), которая не определена. Вот почему мы позаботились о том, чтобы ограничить определение двумя невертикальными линиями. Помните, что горизонтальные линии перпендикулярны вертикальным линиям.
Ключевые выводы
Параллельные линии имеют одинаковый наклон.
Перпендикулярные линии имеют наклоны, противоположные обратным. Другими словами, если \(m=\frac{a}{b}\), то \(m_{⊥}=-\frac{b}{a}\).
Чтобы найти уравнение прямой, сначала используйте данную информацию для определения наклона. Затем используйте наклон и точку на линии, чтобы найти уравнение, используя форму точка-наклон.
Горизонтальные и вертикальные линии перпендикулярны друг другу.
Упражнение \(\PageIndex{3}\) Параллельные и перпендикулярные линии
Определение наклона параллельных и перпендикулярных линий.
\(y=−\frac{3}{4}x+8\)
\(y=\frac{1}{2}x−3\)
\(у=4х+4\)
\(у=-3x+7\)
\(y=−\frac{5}{8}x−12\)
\(y=\frac{7}{3}x+\frac{3}{2}\)
\(у=9x−25\)
\(у=-10x+15\)
\(у=5\)
\(х=-12\)
\(х-у=0\)
\(х+у=0\)
\(4x+3y=0\)
\(3x−5y=10\)
\(−2x+7y=14\)
\(−x−y=\frac{1}{5}\)
\(\frac{1}{2}x−\frac{1}{3}y=-1\)
\(−\frac{2}{3}x+\frac{4}{5}y=8\)
\(2x−\frac{1}{5}y=\frac{1}{10}\)
\(−\frac{4}{5}x−2y=7\)
Ответить
1. \(m_{∥}=−\frac{3}{4}\) и \(m_{⊥}=\frac{4}{3}\)
3. \(m_{∥}=4\) и \(m_{⊥}=−\frac{1}{4}\)
5. \(m_{∥}=−\frac{5}{8}\) и \(m_{⊥}=\frac{8}{5}\)
7. \(m_{∥}=9\) и \(m_{⊥}=−\frac{1}{9}\)
9. \(m_{∥}=0\) и \(m_{⊥}\) не определены
11. \(m_{∥}=1\) и \(m_{⊥}=−1\)
13. \(m_{∥}=−\frac{4}{3}\) и \(m_{⊥}=\frac{3}{4}\)
15. \(m_{∥}=\frac{2}{7}\) и \(m_{⊥}=−\frac{7}{2}\)
17. \(m_{∥}=\frac{3}{2}\) и \(m_{⊥}=-\frac{2}{3}\)
19. \(m_{∥}=10\) и \(m_{⊥}=−\frac{1}{10}\)
Упражнение \(\PageIndex{4}\) Параллельные и перпендикулярные линии
Определите, являются ли линии параллельными, перпендикулярными или ни тем, ни другим.
Кубический и квадратный корень в одном уравнении. Решение примера.
Попробуем решить иррациональное уравнение методом возведения обеих частей уравнения в одну и ту же степень. Напомним его алгоритм:
Переходим к более простому уравнению, для чего один или большее число раз выполняем по кругу три следующих действия:
Уединяем радикал.
Возводим обе части уравнения в одну и ту же степень.
Упрощаем вид полученного после возведения в степень уравнения.
Решаем полученное уравнение.
Отсеиваем посторонние корни, если раннее мы проводили возведение в четную степень.
Начнем с первого прохода тройки действий – уединим радикал, возведем обе части в степень и упростим полученное уравнение.
Уединение радикала приводит к уравнению .
Теперь возведем обе части уравнения в квадрат, что позволит в дальнейшем избавиться от корня в левой части. Имеем .
Упрощаем вид полученного уравнения при помощи преобразования уравнений. Отталкиваясь от определения корня, заменяем выражение в левой части уравнения тождественно равным ему выражением 2·x+1, это дает уравнение . Что касается дальнейшего упрощения вида уравнения, то целесообразно по одному из свойств корней вторую степень отправить под кубический корень, то есть, перейти к уравнению .
Как видно, первый проход цикла тройки действий (уединение радикала, возведение обеих частей уравнения в степень и упрощение вида уравнения) позволил избавиться от одного корня, но остался еще один корень. Чтобы избавиться от него, еще раз выполним три уже упомянутых действия.
Радикал у нас уже уединен в правой части. Переходим к возведению в степень.
Степень корня равна трем, поэтому обе части возведем в третью степень: .
Упростим вид полученного уравнения. Для этого заменим выражение в правой части уравнения тождественно равным ему выражением (x+1)2, получим (2·x+1)3=(x+1)2. После этого перенесем это выражение в левую часть: (2·x+1)3−(x+1)2=0. Дальше воспользуемся формулами сокращенного умножения квадрат суммы и куб суммы, раскроем скобки, а также сгруппируем и приведем подобные слагаемые:
8·x3+12·x2+6·x+1−(x2+2·x+1)=0,
8·x3+12·x2+6·x+1−x2−2·x−1=0,
8·x3+(12·x2−x2)+(6·x−2·x)+(1−1)=0,
8·x3+11·x2+4·x=0.
Так мы получили кубическое уравнение. В еще одном проходе тройки действий нет необходимости, так как полученное уравнение не содержит корней, и мы знаем, как решать кубические уравнения. Поэтому, переходим ко второму этапу алгоритма – решению полученного уравнения.
Для решения полученного кубического уравнения подходит метод разложения на множители. После вынесения за скобки переменной x, уравнение принимает вид x·(8·x2+11·x+4)=0, а оно равносильно совокупности двух уравнений x=0 и 8·x2+11·x+4=0. Отсюда первый корень уравнения очевиден: x1=0. Остальные корни найдем, решив квадратное уравнение 8·x2+11·x+4=0. Вычисляем дискриминант D=112−4·8·4=121−128=−7, он отрицательный, следовательно, квадратное уравнение не имеем действительных корней. Таким образом, кубическое уравнение 8·x3+11·x2+4·x=0 имеет единственный корень x1=0.
Остался последний этап решения – отсеивание посторонних корней. В нашем случае этот этап необходим, так как найденный корень может оказаться посторонним для решаемого иррационального уравнения. Причин для этого две. Первая — выше мы проводили возведение обеих частей уравнения квадрат, а, как известно, это преобразование может привести к появлению посторонних корней. Вторая – мы переходили от уравнения к уравнению , при таком переходе происходит расширение ОДЗ, а это может привести к появлению посторонних корней. Итак, отсеем посторонние корни. Сделаем это через проверку подстановкой. Подставляем x1=0 в исходное уравнение:
Так как подстановка дала верное числовое равенство, то x1=0 – корень исходного уравнения. Других корней уравнение не имеет.
Замечание.
На первом этапе мы избавлялись от корней по очереди, в два приема, сначала от квадратного, затем — от кубического. При этом нам пришлось два раза проходить цикл из трех действий – уединение радикала, возведение в степень, упрощение вида. Но можно было избавиться сразу от обоих радикалов, прибегнув к одному возведению в степень. В какую именно степень? Несложно догадаться, что в шестую, или в двенадцатую, или в восемнадцатую, и т.д., то есть, в любую степень, равную кратному показателей корней. Целесообразно брать наименьшее общее кратное (НОК), так как это дает наиболее простое уравнение из возможных. В нашем случае НОК(2, 3)=6, поэтому, следует выполнять возведение в шестую степень. Покажем, как выглядит решение иррационального уравнения при таком подходе.
Что сумма корней. Действие с корнями: сложение и вычитание
Тема про квадратные корни является обязательной в школьной программе курса математики. Без них не обойтись при решении квадратных уравнений. А позже появляется необходимость не только извлекать корни, но и выполнять с ними другие действия. Среди них достаточно сложные: возведение в степень, умножение и деление. Но есть и достаточно простые: вычитание и сложение корней. Кстати, они только на первый взгляд кажутся такими. Выполнить их без ошибок не всегда оказывается просто для того, кто только начинает с ними знакомиться.
Что такое математический корень?
Это действие возникло в противовес возведению в степень. Математика предполагает наличие двух противоположных операций. На сложение существует вычитание. Умножению противостоит деление. Обратное действие степени — это извлечение соответствующего корня.
Если в степени стоит двойка, то и корень будет квадратным. Он является самым распространенным в школьной математике. У него даже нет указания, что он квадратный, то есть возле него не приписывается цифра 2. Математическая запись этого оператора (радикала) представлена на рисунке.
Из описанного действия плавно вытекает его определение. Чтобы извлечь квадратный корень из некоторого числа, нужно выяснить, какое даст при умножении на себя подкоренное выражение. Это число и будет квадратным корнем. Если записать это математически, то получится следующее: х*х=х 2 =у, значит √у=х.
Какие действия с ними можно выполнять?
По своей сути корень — это дробная степень, у которой в числителе стоит единица. А знаменатель может быть любым. Например, у квадратного корня он равен двум. Поэтому все действия, которые можно выполнить со степенями, будут справедливы и для корней.
И требования к этим действиям у них одинаковые. Если умножение, деление и возведение в степень не встречают затруднений у учеников, то сложение корней, как и их вычитание, иногда приводит в замешательство. А все потому что хочется выполнить эти операции без оглядки на знак корня. И здесь начинаются ошибки.
По каким правилам выполняется их сложение и вычитание?
Сначала нужно запомнить два категорических «нельзя»:
нельзя выполнять сложение и вычитание корней, как у простых чисел, то есть невозможно записать подкоренные выражения суммы под один знак и выполнять с ними математические операции;
нельзя складывать и вычитать корни с разными показателями, например квадратный и кубический.
Наглядный пример первого запрета: √6 + √10 ≠ √16, но √(6 + 10) = √16 .
Во втором случае лучше ограничиться упрощением самих корней. А в ответе оставить их сумму.
Теперь к правилам
Найти и сгруппировать подобные корни. То есть те, у которых не только стоят одинаковые числа под радикалом, но и они сами с одним показателем.
Выполнить сложение корней, объединенных в одну группу первым действием. Оно легко осуществимо, потому что нужно только сложить значения, которые стоят перед радикалами.
Извлечь корни в тех слагаемых, в которых подкоренное выражение образует целый квадрат. Другими словами, не оставлять ничего под знаком радикала.
Упростить подкоренные выражения. Для этого нужно разложить их на простые множители и посмотреть, не дадут ли они квадрата какого-либо числа. Понятно, что это справедливо, если речь идет о квадратном корне. Когда показатель степени три или четыре, то и простые множители должны давать куб или четвертую степень числа.
Вынести из-под знака радикала множитель, который дает целую степень.
Посмотреть, не появилось ли опять подобных слагаемых. Если да, то снова выполнить второе действие.
В ситуации, когда задача не требует точного значения корня, его можно вычислить на калькуляторе. Бесконечную десятичную дробь, которая высветится в его окошке, округлить. Чаще всего это делают до сотых. А потом выполнять все операции для десятичных дробей.
Это вся информация о том, как выполняется сложение корней. Примеры, расположенные ниже, проиллюстрируют вышесказанное.
Первое задание
Вычислить значение выражений:
а) √2 + 3√32 + ½ √128 — 6√18;
б) √75 — √147 + √48 — 1/5 √300;
в) √275 — 10√11 + 2√99 + √396.
а) Если следовать приведенному выше алгоритму, то видно, что для первых двух действий в этом примере ничего нет. Зато можно упростить некоторые подкоренные выражения.
Например, 32 разложить на два множителя 2 и 16; 18 будет равно произведению 9 и 2; 128 — это 2 на 64. Учитывая это, выражение будет записано так:
√2 + 3√(2 * 16) + ½ √(2 * 64) — 6 √(2 * 9).
Теперь нужно вынести из-под знака радикала те множители, которые дают квадрат числа. Это 16=4 2 , 9=3 2 , 64=8 2 . Выражение примет вид:
√2 + 3 * 4√2 + ½ * 8 √2 — 6 * 3√2.
Нужно немного упростить запись. Для этого производится умножение коэффициентов перед знаками корня:
√2 + 12√2 + 4 √2 — 12√2.
В этом выражении все слагаемые оказались подобными. Поэтому их нужно просто сложить. В ответе получится: 5√2.
б) Подобно предыдущему примеру, сложение корней начинается с их упрощения. Подкоренные выражения 75, 147, 48 и 300 будут представлены такими парами: 5 и 25, 3 и 49, 3 и 16, 3 и 100. В каждой из них имеется число, которое можно вынести из-под знака корня:
5√5 — 7√3 + 4√3 — 1/5 * 10√3.
После упрощения получается ответ: 5√5 — 5√3. Его можно оставить в таком виде, но лучше вынести общий множитель 5 за скобку: 5 (√5 — √3).
в) И снова разложение на множители: 275 = 11 * 25, 99 = 11 * 9, 396 = 11 * 36. После вынесения множителей из-под знака корня имеем:
5√11 — 10√11 + 2 * 3√11 + 6√11. После приведения подобных слагаемых получим результат: 7√11.
Пример с дробными выражениями
√(45/4) — √20 — 5√(1/18) — 1/6 √245 + √(49/2).
На множители нужно будет разложить такие числа: 45 = 5 * 9, 20 = 4 * 5, 18 = 2 * 9, 245 = 5 * 49. Аналогично уже рассмотренным, нужно вынести множители из-под знака корня и упростить выражение:
Это выражение требует того, чтобы избавиться от иррациональности в знаменателе. Для этого нужно умножить на √2/√2 второе слагаемое:
5/3 √5 + 16/3 √(½) * √2/√2 = — 5/3 √5 + 8/3 √2.
Для полноты действий нужно выделить целую часть у множителей перед корнями. У первого она равна 1, у второго — 2.
Квадратным корнем из числа X называется число A , которое в процессе умножения самого на себя (A * A ) может дать число X . Т.е. A * A = A 2 = X , и √X = A .
Над квадратными корнями (√x ), как и над другими числами, можно выполнять такие арифметические операции, как вычитание и сложение. Для вычитания и сложения корней их нужно соединить посредством знаков, соответствующих этим действиям (например √x — √y ). А потом привести корни к их простейшей форме — если между ними окажутся подобные, необходимо сделать приведение. Оно заключается в том, что берутся коэффициенты подобных членов со знаками соответствующих членов, далее заключаются в скобки и выводится общий корень за скобками множителя. Коэффициент, который мы получили, упрощается по обычным правилам.
Шаг 1. Извлечение квадратных корней
Во-первых, для сложения квадратных корней сначала нужно эти корни извлечь. Это можно будет сделать в том случае, если числа под знаком корня будут полными квадратами. Для примера возьмем заданное выражение √4 + √9 . Первое число 4 является квадратом числа 2 . Второе число 9 является квадратом числа 3 . Таким образом, можно получить следующее равенство: √4 + √9 = 2 + 3 = 5 . Все, пример решен. Но так просто бывает далеко не всегда.
Шаг 2. Вынесение множителя числа из-под корня
Если полных квадратов нет под знаком корня, можно попробовать вынести множитель числа из-под знака корня. Для примера возьмём выражение √24 + √54 .
Рассматривая данный пример, мы получаем вынос множителя из-под знака корня, тем самым упрощая заданное выражение.
Шаг 3. Сокращение знаменателя
Рассмотрим следующую ситуацию: сумма двух квадратных корней — это знаменатель дроби, например, A / (√a + √b) . Теперь перед нами стоит задача «избавиться от иррациональности в знаменателе». Воспользуемся следующим способом: умножаем числитель и знаменатель дроби на выражение √a — √b .
Формулу сокращённого умножения мы теперь получаем в знаменателе: (√a + √b) * (√a — √b) = a — b .
Аналогично, если в знаменателе имеется разность корней: √a — √b , числитель и знаменатель дроби умножаем на выражение √a + √b .
Шаг 4. Вычисление приблизительного значения на калькуляторе
Если вам требуется только приблизительное значение, это можно сделать на калькуляторе путём подсчёта значения квадратных корней. Отдельно для каждого числа вычисляется значение и записывается с необходимой точностью, которая определяется количеством знаков после запятой. Далее совершаются все требуемые операции, как с обычными числами.
Пример вычисления приблизительного значения
Необходимо вычислить приблизительное значение данного выражения √7 + √5 .
В итоге получаем:
√7 + √5 ≈ 2,65 + 2,24 = 4,89 .
Обратите внимание: ни при каких условиях не следует производить сложение квадратных корней, как простых чисел, это совершенно недопустимо. То есть, если сложить квадратный корень из пяти и из трёх, у нас не может получиться квадратный корень из восьми.
Полезный совет: если вы решили разложить число на множители, для того, чтобы вывести квадрат из-под знака корня, вам необходимо сделать обратную проверку, то есть перемножить все множители, которые получились в результате вычислений, и в конечном результате этого математического расчёта должно получиться число, которое нам было задано первоначально.
Формулы корней. Свойства квадратных корней.
Внимание! К этой теме имеются дополнительные материалы в Особом разделе 555. Для тех, кто сильно «не очень…» И для тех, кто «очень даже…»)
В предыдущем уроке мы разобрались, что такое квадратный корень . Пришла пора разобраться, какие существуют формулы для корней , каковы свойства корней , и что со всем этим можно делать.
Формулы корней, свойства корней и правила действий с корнями — это, по сути, одно и то же. Формул для квадратных корней на удивление немного. Что, безусловно, радует! Вернее, понаписать всяких формул можно много, но для практической и уверенной работы с корнями достаточно всего трёх. Все остальное из этих трёх проистекает. Хотя и в трех формулах корней многие плутают, да…
Начнём с самой простой. Вот она:
Если Вам нравится этот сайт…
Кстати, у меня есть ещё парочка интересных сайтов для Вас.)
Можно потренироваться в решении примеров и узнать свой уровень. Тестирование с мгновенной проверкой. Учимся — с интересом!)
можно познакомиться с функциями и производными.
Факт 1. \(\bullet\)
Возьмем некоторое неотрицательное число \(a\)
(то есть \(a\geqslant 0\)
). Тогда (арифметическим) квадратным корнем из числа \(a\)
называется такое неотрицательное число \(b\)
, при возведении которого в квадрат мы получим число \(a\)
: \[\sqrt a=b\quad \text{то же самое, что }\quad a=b^2\]
Из определения следует, что \(a\geqslant 0, b\geqslant 0\)
. 2=400\\
\hline \end{array}\]
Факт 3. Какие действия можно выполнять с квадратными корнями? \(\bullet\) Сумма или разность квадратных корней НЕ РАВНА квадратному корню из суммы или разности, то есть \[\sqrt a\pm\sqrt b\ne \sqrt{a\pm b}\]
Таким образом, если вам нужно вычислить, например, \(\sqrt{25}+\sqrt{49}\)
, то первоначально вы должны найти значения \(\sqrt{25}\)
и \(\sqrt{49}\)
, а затем их сложить. Следовательно, \[\sqrt{25}+\sqrt{49}=5+7=12\] Если значения \(\sqrt a\)
или \(\sqrt b\)
при сложении \(\sqrt
a+\sqrt b\)
найти не удается, то такое выражение дальше не преобразуется и остается таким, как есть. Например, в сумме \(\sqrt
2+ \sqrt {49}\)
мы можем найти \(\sqrt{49}\)
– это \(7\)
, а вот \(\sqrt
2\)
никак преобразовать нельзя, поэтому \(\sqrt 2+\sqrt{49}=\sqrt
2+7\)
. Дальше это выражение, к сожалению, упростить никак нельзя \(\bullet\)
Произведение/частное квадратных корней равно квадратному корню из произведения/частного, то есть \[\sqrt a\cdot \sqrt b=\sqrt{ab}\quad \text{и}\quad
\sqrt a:\sqrt b=\sqrt{a:b}\]
(при условии, что обе части равенств имеют смысл ) Пример: \(\sqrt{32}\cdot \sqrt 2=\sqrt{32\cdot
2}=\sqrt{64}=8\)
;
\(\sqrt{768}:\sqrt3=\sqrt{768:3}=\sqrt{256}=16\)
;
\(\sqrt{(-25)\cdot (-64)}=\sqrt{25\cdot 64}=\sqrt{25}\cdot \sqrt{64}=
5\cdot 8=40\)
. \(\bullet\)
Пользуясь этими свойствами, удобно находить квадратные корни из больших чисел путем разложения их на множители. Рассмотрим пример. Найдем \(\sqrt{44100}\)
. Так как \(44100:100=441\)
, то \(44100=100\cdot 441\)
. По признаку делимости число \(441\)
делится на \(9\)
(так как сумма его цифр равна 9 и делится на 9), следовательно, \(441:9=49\)
, то есть \(441=9\cdot 49\)
. Таким образом, мы получили: \[\sqrt{44100}=\sqrt{9\cdot 49\cdot 100}=
\sqrt9\cdot \sqrt{49}\cdot \sqrt{100}=3\cdot 7\cdot 10=210\]
Рассмотрим еще один пример: \[\sqrt{\dfrac{32\cdot 294}{27}}=
\sqrt{\dfrac{16\cdot 2\cdot 3\cdot 49\cdot 2}{9\cdot 3}}= \sqrt{
\dfrac{16\cdot4\cdot49}{9}}=\dfrac{\sqrt{16}\cdot \sqrt4 \cdot
\sqrt{49}}{\sqrt9}=\dfrac{4\cdot 2\cdot 7}3=\dfrac{56}3\] \(\bullet\)
Покажем, как вносить числа под знак квадратного корня на примере выражения \(5\sqrt2\)
(сокращенная запись от выражения \(5\cdot
\sqrt2\)
). Так как \(5=\sqrt{25}\)
, то \
Заметим также, что, например, 1) \(\sqrt2+3\sqrt2=4\sqrt2\)
, 2) \(5\sqrt3-\sqrt3=4\sqrt3\) 3) \(\sqrt a+\sqrt a=2\sqrt a\)
. 2\)
, поэтому \(\sqrt{16}=4\)
. А вот извлечь корень из числа \(3\)
, то есть найти \(\sqrt3\)
, нельзя, потому что нет такого числа, которое в квадрате даст \(3\)
. Такие числа (или выражения с такими числами) являются иррациональными. Например, числа \(\sqrt3, \ 1+\sqrt2, \ \sqrt{15}\)
и т.п. являются иррациональными. Также иррациональными являются числа \(\pi\)
(число “пи”, приблизительно равное \(3,14\)
), \(e\)
(это число называют числом Эйлера, приблизительно оно равно \(2,7\)
) и т.д. \(\bullet\)
Обращаем ваше внимание на то, что любое число будет либо рациональным, либо иррациональным. А вместе все рациональные и все иррациональные числа образуют множество, называющееся множеством действительных (вещественных) чисел. Обозначается это множество буквой \(\mathbb{R}\)
. Значит, все числа, которые на данный момент мы знаем, называются вещественными числами.
Факт 5. \(\bullet\)
Модуль вещественного числа \(a\)
– это неотрицательное число \(|a|\)
, равное расстоянию от точки \(a\)
до \(0\)
на вещественной прямой. 2\\
&2>2,25 \end{aligned}\]
Видим, что мы получили неверное неравенство. Следовательно, наше предположение было неверным и \(\sqrt 2-1Заметим, что прибавление некоторого числа к обеим частям неравенства не влияет на его знак. Умножение/деление обеих частей неравенства на положительное число также не влияет на его знак, а умножение/деление на отрицательное число меняет знак неравенства на противоположный! Возводить обе части уравнения/неравенства в квадрат можно ТОЛЬКО ТОГДА, когда обе части неотрицательные. Например, в неравенстве из предыдущего примера возводить обе части в квадрат можно, в неравенстве \(-3
\(\bullet\)
Следует запомнить, что \[\begin{aligned}
&\sqrt 2\approx 1,4\\
&\sqrt 3\approx 1,7 \end{aligned}\]
Знание приблизительного значения данных чисел поможет вам при сравнении чисел!
\(\bullet\)
Для того, чтобы извлечь корень (если он извлекается) из какого-то большого числа, которого нет в таблице квадратов, нужно сначала определить, между какими “сотнями” оно находится, затем – между какими “десятками”, а потом уже определить последнюю цифру этого числа. 2=168\cdot 168=28224\)
. Следовательно, \(\sqrt{28224}=168\)
. Вуаля!
Для того чтобы достойно решить ЕГЭ по математике, прежде всего необходимо изучить теоретический материал, который знакомит с многочисленными теоремами, формулами, алгоритмами и т. д. На первый взгляд может показаться, что это довольно просто. Однако найти источник, в котором теория для ЕГЭ по математике изложена легко и понятно для учащихся с любым уровнем подготовки, — на деле задача довольно сложная. Школьные учебники невозможно всегда держать под рукой. А найти основные формулы для ЕГЭ по математике бывает непросто даже в Интернете.
Почему так важно изучать теорию по математике не только для тех, кто сдает ЕГЭ?
Потому что это расширяет кругозор
. Изучение теоретического материала по математике полезно для всех, кто желает получить ответы на широкий круг вопросов, связанных с познанием окружающего мира. Все в природе упорядоченно и имеет четкую логику. Именно это и отражается в науке, через которую возможно понять мир.
Потому что это развивает интеллект
. Изучая справочные материалы для ЕГЭ по математике, а также решая разнообразные задачи, человек учится логически мыслить и рассуждать, грамотно и четко формулировать мысли. У него вырабатывается способность анализировать, обобщать, делать выводы.
Предлагаем вам лично оценить все преимущества нашего подхода к систематизации и изложению учебных материалов.
Содержимое:
Складывать и вычитать квадратные корни можно только при условии, что у них одинаковое подкоренное выражение, то есть вы можете сложить или вычесть 2√3 и 4√3, но не 2√3 и 2√5. Вы можете упростить подкоренное выражение, чтобы привести их к корням с одинаковыми подкоренными выражениями (а затем сложить или вычесть их).
Шаги
Часть 1
Постигаем основы
1 (выражение под знаком корня). Для этого разложите подкоренное число на два множителя, один из которых является квадратным числом (число, из которого можно извлечь целый корень, например, 25 или 9). После этого извлеките корень из квадратного числа и запишите найденное значение перед знаком корня (под знаком корня останется второй множитель). Например, 6√50 — 2√8 + 5√12. Числа, стоящее перед знаком корня, являются множителями соответствующих корней, а числа под знаком корня – это подкоренные числа (выражения). Вот как решать данную задачу:
6√50 = 6√(25 x 2) = (6 x 5)√2 = 30√2. Здесь вы раскладываете 50 на множители 25 и 2; затем из 25 извлекаете корень, равный 5, и 5 выносите из-под корня. Затем 5 умножаете на 6 (множитель у корня) и получаете 30√2.
2√8 = 2√(4 x 2) = (2 x 2)√2 = 4√2. Здесь вы раскладываете 8 на множители 4 и 2; затем из 4 извлекаете корень, равный 2, и 2 выносите из-под корня. Затем 2 умножаете на 2 (множитель у корня) и получаете 4√2.
5√12 = 5√(4 x 3) = (5 x 2)√3 = 10√3. Здесь вы раскладываете 12 на множители 4 и 3; затем из 4 извлекаете корень, равный 2, и 2 выносите из-под корня. Затем 2 умножаете на 5 (множитель у корня) и получаете 10√3.
2 Подчеркните корни, подкоренные выражения которых одинаковы. В нашем примере упрощенное выражение имеет вид: 30√2 — 4√2 + 10√3. В нем вы должны подчеркнуть первый и второй члены (30√2 и 4√2 ), так как у них одинаковое подкоренное число 2. Только такие корни вы можете складывать и вычитать.
3
Если вам дано выражение с большим количеством членов, многие из которых имеют одинаковые подкоренные выражения, используйте одинарное, двойное, тройное подчеркивание для обозначения таких членов, чтобы облегчить решение этого выражения.
4
У корней, подкоренные выражения которых одинаковы, сложите или вычтите множители, стоящие перед знаком корня, а подкоренное выражение оставьте прежним (не складывайте и не вычитайте подкоренные числа!
). Идея в том, чтобы показать, сколько всего корней с определенным подкоренным выражением содержится в данном выражении.
30√2 — 4√2 + 10√3 =
(30 — 4)√2 + 10√3 =
26√2 + 10√3
Часть 2
Практикуемся на примерах
1 Пример 1: √(45) + 4√5.
Упростите √(45). Разложите 45 на множители: √(45) = √(9 x 5).
Вынесите 3 из-под корня (√9 = 3): √(45) = 3√5.
Теперь сложите множители у корней: 3√5 + 4√5 = 7√5
2 Пример 2: 6√(40) — 3√(10) + √5.
Упростите 6√(40). Разложите 40 на множители: 6√(40) = 6√(4 x 10).
Вынесите 2 из-под корня (√4 = 2): 6√(40) = 6√(4 x 10) = (6 x 2)√10.
Перемножьте множители перед корнем и получите 12√10.
Теперь выражение можно записать в виде 12√10 — 3√(10) + √5. Так как у первых двух членов одинаковые подкоренные числа, вы можете вычесть второй член из первого, а первый оставить без изменений.
Вы получите: (12-3)√10 + √5 = 9√10 + √5.
3 Пример 3. 9√5 -2√3 — 4√5. Здесь ни одно из подкоренных выражений нельзя разложить на множители, поэтому упростить это выражение не получится. Вы можете вычесть третий член из первого (так как у них одинаковые подкоренные числа), а второй член оставить без изменений. Вы получите: (9-4)√5 -2√3 = 5√5 — 2√3.
4 Пример 4. √9 + √4 — 3√2.
√9 = √(3 х 3) = 3.
√4 = √(2 х 2) = 2.
Теперь вы можете просто сложить 3 + 2, чтобы получить 5.
Окончательный ответ: 5 — 3√2.
5 Пример 5. Решите выражение, содержащее корни и дроби. Вы можете складывать и вычислять только те дроби, у которых общий (одинаковый) знаменатель. Дано выражение (√2)/4 + (√2)/2.
Найдите наименьший общий знаменатель этих дробей. Это число, которое делится нацело на каждый знаменатель. В нашем примере на 4 и на 2 делится число 4.
Теперь вторую дробь умножьте на 2/2 (чтобы привести ее к общему знаменателю; первая дробь уже приведена к нему): (√2)/2 х 2/2 = (2√2)/4.
Перед суммированием или вычитанием корней обязательно упростите (если возможно) подкоренные выражения.
Предупреждения
Никогда не суммируйте и не вычитайте корни с разными подкоренными выражениями.
Никогда не суммируйте и не вычитайте целое число и корень, например, 3 + (2x) 1/2 .
Примечание: «х» в одной второй степени и квадратный корень из «х» – это одно и то же (то есть x 1/2 = √х).
2 — 2x + 1=0$?
Задавать вопрос
спросил
Изменено
3 года, 11 месяцев назад
Просмотрено
1к раз
$\begingroup$
Я знаю, что есть корни, потому что если мы примем уравнение как функцию и зададим -3 и 1 как $x$: 92 — 2(1) + 1 > 0
$$
Должен иметь корень между $[-3,1]$. Тем не менее, корень очень жесткий, и он появился на школьном тесте. Как я могу решить это просто?
Были даны варианты -10, -5, 0 , 5 и 10. Примечание: мы даже не выучили формулу кубического корня. В тесте были только логические задачи, и я не использовал никаких вычислений или сложных вещей. Поэтому должен быть более простой способ без использования концепций или формул кубического корня.
полиномы
конкурс-математика 92-3(ab+bc+ca)\right)$$
Формулы Виета — это формулы, связывающие коэффициенты многочлена с суммами и произведениями его корней.
$\endgroup$
4
$\begingroup$
Подсказка: используйте формулы Ньютона и формулы Виета
https://brilliant.org/wiki/newtons-identities/
$\endgroup$
2
алгебраическое предварительное исчисление — сумма кубических корней комплексно-сопряженных чисел
спросил
Изменено
1 год, 9 месяцев назад
Просмотрено
3к раз
$\begingroup$
При решении следующего кубического уравнения: 93 — 15x — 4 = 0$$
Когда я подсчитал на ручном калькуляторе, получилось ровно 4$. И действительно, когда я подставляю $x=4$ в исходное уравнение, получается решение. Итак, это похоже на правду:
Итак, у нас есть сумма кубических корней из комплексных чисел, которая, тем не менее, дает реальный результат . Итак, я предполагаю, что эти два кубических корня должны быть комплексно-сопряженными друг другу, что, по-видимому, так и есть, судя по тому факту, что числа под кубическими корнями также являются комплексно-сопряженными друг другу (обратите внимание на знаки, отмеченные красным ). Комплексные конъюгаты являются «зеркальными отражениями» друг друга, поэтому при сложении они дают реальный результат. Кубические корни комплексно-сопряженных чисел делят их углы на 3, поэтому результаты должны оставаться комплексно-сопряженными числами, и я полагаю, что именно по этой причине их сумма также дает действительное число. Я прав? 93 — 15x — 4 = 0$$
Почему я хожу по кругу с этим? И какие еще методы я могу использовать, чтобы взломать это и доказать, что эта сумма кубических корней действительно равна действительному числу $4$?
Inb4: Я уже установил геометрически, что эта сумма кубов действительно равна $4$, но теперь я хотел бы доказать это алгебраически и изучить общий метод работы с такими суммами кубов комплексно-сопряженных чисел.
Редактировать: Все ответы до сих пор, кажется, основаны на предположении, что я знаю, что это сложное выражение уже равно 2 (например, путем восстановления исходного кубического уравнения и нахождения его рациональных корней). Что меня больше интересует, так это то, как найти эквивалентные действительные решения, когда восстановление исходного кубического уравнения не работает, потому что оно не может быть решено с помощью теоремы о рациональных корнях. 93-15x-4$ на $x-4$.
$\endgroup$
4
$\begingroup$
Это не всегда будет работать так хорошо, и, вероятно, есть более короткий или, по крайней мере, более эффективный способ решения геометрических уравнений, но вот один подход, который работает для множества особых случаев, включая $\sqrt [3]{2+11i}+\sqrt[3]{2-11i}$.
Используя полярную форму комплексных чисел, мы видим, что искомое значение равно $2\sqrt{5}\cos\left(\dfrac{\arctan\frac{11}{2}}{3} \справа)$. Мы можем вычислить этот косинус, нарисовав на комплексной плоскости треугольник с вершинами $0$, $2$ и $2+11i$ и разделив угол на три части при $0$. 92-200000\шляпа{х}+1000000=0$. Проверка рациональных корней говорит нам, что у нас $\hat{x}=-5,1,-\frac{10}{11},\frac{10}{11}$ в качестве корней. Остальные являются корнями полученного квадратного числа: $\hat{x}=4(-2\pm\sqrt{3})$.
Мы знаем, что $\hat{x}$ должно быть положительным, так что это либо $1$, либо $10/11$, а проверка других уравнений показывает, что $\hat{x}=1$ верно, что приводит к косинусу равно $\dfrac{2}{\sqrt{5}*1}$, а исходное значение равно $2\sqrt{5}*\dfrac{2}{\sqrt{5}*1}=4\checkmark$.
$\endgroup$
1
$\begingroup$
В качестве альтернативы мы могли бы показать алгебраически, что
Думаю, у вас возникнут проблемы с алгебраическим доказательством этого .
С точки зрения -неупорядоченного -поля, где доступны только операции $+ — \times\ \div$, это невозможно, так как три корня каждого числа неотличимы друг от друга, и для некоторых пар этих значения, тождество ложно.
Итак, нам предстоит работать в комплексной плоскости. Суть задачи состоит в том, чтобы найти действительную компоненту $\sqrt[3]{2+11i}$. «Очевидный» способ сделать это — вычислить арктангенс, разделить угол на три, а затем взять косинус. Реальное выполнение этого вычисления в общем случае и без приближения выходит за рамки возможностей нашей современной математики.
Это задача на три сечения угла, и это означает, что вы всегда будете возвращаться к решению кубического уравнения.
Я бы предложил подойти к проблеме факторизации иначе, используя алгоритм Кронекера (более старый, но более понятный) или алгоритм Кантора-Цассенхауса (наиболее широко применяемый в системах компьютерной алгебры). См. статью в Википедии о полиномиальной факторизации.
Идея состоит в том, чтобы разложить полином над полем рациональных чисел, а не над комплексным полем. Если ваш ответ не является рациональным, это не сработает (алгоритм сообщит вам, что полином неприводим), но если один из ваших корней является рациональным числом (например, 4), этот метод найдет его без циклических аргументов. .
Что делать, если ваш ответ нелогичен? Тогда лучшее, на что вы можете надеяться, — это найти неприводимый многочлен с одним действительным корнем, который и будет вашим решением, и я думаю, что вы упираетесь в фундаментальные теоретические ограничения, чтобы добиться большего.
$\endgroup$
$\begingroup$
Я думаю, что весь ваш путь совершенно неверен, потому что $\sqrt[3]{2+11i}$ — это набор из трех чисел, а $\sqrt[3]{2-11i}$ — это набор из трех чисел.
$\endgroup$
2
$\begingroup$
Вот странный подход: теорема о рациональном корне.
Соли — это сложные вещества, которые
диссоциируют, образуя катионы
металла (или Nh5+) и анионы
кислотного остатка. Кислые соли при
диссоциации дают дополнительно
ионы H+, а основные соли — ионы
гидроксила OH-. Среди солей выделяют
двойные, смешанные и комплексные.
Получение
1. Взаимодействием металлов и
металло-неметаллов с неметаллами:
3. Вытеснением более активным
металлом более поссивного из его соли:
CuSO4 + Fe = FeSO4 + Cu.
4. Взаимодействием основных,
амфотерных и кислотных оксидов:
Li2O + ZnO = Li2ZnO2,
CaO + SiO2 = CaSiO3,
Al2O3 + 3SO3 = Al2(SO4)3.
5. Взаимодействием основных,
амфотерных и кислотных оксидов с
основными, амфотерными и кислотными
гидроксидами:
2LiOH + ZnO = Li2ZnO2 + 2h3O,
Ca(OH)2 + CO2 = CaCO3 + h3O,
Al2O3 + 3h3SO4 = Al2(SO4)3 + 3h3O.
6. Взаимодействием основных,
амфотерных гидрооксидов с кислотами:
Al(OH)3 + KOH = K[Al(OH)4],
Cu(OH)2 + h3SO4 = CuSO4 + 2h3O,
Zn(OH)2 + 2HBr = ZnBr2 + 2h3O.
7. Путем обменных реакций основных,
амфотерных гидрооксидов, кислот и
солей с солями:
Ba(OH)2 + Na2SO4 = BaSO4 + 2NaOH,
Ca(OH)2 + Ca(HCO3)2 = 2CaCO3 +
2h3O,
KOH + AlCl3 = Al(OH)Cl2 + KCl,
h3SO4 + Ba(NO3)2 = BaSO4+ 2HNO3,
HCl + Mg(OH)Cl = MgCl2 + h3O,
Ca3(PO4)2 + 4h4PO4 = 3Ca(h3PO4)2,
FeCl3 + 3 KSCN = Fe(SCN)3 + 3KCl.
8. Термическим разложением солей:
2KClO3 = 2KCl + 3O2,
2NaNO3 = 2NaNO2 + O2.
Свойства
1. Диссоциируют в растворах или
расплавах:
2. Способны подвергаться гидролизу
(кроме солей, образованных
одновременно сильными
одноосновными кислотами и
щелочными металлами):
3. Могут взаимодействовать с
основными и амфотерными
гидроксидами, кислотами и солями,
вступая в реакции обмена (смотри
выше: пункт 7).
4. Способны вступать в многочисленные
окислительно-восстановительные
реакции, реакции
диспропорционирования:
3KClO = KClO3 + 2KCl,
2FeCl3 + h3S = S + 2FeCl2 + 2HCl.
Как пример реакций получения
комплексных солей рассмотрим
образование и использование красной
и желтой кровяной соли:
FeCl2 + 6KCN = 2KCl + K4[Fe(CN)6] ,
FeCl3 + 6KCN = 3KCl + K3[Fe(CN)6] ,
3 FeCl2 + 2 K3[Fe(CN)6]
= 6KCl + Fe3[Fe(CN)6]2,
4 FeCl3 + 3 K4[Fe(CN)6] =
12KCl + Fe4[Fe(CN)6]3.
K4[Fe(CN)6] — гексоцианоферрат
(II) калия или желтая кровяная
соль,
K3[Fe(CN)6] — гексоцианоферрат
(III) калия или красная кровяная
соль,
Fe3[Fe(CN)6]2 — гексоцианоферрат
(III) железа (II) или турнбулева
синь,
Fe4[Fe(CN)6]3 — гексоцианоферрат
(II) железа (III) или берлинская
лазурь.
Примером двойной соли могут служить
алюмокалиевые квасцы KAl(SO4)2 —
здесь два разных катиона и один анион:
Примером смешанной соли является
хлорная известь Ca(OCl)Cl. При ее
диссоциации образуется один катион
Ca2+ и два аниона Cl- и OCl-.
Гексацианоферрат(II) калия
[5].
Токсичность
Нейтральное вещество, поскольку анион (см. выше) не разлагается в воде и внутри человеческого организма. Летальная доза (LD50) для крыс при приёме перорально составляет 6400 мг/кг[6]. 4- -> Cu2[Fe(CN)6]v}}}
Может использоваться для получения синильной кислоты:
Для того, чтобы запомнить формулу жёлтой кровяной соли K4[Fe(CN)6]{\displaystyle {\ce {K4[Fe(CN)6]}}}
и не спутать её с красной кровяной сольюK3[Fe(CN)6],{\displaystyle {\ce {K3[Fe(CN)6],}}}
существуют мнемонические правила:
Число атомов калия соответствует числу букв в английских названиях солей: «gold» — 4 буквы, то есть 4 атома калия — жёлтая кровяная соль K4[Fe(CN)6]{\displaystyle {\ce {K4[Fe(CN)6]}}}
. «Red» — 3 буквы, то есть 3 атома калия — красная кровяная соль — K3[Fe(CN)6]{\displaystyle {\ce {K3[Fe(CN)6]}}}
.
«Калий три — будет красный». Мнемоника построена на игре слов: «три» (число) и «три» (повелительное наклонение глагола «тереть»). Как известно при потирании, например, кожи, последняя краснеет.
Примечания
↑ 12 Желтая или кровещелочная соль // Энциклопедический словарь Брокгауза и Ефрона : в 86 т. (82 т. и 4 доп.). — СПб., 1890—1907.
↑ 123456Некрасов Б.В. Основы общей химии. — М., 1973. — Т. 2. — С. 360.
↑ 123 Тананаев, 1971, с. 38-39.
↑ Тананаев, 1971, с. 5.
↑ Ф. Файгль, В.Ангер. Капельный анализ неорганических веществ. — 1976. — Т. 1. — С. 242.
↑ POTASSIUM FERROCYANIDE (неопр. ). hazard.com. Дата обращения: 16 января 2016.
↑ Аналитические методы определения цинка
Литература
Тананаев И.В. и др. Химия ферроцианидов. — М., 1971.
Гексацианоферрат, калия, Гексацианоферра, ка, лия, железистосинеро, дистый, ка, лий, ферроциани, ка, лия, гексацианоферроа, ка, лия, жёлтая, кровяная, соль, неорганическое, соединение, из, группы, гексацианоферратов, комплексная, соль, состава, displaystyle, с. Geksacianoferra t II ka liya zhelezistosinero distyj ka lij ferrociani d ka liya geksacianoferroa t ka liya zhyoltaya krovyanaya sol neorganicheskoe soedinenie iz gruppy geksacianoferratov kompleksnaya sol sostava K 4 Fe CN 6 displaystyle ce K4 Fe CN 6 soderzhashaya v anione dvuhvalentnoe zhelezo vydelyayushayasya iz vodnogo rastvora v vide kristallogidrata K 4 Fe CN 6 3 H 2 O displaystyle ce K4 Fe CN 6 3h3O Geksacianoferrat II kaliyaObshieSistematicheskoenaimenovanie Geksacianoferrat II kaliyaHim formula C6N6FeK4Rac formula K4 Fe CN 6 Fizicheskie svojstvaSostoyanie bezvodnyj bescvetnye kristally trigidrat bledno zhyoltye kristallyMolyarnaya massa 368 35 g molPlotnost trigidrata 1 85 g sm Termicheskie svojstvaTemperatura plavleniya 69 71 C razlozheniya 650 CKlassifikaciyaReg nomer CAS 13943 58 3PubChem 9605257Reg nomer EINECS 237 722 2SMILES C N C N C N C N C N C N K K K K Fe 2 C N C N C N C N C N C N Fe 4 K K K K InChI InChI 1S 6CN Fe 4K c6 1 2 q6 1 2 4 1 InChI 1S 6CN Fe 4K c6 1 2 q 4 4 1XOGGUFAVLNCTRS UHFFFAOYSA N OCPOWIWGGAATRP UHFFFAOYSA NKodeks Alimentarius E536ChEBI 30059ChemSpider 20162028BezopasnostKratkie harakter opasnosti H h512 EUH032Mery predostor P P273 P260NFPA 704 010Privedeny dannye dlya standartnyh uslovij 25 C 100 kPa esli ne ukazano inoe Mediafajly na Vikisklade Soderzhanie 1 Istoriya otkrytiya 2 Nazvaniya 3 Poluchenie 4 Fizicheskie i himicheskie svojstva 5 Toksichnost 6 Primenenie 7 Mnemonicheskoe pravilo 8 Primechaniya 9 LiteraturaIstoriya otkrytiya PravitV 1752 godu Maker en obnaruzhil chto pri kipyachenii berlinskoj lazuri s rastvorom shyolochi edkogo kali sinij cvet eyo ischezaet a v rastvor perehodit zheltoe veshestvo Issledovaniya Bertolle 1787 Gej Lyussaka i Berceliusa 1819 pokazali chto veshestvo eto soderzhit kalij zhelezo II i ostatok sinilnoj kisloty snachala ego prinimali za dvojnuyu sol Vposledstvii usiliyami Gmelina Gej Lyussaka i Libiha vyyasnilos chto eto kalievaya sol zhelezistosinerodistoj kisloty 1 Nazvaniya PravitPomimo zhyoltoj krovyanoj soli v ESBE otmecheny takzhe sleduyushie trivialnye nazvaniya 1 zhyoltoe sin kali zhyoltaya sol kroveshelochnaya sol Nazvanie zhyoltaya krovyanaya sol poyavilos iz za togo chto ranshe eyo poluchali prokalivaniem othodov boen soderzhashih krov s potashom i zheleznymi opilkami Eto a takzhe zhyoltyj cvet kristallov obuslovili nazvanie soedineniya 2 Poluchenie PravitV nastoyashee vremya v promyshlennosti poluchayut iz otrabotannoj massy posle ochistki gazov na gazovyh zavodah soderzhit cianistye soedineniya etu massu obrabatyvayut suspenziej Ca OH 2 filtrat soderzhashij Ca2 Fe CN 6 pererabatyvayut putyom posledovatelnogo dobavleniya snachala KCl a zatem K2CO3 On takzhe mozhet byt poluchen putyom vzaimodejstviya suspenzii FeS s vodnym rastvorom KCN Reakciyu mozhno predstavit sleduyushej shemoj 1 cianid kaliya perevodit Fe 2 displaystyle ce Fe 2 v belyj osadok geksacianoferrata II zheleza II 3 Fe 2 6 CN Fe 2 Fe CN 6 displaystyle ce 3Fe 2 6CN gt Fe2 Fe CN 6 v a ne v cianid zheleza II kak schitalos ranee chto vytekaet iz vzaimodejstviya etogo osadka so shyolochyu Fe 2 Fe CN 6 4 KOH 2 Fe OH 2 K 4 Fe CN 6 displaystyle ce Fe2 Fe CN 6 4KOH gt 2Fe OH 2v K4 Fe CN 6 2 2 zatem osadok rastvoryaetsya v izbytke KCN s obrazovaniem zhyoltoj krovyanoj soli Fe 2 Fe CN 6 4 KCN K 4 Fe CN 6 2 Fe CN 2 displaystyle ce Fe2 Fe CN 6 4KCN gt K4 Fe CN 6 2Fe CN 2 Fizicheskie i himicheskie svojstva PravitSvetlo zhyoltye kristally s tetragonalnoj sushestvuyut takzhe rombicheskaya i monoklinnaya modifikacii reshyotkoj sushestvuyushie v vide trigidrata K 4 Fe CN 6 3 H 2 O displaystyle ce K4 Fe CN 6 3h3O 3 Plotnost 1 853 g sm pri 17 C Rastvorimost v vode 35 8 g 100 g pri 25 C ona umenshaetsya v prisutstvii ammiaka ili drugih solej kaliya V absolyutnom spirte nerastvorim no rastvoryaetsya v smesyah spirta s vodoj Malo rastvorim v metanole 0 024 mol l prakticheski ne rastvoryaetsya v efire piridine aniline etilacetate zhidkih hlore i ammiake 3 Geksacianoferrat II kaliya diamagniten pri nizkih temperaturah yavlyaetsya segnetoelektrikom 3 Vyshe 120 C po drugim dannym vyshe 87 3 C istochnik prevrashaetsya v bezvodnuyu sol s plotnostyu 1 935 g sm Vyshe 650 C razlagaetsya 2 3 K 4 Fe CN 6 t Fe 3 C 5 C 3 N 2 12 KCN displaystyle ce 3K4 Fe CN 6 gt t Fe3C 5C 3N2 12KCN V reakcii s koncentrirovannoj solyanoj kislotoj vydelyaetsya belyj osadok zhelezistosinerodistoj kisloty H 4 Fe CN 6 displaystyle ce h5 Fe CN 6 2 S koncentrirovannoj sernoj kislotoj reagiruet po uravneniyu K 4 Fe CN 6 6 H 2 SO 4 6 H 2 O t 2 K 2 SO 4 FeSO 4 3 NH 4 2 SO 4 6 CO displaystyle ce K4 Fe CN 6 6h3SO4 6h3O gt t 2K2SO4 FeSO4 3 Nh5 2SO4 6CO Etim sposobom mozhno polzovatsya v laboratorii dlya polucheniya monooksida ugleroda S solyami metallov v stepeni okisleniya 2 i 3 obrazuet malorastvorimye soedineniya geksacianoferratov II V vodnyh rastvorah okislyaetsya hlorom i drugimi okislitelyami takimi kak peroksid vodoroda do geksacianoferrata III kaliya 2 K 4 Fe CN 6 H 2 O 2 2 HCl 2 K 3 Fe CN 6 2 KCl 2 H 2 O displaystyle ce 2K4 Fe CN 6 h3O2 2HCl gt 2K3 Fe CN 6 2KCl 2h3O Anion Fe CN 6 4 displaystyle ce Fe CN 6 4 ochen prochen ego konstanta nestojkosti po raznym dannym 2 4 ot 4 10 36 do 1 10 35 ne razlagaetsya ni shelochami ni kislotami ustojchiv po otnosheniyu k vozduhu poetomu s tradicionnymi reagentami rastvory geksacianoferratov II ne dayut reakcij ni na Fe 2 displaystyle ce Fe 2 ni na CN displaystyle ce CN 2 Odnako dissociativno razrushit etot kompleks i obnaruzhit v nyom nalichie dvuhvalentnogo zheleza vsyo taki udayotsya odnovremenno svyazav i zhelezo i cianid v otdelnye prochnye kompleksy naprimer obrabotav rastvor geksacianoferrata II smesyu a a dipiridila i hlorida rtuti Pri etom poyavlyaetsya krasnoe okrashivanie vsledstvie obrazovaniya kompleksa Fe 2 displaystyle ce Fe 2 s a a dipiridilom pri odnovremennom svyazyvanii cianida v vide Hg CN 2 displaystyle ce Hg CN 2 5 Toksichnost PravitNejtralnoe veshestvo poskolku anion sm vyshe ne razlagaetsya v vode i vnutri chelovecheskogo organizma Letalnaya doza LD50 dlya krys pri priyome peroralno sostavlyaet 6400 mg kg 6 Primenenie PravitPrimenyayut pri izgotovlenii pigmentov krashenii shyolka v proizvodstve cianistyh soedinenij ferritov cvetnoj bumagi kak komponent ingibiruyushih pokrytij i pri cianirovanii stalej dlya vydeleniya i utilizacii radioaktivnogo ceziya V pishevoj promyshlennosti ferrocianid kaliya zaregistrirovan v kachestve pishevoj dobavki E536 prepyatstvuyushej slyozhivaniyu i komkovaniyu Primenyaetsya kak dobavka k povarennoj soli V Rossijskoj Federacii shiroko primenyayut pri proizvodstve produktov pitaniya soli tvorozhnyh produktov v vinodelii i prochem istochnik ne ukazan 530 dnej Geksacianoferrat II kaliya primenyaetsya v analiticheskoj himii kak reaktiv dlya obnaruzheniya nekotoryh kationov 1 Fe 3 displaystyle ce Fe 3 obrazuetsya malorastvorimyj sinij osadok berlinskoj lazuri Fe III Cl 3 K 4 Fe II CN 6 KFe III Fe II CN 6 3 KCl displaystyle ce Fe III Cl3 K4 Fe II CN 6 gt KFe III Fe II CN 6 3KCl ili v ionnoj forme Fe 3 Fe CN 6 4 Fe Fe CN 6 displaystyle ce Fe 3 Fe CN 6 4 gt Fe Fe CN 6 Poluchayushijsya geksacianoferrat II kaliya zheleza III slabo rastvorim s obrazovaniem kolloidnogo rastvora poetomu nosit nazvanie rastvorimaya berlinskaya lazur 2 Zn 2 displaystyle ce Zn 2 obrazuetsya belyj osadok geksacianoferrata II cinka kaliya 7 3 ZnCl 2 2 K 4 Fe II CN 6 K 2 Zn 3 Fe CN 6 2 6 KCl displaystyle ce 3ZnCl2 2K4 Fe II CN 6 gt K2Zn3 Fe CN 6 2v 6KCl ili v ionnoj forme 3 Zn 2 2 K 2 Fe CN 6 4 K 2 Zn 3 Fe CN 6 2 displaystyle ce 3Zn 2 2K 2 Fe CN 6 4 gt K2Zn3 Fe CN 6 2v 3 Cu 2 displaystyle ce Cu 2 iz nejtralnyh ili slabokislyh rastvorov vypadaet krasno buryj osadok geksacianoferrata II medi II 2 CuCl 2 K 4 Fe II CN 6 Cu 2 Fe CN 6 4 KCl displaystyle ce 2CuCl2 K4 Fe II CN 6 gt Cu2 Fe CN 6 v 4KCl ili v ionnoj forme 2 Cu 2 Fe CN 6 4 Cu 2 Fe CN 6 displaystyle ce 2Cu 2 Fe CN 6 4 gt Cu2 Fe CN 6 v Mozhet ispolzovatsya dlya polucheniya sinilnoj kisloty K 4 Fe CN 6 2 H 2 SO 4 H 4 Fe CN 6 2 K 2 SO 4 displaystyle ce K4 Fe CN 6 2h3SO4 gt h5 Fe CN 6 2K2SO4 3 H 4 Fe CN 6 Fe 3 C 5 C 3 N 2 12 HCN displaystyle ce 3h5 Fe CN 6 gt Fe3C 5C 3N2 12HCN Mnemonicheskoe pravilo PravitDlya togo chtoby zapomnit formulu zhyoltoj krovyanoj soli K 4 Fe CN 6 displaystyle ce K4 Fe CN 6 i ne sputat eyo s krasnoj krovyanoj solyu K 3 Fe CN 6 displaystyle ce K3 Fe CN 6 sushestvuyut mnemonicheskie pravila Chislo atomov kaliya sootvetstvuet chislu bukv v anglijskih nazvaniyah solej gold 4 bukvy to est 4 atoma kaliya zhyoltaya krovyanaya sol K 4 Fe CN 6 displaystyle ce K4 Fe CN 6 Red 3 bukvy to est 3 atoma kaliya krasnaya krovyanaya sol K 3 Fe CN 6 displaystyle ce K3 Fe CN 6 Kalij tri budet krasnyj Mnemonika postroena na igre slov tri chislo i tri povelitelnoe naklonenie glagola teret Kak izvestno pri potiranii naprimer kozhi poslednyaya krasneet Primechaniya Pravit 1 2 Zheltaya ili kroveshelochnaya sol Enciklopedicheskij slovar Brokgauza i Efrona v 86 t 82 t i 4 dop SPb 1890 1907 1 2 3 4 5 6 Nekrasov B V Osnovy obshej himii M 1973 T 2 S 360 1 2 3 Tananaev 1971 s 38 39 Tananaev 1971 s 5 F Fajgl V Anger Kapelnyj analiz neorganicheskih veshestv 1976 T 1 S 242 POTASSIUM FERROCYANIDE neopr hazard com Data obrasheniya 16 yanvarya 2016 Analiticheskie metody opredeleniya cinkaLiteratura PravitTananaev I V i dr Himiya ferrocianidov M 1971 Istochnik https ru wikipedia org w index php title Geksacianoferrat II kaliya amp oldid 113169845, Википедия, чтение, книга, библиотека, поиск, нажмите,
ХимияВсеУчебные пособияПомощь по выполнению домашних заданийПланы уроков
Искать на этом сайте
Цитата страницы
Начать эссе
значок-вопрос
Задайте вопрос
Начать бесплатную пробную версию
Скачать PDF
PDF
Цитата страницы
Цитировать
Поделиться ссылкой
Делиться
Ссылайтесь на эту страницу следующим образом:
«Сбалансируйте химическое уравнение: K4Fe(CN)6 + h3SO4 + h3O = K2SO4 + FeSO4 + (Nh5)2SO4 + CO». eNotes Editorial , 10 апреля 2011 г., https://www.enotes.com/homework-help/balance-chemical-equation-k4fe-cn-6-h3so4-h3o-253381.
По состоянию на 29 апреля 2023 г.
Ответы экспертов
Химическое уравнение, которое необходимо сбалансировать: K4Fe(CN)6 + h3SO4 + h3O = K2SO4 + FeSO4 + (Nh5)2SO4 + CO
Сделайте это следующим образом:
Рассмотрим первое соединение K4Fe(CN)6, K присутствует только в одном из соединений слева. Таким образом, вы можете записать коэффициент K2SO4 как 2. Точно так же, учитывая Fe и N, мы получаем коэффициенты FeSO4 как 1 и (Nh5)2SO4 как 3.
Так как K2SO4, FeSO4 и (Nh5)2SO4 имеют SO4, и мы получаем Всего 6 SO4, мы можем сделать коэффициент h3SO4 равным 6. (Nh5)2SO4 имеет 8 атомов H, а их 3, это дает 24 атома H. Из 6 h3SO4 мы получаем 12 атомов H, остальные или еще 12 приходятся на h30, поэтому h3O можно присвоить коэффициент 6. Наконец, в K4Fe(CN)6 имеется 6 атомов C, чтобы учесть это, присвойте CO коэффициент из 6. В итоге получаем:
Начните 48-часовую бесплатную пробную версию , чтобы получить доступ к более чем 30 000 дополнительных руководств и более чем 350 000 вопросов помощи при выполнении домашних заданий, на которые наши эксперты ответили.
Получите 48 часов бесплатного доступа
Уже зарегистрированы? Войдите здесь.
Утверждено редакцией eNotes
Задайте вопрос
Похожие вопросы
Просмотреть все
Химия
Последний ответ опубликован 21 июня 2012 г. в 16:50:16.
Сколько молей h3SO4 вступит в реакцию с 18 моль Al в химической реакции с их участием.
4 Ответы воспитателя
Химия
Последний ответ опубликован 21 июня 2012 г. в 12:56:39.ВЕЧЕРА
Алюминий реагирует с кислородом в следующей химической реакции: Al + O2 → Al2O3. Сколько молей Al2O3 образуется при взаимодействии 6,38 моль O2 и 9,15 моль Al?
2 Ответы воспитателя
Химия
Последний ответ опубликован 02 июня 2013 г. в 20:16:05.
Что удерживает атомы вместе в ионных связях?
2 Ответы воспитателя
Химия
Последний ответ опубликован 19 ноября 2012 г. в 17:41:47.
Оксид ванадия (V) используется в качестве катализатора в приведенной ниже реакции.2 SO2 (г) + O2 (г) —> 2 SO3 (г)Объясните, как оксид ванадия (V) способен снижать энергию активации и повышать скорость…
1 Ответ воспитателя
Химия
Последний ответ опубликован 23 сентября 2012 г. в 3:35:39.
Является ли CO2 + h3O ->h3CO3 окислительно-восстановительной реакцией?
1 Ответ воспитателя
Ферроцианид калия – определение, структура, свойства, использование и применение
Ферроцианид калия K 4 (Fe(CN) 6 ) используется в пищевой и фармацевтической промышленности для окрашивания. Ферроцианид калия используется в качестве красителя в сахаре, в пищевых продуктах, таких как выпечка, макаронные изделия, фрукты, желе и начинки для пирогов, молочные продукты, мясные и рыбные продукты, кофе и жевательная резинка; и в качестве компонента в лекарствах и фармацевтических препаратах, таких как вакцины, витамины и антибиотики. Он обычно используется в производстве продуктов, не содержащих сухих веществ, и пищевых красителей, но все чаще используются продукты, содержащие небольшое количество добавленного ферроцианида калия. Этот краситель не является пищевой добавкой и не влияет на безопасность пищевых продуктов. Ферроцианид калия обычно добавляют в сахар для улучшения цвета пищевого продукта. Он одобрен для использования в качестве красителя в пищевых продуктах и в качестве компонента лекарственных средств и пищевых продуктов для контроля цвета.
Преимущества
Ферроцианид калия не влияет на безопасность пищевых продуктов.
Ферроцианид калия водорастворим, устойчив к гидролизу и не образует токсичных веществ.
Высокая стабильность.
Не окрашивает рот, губы, зубы или пальцы.
Пятна можно смыть холодной водой.
Простота в обращении.
Высокая стабильность и низкая температура плавления.
Цвет является постоянным и не будет работать.
Насыщенность цвета можно варьировать, регулируя уровень pH и количество ингредиентов.
Красители с этим свойством помогают производить яркую и привлекательную выпечку.
Хорошо впитывает влагу из сахара.
Не является канцерогеном.
Не окисляется быстро и может использоваться в больших количествах.
Безопасен для детей, беременных женщин, пожилых людей и больных.
Не воспламеняется.
Низкая стоимость.
Не вступает в реакцию с большинством других пищевых продуктов.
Не изменяет вкус пищи, в которую добавляется.
Ферроцианид калия не наносит вреда окружающей среде.
Области применения
Ферроцианид калия используется в качестве красителя в пищевых продуктах и напитках, таких как варенье, желе, мармелад и разрыхлитель. Его также можно использовать в некоторых лекарствах и препаратах, содержащих кальций или железо. Он также входит в состав некоторых лечебных продуктов. Ферроцианид калия также можно использовать для окрашивания некоторых медицинских продуктов для определенных групп пациентов, например, с проблемами почек или сердца.
История
Соли ферроцианидов были впервые получены в 1840-х годах. Ферроцианид калия был произведен в 1891 году Томасом Грэмом и Альфредом Роузом из Английского фармацевтического общества. Это было первое успешное коммерческое производство ферроцианида калия.
Ферроцианид калия был впервые использован в качестве красителя в пищевой промышленности в 1890-х годах. Затем его использовали для окрашивания желатина в пищевых продуктах, таких как варенье и мармелад, чтобы отличить их от более дешевых продуктов синтетического цвета. Было получено множество патентов для защиты нового красителя.
Использование ферроцианида в качестве пищевого красителя было разрешено в 1930-х годах. Он был одобрен для использования в желе, джеме, мармеладе, конфетах и разрыхлителе.
Есть некоторые споры по поводу названия ферроцианид калия. Ее также называют «зеленой солью».
Преимущества:
Хороший краситель, так как придает продукту ярко-зеленый цвет. Он не теряет цвет при нагревании или воздействии солнечных лучей.
Имеет меньше недостатков, чем синтетический краситель.
Безопасен для использования в пищевых продуктах.
Может использоваться в широком ассортименте пищевых продуктов.
Он не летучий, поэтому нет риска обжечь рот.
Не вызывает рак.
Недорого.
О калиях ферроцианид -K
4 (Fe (CN) 6 )
Ферроцианид калия -K 4 (Fe (CN)) 6 представляет собой феррицианидную соль или калиевую соль. Это бесцветный или желтоватый твердый материал. Как соль, это соль сильной кислоты. Растворим в воде, эфире и других растворителях.
Ферроцианид калия -K 4 (Fe(CN) 6 ) используется в качестве катализатора в фотографической, биохимической и аналитической химии. Он в основном используется в качестве калиевой соли для получения ферроцианида калия — K 4 (Fe(CN)) 6 . Ферроцианид калия -K 4 (Fe(CN) 6 ) также называется феррицианид калия -K 4 (Fe(CN)) 6 , феррицианид калия -K 4 (Fe(CN) 2 ), феррицианид калия — K 4 (Fe(CN) 2 ), феррицианид калия -K 4 (Fe(CN) 2 ) и феррицианид калия -K 4 (Fe(CN) 3 ). Ферроцианид калия -K 4 (Fe(CN) 6 ) также называют дихроматом калия -K 4 Cr 2 О 7 ).
Ферроцианид калия — K 4 (Fe(CN) 6 ) используется во многих процессах в аналитической лаборатории для обнаружения и определения металлов. Он используется в аналитической химии для определения следовых количеств меди и железа, для определения железа в стали и для анализа меди и железа. Он используется для обнаружения присутствия меди в железной руде, определения железа и меди в алюминии атомно-абсорбционным методом, а также для идентификации и определения меди в растворе. Применяется в аналитической химии для определения мышьяка, кадмия, свинца и цинка, при определении мышьяка и свинца в руде, свинца в стали и стали всех марок.
Широко известно использование ферроцианида калия — K 4 (Fe(CN) 6 ) в качестве индикатора в методике титриметрического осаждения. Реакцию проводят при низких значениях рН в присутствии различных оснований, избыток основания определяют титрованием кислотой. Также проводят осаждение ферроцианидом калия — К 4 (Fe(CN) 6 ) в присутствии различных металлов. Осаждение железа (II) ферроцианидом калия — К 4 (Fe(CN) 6 ) используется для идентификации и определения железа в железной руде, определения железа в стали и для идентификации меди в алюминиевом сплаве. Осаждение алюминия в присутствии ферроцианида калия -К 4 (Fe(CN) 6 ) может быть использовано для идентификации и определения алюминия в алюминиевом сплаве.
Простая процедура экстракции для определения алюминия в диапазоне 5-70 мкг/г с использованием 2-(п-толуолсульфокислоты)-3,6-диамино-9-этилкарбазол — описан DAEC. Результаты, полученные методом экстракции и хелатирования, согласуются с результатами, полученными методом электротермической атомно-абсорбционной спектрометрии.
Ферроцианид калия определяется как неорганическое соединение, имеющее химическую формулу K 4 (Fe(CN) 6 ) (формула ферроцианида калия). Ферроцианид калия иначе называют желтым пруссатом калия, который представляет собой желтый кристалл. Он был сделан из роговых или шерстяных обрезков, перемешивающих горячий карбонат калия с помощью железного стержня. Соединение ферроцианида калия можно использовать в некоторых процессах получения железа в качестве проявителя и в качестве добавки для щелочных пиропроявителей.
Гексацианоферрат калия (II), желтая пруссия калия — другие названия ферроцианида калия.
Структура ферроцианида калия –K
4 (Fe(CN) 6 )
Структуру ферроцианида калия можно представить следующим образом:
(Изображение будет добавлено позже)
в важные свойства ферроцианида калия
Свойства ферроцианида калия
Название IUPAC
Ферроцианид калия
Химическая формула
K 4 (Fe(CN) 6 )
Молекулярный вес или молярная масса
368,35 г/моль
Плотность
1,85 г/см³
90 330
Температура плавления
300 °C
Температура кипения
400 °C
Физические свойства ферроцианида калия – K 9008 9 4
(Fe(CN) 6 )
Внешний вид
Светло-желтые кристаллические гранулы
Запах
Без запаха
90 330
Акцептор водородных связей
12
Ковалентно-связанный блок
10
Растворимость
Нерастворим в эфире, этаноле
Сложность
127
9033 4
Химические свойства ферроцианида калия – K
4 (Fe(CN) 6 )
Ферроцианид калия дополнительно реагирует с серной кислотой в водной форме сульфата железа ели, сульфат калия, окись углерода и сульфат аммония. Химическую реакцию можно представить следующим образом:
Ферроцианид калия можно использовать в процессах отпуска стали и гравировки. Он также используется в производстве пигментов и в качестве химического реагента.
Небольшое количество гидрохинона и пиропроявителей способствует уменьшению запотевания и увеличению плотности.
Он также используется в производстве цианида калия, который может широко использоваться в добыче золота.
Кроме того, Ferro Cyanogen производит большинство соединений металлов, нерастворимых в воде, некоторые из которых имеют очень характерные цвета. Он служит тестом для соединений железа и меди.
Применение ферроцианида калия
Важные области применения ферроцианида калия можно перечислить следующим образом:
Ферроцианид калия находит несколько нишевых применений в промышленности. Родственная натриевая соль широко используется в качестве противослеживающего агента как для поваренной соли, так и для дорожной соли. Ферроцианиды натрия и калия также могут быть использованы при выделении меди из молибденовых руд и очистке олова. Соединение ферроцианида калия используется в производстве лимонной кислоты и вина.
Также используется в кормлении животных.
Ферроцианид калия используется для определения концентрации перманганата калия в лаборатории, соединения, часто используемого при титровании по окислительно-восстановительным реакциям. Соединение ферроцианида калия используют в смеси с фосфатно-буферным раствором и феррицианидом калия для получения буфера для бета-галактозидазы. Его можно использовать для расщепления X-Gal, создавая ярко-синюю визуализацию, где антитело (иначе другая молекула), конъюгированное с Beta-gal, связано со своей мишенью. Кроме того, при реакции с Fe (3) он дает берлинскую лазурь. Поэтому его можно использовать в качестве реагента для идентификации железа в лабораториях.
Соединение ферроцианида калия используется в качестве удобрения для растений.
До 1900 г. н.э., до изобретения процесса Кастнера, соединение ферроцианида калия было важным источником цианидов щелочных металлов. Цианид калия был получен путем разложения ферроцианида калия в этом историческом процессе. Химическая реакция может быть представлена следующим образом:
Острая пероральная токсичность ферроцианида калия указана как 5000 мг/кг. Это означает 350 г для человека весом 70 кг. Хотя молекулы состоят из ионов цианида, они более прочно связаны с атомом железа и остаются с железом на протяжении всей пищеварительной системы человека.
Количество, добавляемое в поваренную соль для предотвращения слеживания, составляет порядка 20 мг/кг хлорида натрия, а это означает, что нам придется съесть целый грузовик поваренной соли, чтобы достичь уровня токсичности ферроцианида калия. Конечно, соль убьет нас задолго до ферроцианида калия.
Наличие на рынке ферроцианида калия
Соединения ферроцианида калия существуют в виде светло-желтых кристаллов без запаха, растворимых в воде, но нерастворимых в спиртах. Это соединение также называют либо желтым пруссатом гексацианоферрата (II) калия, либо калием. Ферроцианид калия производится из гидроксида калия и ферроцианида водорода. Это соединение не считается токсичным, но оно опасно при смешивании с другими химическими веществами или при нагревании с ними.
Что произойдет, если разделить квадратный корень на 2? – Обзоры Вики
Чему равно число 3, деленное на корень 3? Это не может быть упрощено без использования калькулятора, и в этом случае ответ 1.732050807569 .
Дополнительно Каково значение 2 корня 2? Значение 2√2 равно 2.828. Следовательно, 2√2 = 2(1.414) = 2.828.
Как найти корень 2? √2 = 1.41421356237309504880168872420969807856967187537694…
В настоящее время значение корня 2 вычисляется до 10 триллионов цифр. Для общего использования его значение усекается и используется как 1.414 для упрощения вычислений. Дробь 99/70 также иногда используется как значение √2.
Как вы делите радикальные примеры?
Можно ли упростить радикал 3 3?
Обратите внимание, что как только вы рационализируете дробь, знаменатель больше не будет иррациональным. Также имейте в виду, что вы не меняли значение упрощенной дроби. Поскольку √3√3 равно 1, вы просто изменили способ записи. Значение упрощенная дробь остается прежней.
И сколько 2 разделить на 3 как дробь? Ответ: 2 разделить на 3 как дробь 2/3.
Делитель представлен знаменателем, то есть 3.
Как упростить радикал 3?
Где корень из 3? Квадратный корень из 3 равен 1.732.
Как упростить 2 на 2?
Как я могу получить значение root 3?
Значение √3 примерно равно 1.732.
Что такое куб числа 2? Кубы и кубические корни Список от 1 до 15
Число
Куб (а 3 )
Кубический корень ∛a
2
8
1.260
3
27
1.442
4
64
1.587
5
125
1. 710
Почему √ 2 иррациональное число? В частности, греки обнаружили, что диагональ квадрата со стороной в 1 единицу имеет диагональ, длина которой не может быть рациональной. По теореме Пифагора длина диагонали равна квадратному корню из 2. Итак, квадратный корень из 2 иррационален!
Как делить?
Можно ли разделить радикал на целое число? Радикал нельзя разделить на целое число, если он не упрощен..
Как решить радикал 2?
Каковы радикальные правила? Число внутри подкоренного знака называется подкоренным. Чай все выражение называют радикалом. … Правила радикалов.
Правила радикалов Если n — положительное целое число больше 1, а a и b — положительные действительные числа, тогда,
2. Правило продукта
п√ аб = п√ а · п√ б
Как упростить радикал, разделенный на радикал?
Какие ключевые понятия необходимы для умножения и деления радикалов? Прежде чем выполнять какое-либо умножение или деление, нам нужно убедиться, что индексы одинаковы. Умножение радикалов — это просто перемножение чисел внутри знака радикала, подкоренных, вместе. При делении радикалов можно поместите числитель и знаменатель в один и тот же квадратный корень.
Мэтуэй | Популярные задачи
1
Оценка с использованием заданного значения
квадратный корень из 50
2
Оценка с использованием заданного значения
квадратный корень из 45
3
Оценить
5+5
4
Оценить
7*7
5
Найти простую факторизацию
24
6
Преобразование в смешанный номер
52/6
7
Преобразование в смешанный номер
93/8
8
Преобразование в смешанный номер
34/5
9
График
у=х+1
10
Оценка с использованием заданного значения
квадратный корень из 128
11
Найдите площадь поверхности
сфера (3)
12
Оценить
54-6÷2+6
13
График
г=-2x
14
Оценить
8*8
15
Преобразование в десятичное число
5/9
16
Оценка с использованием заданного значения
квадратный корень из 180
17
График
у=2
18
Преобразование в смешанный номер
7/8
19
Оценить
9*9
20
Решите для C
С=5/9*(Ф-32)
21
Упростить
1/3+1 1/12
22
График
у=х+4
23
График
г=-3
24
График
х+у=3
25
График
х=5
26
Оценить
6*6
27
Оценить
2*2
28
Оценить
4*4
29
Оценить
1/2+(2/3)÷(3/4)-(4/5*5/6)
30
Оценить
1/3+13/12
31
Оценка
5*5
32
Решить для d
2д=5в(о)-вр
33
Преобразование в смешанный номер
3/7
34
График
г=-2
35
Найдите склон
у=6
36
Преобразование в проценты
9
37
График
у=2х+2
38
92+5х+6=0
41
Преобразование в смешанный номер
1/6
42
Преобразование в десятичное число
9%
43
Найти n
12н-24=14н+28
44
Оценить
16*4
45
Упростить
кубический корень из 125
46
Преобразование в упрощенную дробь
43%
47
График
х=1
48
График
у=6
49
График
г=-7
50
График
у=4х+2
51
Найдите склон
у=7
52
График
у=3х+4
53
График
у=х+5
54
График
92-9=0
58
Оценка с использованием заданного значения
квадратный корень из 192
59
Оценка с использованием заданного значения
квадратный корень из 25/36
60
Найти простую факторизацию
14
61
Преобразование в смешанный номер
7/10
62
Решите для
(-5а)/2=75
63
Упростить
х
64
Оценить
6*4
65
Оценить
6+6
66
Оценить
-3-5
67
Оценить
-2-2
68
Упростить
квадратный корень из 1
69
Упростить
квадратный корень из 4
70
Найди обратное
1/3
71
Преобразование в смешанный номер
20. 11.
72
Преобразование в смешанный номер
7/9
73
Найти LCM
11, 13, 5, 15, 14
, , , ,
92+2x-8=0
76
График
3x+4y=12
77
График
3x-2y=6
78
График
у=-х-2
79
График
у=3х+7
80
Определить, является ли многочлен
2x+2
81
График
у=2х-6
82
График
у=2х-7
83
График
у=2х-2
84
График
у=-2х+1
85
График
у=-3х+4
86
График
у=-3х+2
87
График
у=х-4
88
Оценить
(4/3)÷(7/2)
89
График
2x-3y=6
90
График
х+2у=4
91
График
х=7
92
График
х-у=5
93
Решение с использованием свойства квадратного корня 92-2x-3=0
95
Найдите площадь поверхности
конус (12)(9)
96
Преобразование в смешанный номер
3/10
97
Преобразование в смешанный номер
7/20
92)
Как делить квадратные корни
Все математические ресурсы GRE
13 диагностических тестов
452 практических теста
Вопрос дня
Карточки
Learn by Concept
← Предыдущая 1 2 Следующая →
GRE Math Help »
Арифметика »
Базовое возведение в квадрат / квадратные корни »
Квадратные корни и операции »
Как разделить квадратный корень
Что из следующего равно
Возможные ответы:
Правильный ответ:
Объяснение:
Затем мы умножаем нашу дробь на , потому что мы не можем оставить радикал в знаменателе. Это дает нам . Наконец, мы можем упростить нашу дробь, разделив 3, оставив нам
Сообщить об ошибке
Упростить:
Возможные ответы:
Правильный ответ:
Пояснение:
Объединим два радикала в один радикал и упростим.
Помните, что при делении показателей одного и того же основания просто вычтите степень.
Окончательный ответ .
Сообщить об ошибке.
Объяснение:
Если мы умножим верх и низ на , мы ничего не получим, так как получится: . Вместо этого давайте перемножим.
Затем возведите обе стороны в квадрат, чтобы избавиться от радикала.
Разделите обе части на .
Причина, по которой отрицательное значение не является ответом, заключается в том, что отрицательное значение в радикале является мнимым числом.
Сообщить об ошибке
Рационализировать знаменатель:
Возможные ответы:
Правильный ответ:
Объяснение:
Нам не нужны радикалы в знаменателе. Чтобы избавиться от радикалов, просто умножьте верх и низ на этот радикал.
Сообщить об ошибке
Упрощение:
Возможные ответы:
Правильный ответ:
1
1
Объяснение:
Есть два метода, которые мы можем использовать, чтобы упростить эту дробь:
Метод 1:
Разложить числитель на множители:
Помните, нам нужно вынести на множители правильные квадраты.
Способ 2:
Вы можете объединить дробь в один большой квадратный корень.
Тогда можно упростить дробь.
Сообщить об ошибке
Упростить:
Возможные ответы:
Правильный ответ:
Пояснение:
Разложим квадратные корни.
Затем умножьте числитель и знаменатель на , чтобы избавиться от радикала в знаменателе.
Сообщить об ошибке
Что из следующего эквивалентно ?
Возможные ответы:
Правильный ответ:
Объяснение:
Мы определенно можем исключить некоторые варианты ответов. и не имеют смысла, потому что у нас есть иррациональное число. Далее умножим числитель и знаменатель на . Когда мы упрощаем радикальные дроби, мы пытаемся исключить радикалы, но здесь мы пойдем назад.
, таков и ответ.
Сообщить об ошибке
Рационализировать знаменатель и упростить:
Возможные ответы:
Правильный ответ:
Объяснение:
Нам не нужны радикалы в знаменателе. Чтобы избавиться от радикалов, просто умножьте числитель и знаменатель на этот радикал.
Не забудьте распределить радикал в числителе при умножении.
Это может быть ответ; однако числитель можно упростить. Разберем квадраты.
Наконец, если мы вычтем a , мы получим:
Сообщить об ошибке
Упрощение:
Возможные ответы:
Правильный ответ:
Пояснение:
Давайте избавимся от радикалов в знаменателе каждой отдельной дроби.